Tuesday 9 January 2018

TEST - 1 (MOCK TEST - 1)
www.insightsias.com 1
© Insights Active Learning| All rights reserved. You may not reproduce, distribute or exploit the contents in any form without written
permission by copyright owner. Copyright infringers may face civil and criminal liability
1 Social Entrepreneurship is running
A. Any non-profit business
B. An ecologically sustainable business
C. An inclusive business that has participation from major social groups and local community
D. A business that makes its money in a socially responsible way
Your Answer :
Correct Answer : D
Answer Justification :
Learning: It is the use of the techniques by start-up companies and other entrepreneurs to develop,
fund and implement solutions to social, cultural, or environmental issues.
Muhammad Yunus (Grameen Bank) was a famous social entrepreneur. Another example can be a
crowdfunding internet platform like https://milaap.org
These ventures are not necessarily formed to reinvest all profits into the communities.
So, these firms apply commercial strategies to maximize improvements in human and
environmental well-being.
Q Source: Often seen in news: A recent speech by the PM
TEST - 1 (MOCK TEST - 1)
www.insightsias.com 2
© Insights Active Learning| All rights reserved. You may not reproduce, distribute or exploit the contents in any form without written
permission by copyright owner. Copyright infringers may face civil and criminal liability
2 Which of these are the chief difference(s) between Mesolithic and Neolithic cultures?
1. Hunting-gathering pattern of life began not before the Neolithic period.
2. Mesolithic cultures did not practice domestication of animals unlike Neolithic people.
3. The use of bow and arrow cannot be found in Mesolithic culture, unlike Neolithic culture.
4. Evidence of pottery is absent from the Neolithic culture but is markedly found in Mesolithic culture.
Select the correct answer using the codes below.
A. 3 and 4 only
B. 2 only
C. 1 and 3 only
D. None of the above
Your Answer : B
Correct Answer : D
Answer Justification :
Justification: Statement 1 and 3: Hunting-gathering pattern of life was prevalent since Old stone
age to Mesolithic and Neolithic.
However, there seems to have been a shift from big animal hunting to small animal hunting and
fishing. This is how bows and arrows were used in this period. Both 1 and 3 are thus wrong.
Statement 2: Domestication of animals, horticulture and primitive cultivation started during
Mesolithic period itself.
However, during Neolithic period, domestication of sheep, goats and cattle was widely prevalent.
Cattle were used for cultivation and for transport.
Statement 4: During Neolithic age, wheels were used to make pottery. Pottery was used for cooking
as well as storage of food grains. So, 4 is wrong.
Learning: During the Neolithic phase, the cultivation of plants and domestication of animals led to
the emergence of village communities based on sedentary life.
There was a great improvement in technology of making tools and other equipments used by
man. Stone tools were now polished.
Mud brick houses were built instead of grass huts.
Large urns were used as coffins for the burial of the dead.
Wheat, barely, rice, millets were cultivated in different areas at different points of time. Rice
cultivation was extensive in eastern India.
The people of Neolithic Age used clothes made of cotton and wool
TEST - 1 (MOCK TEST - 1)
www.insightsias.com 3
© Insights Active Learning| All rights reserved. You may not reproduce, distribute or exploit the contents in any form without written
permission by copyright owner. Copyright infringers may face civil and criminal liability
Q Source: Page 13-14: TN 11th Standard History Textbook
56154 - ROHIT YADAV - rohit.kshaurya@gmail.com - 9410878797
TEST - 1 (MOCK TEST - 1)
Total Marks : 200
Mark Scored :
www.insightsias.com 4
© Insights Active Learning| All rights reserved. You may not reproduce, distribute or exploit the contents in any form without written
permission by copyright owner. Copyright infringers may face civil and criminal liability
3 The Government of India has recently announced the sale of “Government of India Floating Rate Bonds
2024”. With reference to government securities, consider the following:
1. Floating Rate Bonds are sold by the Reserve Bank of India.
2. RBI issues Cash Management Bills (CMBs) to meet long-term borrowing needs of the Government
of India.
3. Treasury bills are short term debt instruments issued by the Government of India.
Select the correct answer using the codes below.
A. 1 and 2 only
B. 1 and 3 only
C. 2 only
D. 2 and 3 only
Your Answer : B
Correct Answer : B
Answer Justification :
Concept: A Government security is a tradable instrument issued by the Central Government or the
State Governments.
It acknowledges the Government’s debt obligation (to raise money from the market for
various purposes).
Such securities are short term (usually called treasury bills, with original maturities of less
than one year) or long term (usually called Government bonds or dated securities with
original maturity of one year or more).
Besides providing a return in the form of coupons (interest), Government securities offer the
maximum safety as they carry the Sovereign’s commitment for payment of interest and
repayment of principal.
Justification: Statement 1: There are two types of bonds:
Fixed Rate Bonds – These are bonds on which the coupon rate is fixed for the entire life of the
bond. Most Government bonds are issued as fixed rate bonds.
Floating Rate Bonds – Floating Rate Bonds are securities which do not have a fixed coupon rate.
The coupon is re-set at pre-announced intervals (say, every six months or one year) by adding a
spread over a base rate. These are issued by RBI.
Statement 2: Cash Management Bills (CMBs), are a relatively new short-term instrument to meet
the temporary mismatches in the cash flow of the Government. The CMBs have the generic
character of T-bills but are issued for maturities less than 91 days.
Statement 3: Treasury bills or T-bills, which are money market instruments, are short term debt
instruments issued by the Government of India and are presently issued in three tenors, namely, 91
day, 182 day and 364 day.
TEST - 1 (MOCK TEST - 1)
www.insightsias.com 5
© Insights Active Learning| All rights reserved. You may not reproduce, distribute or exploit the contents in any form without written
permission by copyright owner. Copyright infringers may face civil and criminal liability
Q Source: RBI Bond sales
TEST - 1 (MOCK TEST - 1)
www.insightsias.com 6
© Insights Active Learning| All rights reserved. You may not reproduce, distribute or exploit the contents in any form without written
permission by copyright owner. Copyright infringers may face civil and criminal liability
4 Ashokan inscriptions remain valuable sources for the study of Asoka and the Mauryan Empire. This is
because
1. His wooden pillars heralded a new architectural era because most of the monuments before his
period were made of stone.
2. Pillar edicts give a summary of his efforts to promote the Dhamma within his kingdom and outside.
3. They often deal with instructions given to his officials which is informative of Mauryan polity.
Select the correct answer using the codes below.
A. 2 and 3 only
B. 1 and 3 only
C. 1 and 2 only
D. 2 only
Your Answer : A
Correct Answer : A
Answer Justification :
Justification: Statement 1: The monuments before the period of Asoka were mostly made of wood
and therefore perished.
The use of stone started from the time of Asoka. Even of the numerous monuments of Asoka, only a
few have remained.
His palace and monasteries and most of his stupas have disappeared. The only remaining stupa is at
Sanchi.
Statement 2: The state and spread of Dhamma gives crucial information about the social life at the
time of Ashoka.
Statement 3: Ashoka often gave instructions to his Dhamma mahamahattas to spread Dhamma.
These instructions, their style etc were recorded on the edicts.
Learning: Ashokan inscriptions (first deciphered by James Princep) are written in Pali language
and in some places Prakrit was used.
The Brahmi script was employed for writing. In the northwestern India Asokan inscriptions were
found in Karoshti script.
There are fourteen Major Rock Edicts. The two Kaling Edicts are found in the newly conquered
territory. The major pillar Edicts were erected in important cities. There are minor Rock Edicts and
minor pillar Edicts.
Q Source: Page 60-61: TN 11th Standard History Textbook
TEST - 1 (MOCK TEST - 1)
www.insightsias.com 7
© Insights Active Learning| All rights reserved. You may not reproduce, distribute or exploit the contents in any form without written
permission by copyright owner. Copyright infringers may face civil and criminal liability
5 Which of the following is the closest analogy of the Indian Constitution?
A. Rulebook
B. Anthology
C. Dictionary
D. Social Register
Your Answer : A
Correct Answer : A
Answer Justification :
Justification: Option A: The Indian Constitution lays down the basic rules or laws that have to be
followed by everyone. These laws are for both the government and the people. Hence, A is
appropriate.
Option B: An anthology is a collection of literary works, originally of poems, which constitution is
not.
Option C: Constitution is not a mere dictionary of political and legal terms, it is much more than
that.
Option D: Social Register (SR) is a directory of people from certain social classes. Constitution is for
everyone. It isn’t a directory.
Q Source: Improvisation: Page 39: NCERT Class VI Social and Political Life - I
TEST - 1 (MOCK TEST - 1)
www.insightsias.com 8
© Insights Active Learning| All rights reserved. You may not reproduce, distribute or exploit the contents in any form without written
permission by copyright owner. Copyright infringers may face civil and criminal liability
6 ‘JIGYASA’, recently seen in news is a
A. Interstellar exploration programme of ISRO
B. Student- scientist connect programme
C. Earth core drilling programme of Ministry of Earth Sciences
D. Biodiversity documentation programme in the Western Ghats
Your Answer : B
Correct Answer : B
Answer Justification :
Learning: Council of Scientific and Industrial Research (CSIR), has joined hands with Kendriya
Vidyalaya Sangathan (KVS) to implement this programme.
The program will also enable the students and teachers to practically live the theoretical concepts
taught in science by visiting CSIR laboratories and by participating in mini-science projects. The
model of engagement includes:
Student Residential Programmes;
Scientists as Teachers and Teachers as Scientists;
Lab specific activities / Onsite Experiments;
The focus is on connecting school students and scientists so as to extend student’s classroom
learning with that of a very well-planned research laboratory based learning.
Q Source: http://pib.nic.in/newsite/PrintRelease.aspx?relid=167194
TEST - 1 (MOCK TEST - 1)
www.insightsias.com 9
© Insights Active Learning| All rights reserved. You may not reproduce, distribute or exploit the contents in any form without written
permission by copyright owner. Copyright infringers may face civil and criminal liability
7 Consider the following statements. Assertion (A): Every elector at the Presidential election is bound by
the voting instructions issued by the whip of the respective political party. Reason (R): Anti-defection
provisions are enforced by the constitution under the Tenth Schedule.
In the context of the above, which of these is correct?
A. A is correct, and R is an appropriate explanation of A.
B. A is correct, but R is not an appropriate explanation of A.
C. A is correct, but R is incorrect.
D. A is incorrect, but R is correct.
Your Answer : C
Correct Answer : D
Answer Justification :
Concept: Defection is defined as “to abandon a position or association, often to join an opposing
group” which essentially describes a situation when a member of a particular party abandons his
loyalty towards that party and provide his support (in the form of his vote or otherwise) to another
party.
To avoid such a situation (about which you’ll read later in Laxmikanth and 11th NCERT), an antidefection
provision was added in the constitution. If a member of a political party disobeys party
directions concerning matters such as voting on a bill or for a candidate, this amounts to defection.
The member is then liable to be punished by the party.
Justification: However, the ECI has clarified recently that voting or not voting as per his/her own
free will at the Presidential election and the electors are at liberty to vote or not to vote at the
Presidential election as per their own free will and choice.
This will equally apply to the political parties and they are free to canvas or seek votes of
electors for any candidate or requesting or appealing to them to refrain from voting.
However, the political parties cannot issue any direction or whip to their members to vote in a
particular manner or not to vote at the election leaving them with no choice, as that would
tantamount to the offence of undue influence within the meaning of section 171C of the IPC.
This is because, as per the ECI, voting at election to the office of President is different from
voting by a member of Parliament or State Legislature inside the House.
Q Source: RM/ RS: Release ID :167193
TEST - 1 (MOCK TEST - 1)
www.insightsias.com 10
© Insights Active Learning| All rights reserved. You may not reproduce, distribute or exploit the contents in any form without written
permission by copyright owner. Copyright infringers may face civil and criminal liability
8 With reference to unique BRICS countries initiatives, consider the following statements:
1. BRICS Network University (NU) is an open educational project and does not involve establishment
of rigid organizational forms such as a secretariat.
2. BRICS Think Tanks Council (BTTC) comprises research organizations from BRICS countries
including Observer Research Foundation (ORF) from India.
Which of the above is/are correct?
A. 1 only
B. 2 only
C. Both 1 and 2
D. None
Your Answer :
Correct Answer : C
Answer Justification :
Justification: Recently at the 5th BRICS Education ministers meeting, Beijing declaration on
education was adopted which advocated support for both these organizations.
Statement 1: NU is a consortium of the universities, elaborating Master and PhD programmes in
different disciplines. It was established in 2015 in BRICS meeting at Moscow.
NU follows principles of:
Openness, which enables various universities to join the network respecting national criteria.
Focus on educational programmes that can be supplemented with network research and
innovation projects.
There is also a BRICS Universities League which is a consortium of leading research universities
from BRICS countries.
Statement 2: The BTTC (established in 2013) comprises the Institute for Applied Economic
Research (IPEA) (Brazil), the National Committee for BRICS Research (NRC/BRICS) (Russia), the
Observer Research Foundation (ORF) etc.
The main purposes of its establishment was to form a platform for the exchange of ideas and
present policy recommendations for BRICS leaders.
Q Source: Beijing declaration on Education: 5th July 2017:
http://pib.nic.in/newsite/PrintRelease.aspx?relid=167154
TEST - 1 (MOCK TEST - 1)
www.insightsias.com 11
© Insights Active Learning| All rights reserved. You may not reproduce, distribute or exploit the contents in any form without written
permission by copyright owner. Copyright infringers may face civil and criminal liability
9 With reference to the Sangam age, Panar and Viraliyar were
A. Temple guards
B. Bull fighters
C. Singing bards
D. Charity donors
Your Answer :
Correct Answer : C
Answer Justification :
Learning: Poetry, music and dancing were popular among the people of the Sangam age. Liberal
donations were given to poets by the kings, chieftains and nobles.
The royal courts were crowded with singing bards called Panar and Viraliyar. They were
experts in folk songs and folk dances.
The arts of music and dancing were highly developed. A variety of Yazhs and drums are
referred to in the Sangam literature.
Dancing was performed by Kanigaiyar. Koothu was the most popular entertainment of the
people.
Q Source: Page 91: TN 11th Standard History Textbook
TEST - 1 (MOCK TEST - 1)
www.insightsias.com 12
© Insights Active Learning| All rights reserved. You may not reproduce, distribute or exploit the contents in any form without written
permission by copyright owner. Copyright infringers may face civil and criminal liability
10 The 600-year-old Walled City of Ahmedabad was recently declared a World Heritage City. With
reference to the World Heritage Cities Programme and associated initiatives, consider the following:
1. It is a thematic programmes formally approved and monitored by the World Heritage Committee.
2. The programme has provisions for technical assistance to States Parties for the implementation of
new approaches and schemes for urban heritage conservation.
3. IUCN has set up the ‘Historic Urban Landscape initiative’ which is an international working group
comprising, inter alia, UNESCO as an advisory body to the World Heritage Convention.
4. The Organization of World Heritage Cities (OWHC) is an inter-governmental organization of the
municipal corporations of cities in which sites of the UNESCO World Heritage list are located.
Select the correct answer using the codes below.
A. 1 and 2 only
B. 3 and 4 only
C. 1, 2 and 3 only
D. 1, 2, 3 and 4
Your Answer :
Correct Answer : A
Answer Justification :
Justification: Statement 1 and 2: It aims to assist States Parties in the challenges of protecting and
managing their urban heritage. The programme is structured along a two-way process, with 1) the
development of a theoretical framework for urban heritage conservation, and 2) the provision of
technical assistance to States Parties for the implementation of new approaches and schemes.
Statement 3: As part of this policy process of conservation, UNESCO (not IUCN) set up the Historic
Urban Landscape initiative, an international working group comprising ICOMOS, IUCN and
ICCROM (as Advisory Bodies to the 1972 World Heritage Convention) and other partner
organizations, including UN-Habitat etc.
Statement 4: OWHC is an international non-profit, non-governmental organization of 250 cities in
which sites of the UNESCO World Heritage list are located.
Learning: It is first Indian city get the honour. The Walled City of Ahmedabad is located on the
eastern banks of Sabarmati river.
It was founded by Sultan Ahmed Shah I of Gujarat Sultanate in in 1411.
It remained the capital of the Gujarat Sultanate and later important political and commercial
centre of Gujarat.
It presents a rich architectural heritage from the sultanate period, notably the Bhadra citadel,
walls and gates of the Fort city and numerous mosques and tombs, as well as important Hindu
and Jain temples of later periods.
Q Source: Improvisation:
http://www.thehindu.com/news/national/other-states/ahmedabad-declared-indias-first-heritage-cityTEST
- 1 (MOCK TEST - 1)
www.insightsias.com 13
© Insights Active Learning| All rights reserved. You may not reproduce, distribute or exploit the contents in any form without written
permission by copyright owner. Copyright infringers may face civil and criminal liability
by-unesco/article19244478.ece
TEST - 1 (MOCK TEST - 1)
www.insightsias.com 14
© Insights Active Learning| All rights reserved. You may not reproduce, distribute or exploit the contents in any form without written
permission by copyright owner. Copyright infringers may face civil and criminal liability
11 What argument(s) would you advance to justify that the position of women had become miserable
during the Gupta period?
1. Women were prohibited from studying religious texts.
2. The practice of Swyamvara was given up and Manusmriti suggested early marriage for girls.
Which of the above is/are correct?
A. 1 only
B. 2 only
C. Both 1 and 2
D. None
Your Answer : A
Correct Answer : C
Answer Justification :
Justification: Statement 1: This ensured the domination of men over women.
Statement 2: Swaymvara was the practice of women choosing their own partners by way of a
competition/selection. When this was given up, women lost an important right. Manusmriti (law
book) further entrenched patriarchal rules to worsen the position of women.
Learning: During the Gupta dynasty period the male members of a family were entitled to inherit
property left behind by their fathers. However, such right of inheritance was not given to women or
girls.
The power of inheritance enabled the male members of a family to exercise dominance over the
women.
Q Source: Page 104: TN 11th Standard History Textbook
TEST - 1 (MOCK TEST - 1)
www.insightsias.com 15
© Insights Active Learning| All rights reserved. You may not reproduce, distribute or exploit the contents in any form without written
permission by copyright owner. Copyright infringers may face civil and criminal liability
12 The year of 2017 marks 25 years of dialogue partnership between India and ASEAN and 15 years of
summit-level interaction. With reference to India-ASEAN relations, consider the following:
1. India's relationship with ASEAN is a key pillar of our Act East Policy.
2. AIFTA is a free trade area among the member states of the ASEAN and India.
3. India and ASEAN are yet to upgrade their relationship to a ‘strategic partnership’.
Select the correct answer using the codes below.
A. 2 only
B. 1 and 3 only
C. 1 and 2 only
D. 3 only
Your Answer : C
Correct Answer : C
Answer Justification :
Justification: Statement 1: India’s commitment to strengthen its ties with ASEAN countries is
reflected by its ‘Look East Policy’ becoming ‘Act East Policy.’ Three Cs are the focus -Commerce,
Connectivity and Culture.
Statement 2: ASEAN became ASEAN Plus Six with additional countries: Australia, New Zealand and
India. Codification of the relations between these nations has seen progress through the
development of the Regional Comprehensive Economic Partnership (RCEP), a proposed free-trade
agreement involving the 16 countries of ASEAN plus six.
Statement 3: India and ASEAN had upgraded their relationship to a strategic partnership in 2012.
ASEAN consists of 10-member countries- Brunei, Cambodia, Indonesia, Laos, Malaysia, Myanmar,
Philippines, Singapore, Thailand and Vietnam.
It is a regional organisation that aims to promote intergovernmental cooperation and economic
integration amongst its members.
Q Source: 25 years of Dialogue partnership between India-ASEAN
TEST - 1 (MOCK TEST - 1)
www.insightsias.com 16
© Insights Active Learning| All rights reserved. You may not reproduce, distribute or exploit the contents in any form without written
permission by copyright owner. Copyright infringers may face civil and criminal liability
13 Universal suffrage (voting rights to all eligible) was granted to women all over India in
A. 1950 once Indian republic was formed
B. Morley-Minto Act
C. Government of India Act, 1935
D. 1939 General elections
Your Answer : B
Correct Answer : A
Answer Justification :
Justification & Learning: First local effort was made in Madras, which granted women’s suffrage
in 1921, but only to those men and women who owned land property according to British
administration's records. This was neither Universal nor equitable.
In 1950, universal suffrage granted voting rights to all women. This is enshrined in Article 326 in
our constitution.
Q Source: Improvisation: Page 32: NCERT Class VI Social and Political Life - I
TEST - 1 (MOCK TEST - 1)
www.insightsias.com 17
© Insights Active Learning| All rights reserved. You may not reproduce, distribute or exploit the contents in any form without written
permission by copyright owner. Copyright infringers may face civil and criminal liability
14 Karla is one of the most famous centres of early rock-cut architecture and has received much
attention of scholars and commoners alike. What is important about Karla caves?
1. The grand Chaitya griha of Karla is the largest of all the chaita-grihas of India.
2. The rock-cut cave was completed without accepting any financial donation from individuals apart
from their voluntary labour.
Which of the above is/are correct?
A. 1 only
B. 2 only
C. Both 1 and 2
D. None
Your Answer :
Correct Answer : A
Answer Justification :
Justification: Statement 1: The chaityagriha at Karla is the biggest of its type in the whole of India.
The hall measures around thirty eight m deep from door to back. The object of worship is the stupa
at the rear end of the chaityagriha.
Statement 2: The caves were caused from the donations and support of a group of assorted
individuals. This includes a prince of Maharathi family; merchants, monks etc.
The inscriptions of Usavadata and Vasisthiputra Pulumavi mention donation of land to the Veluraka
Samgha for this.
Q Source: Page 105: NCERT Class VI Our Pasts - I
TEST - 1 (MOCK TEST - 1)
www.insightsias.com 18
© Insights Active Learning| All rights reserved. You may not reproduce, distribute or exploit the contents in any form without written
permission by copyright owner. Copyright infringers may face civil and criminal liability
15 With reference to the National Disaster Response Force (NDRF), consider the following statements:
1. President of India serves as the Director General (DG) of NDRF.
2. NDRF was established under the Disaster Management Act, 2005.
3. It is housed directly under the Prime Minister’s Office and functions from there.
Select the correct answer using the codes below.
A. 2 only
B. 1 and 3 only
C. 1 only
D. 2 and 3 only
Your Answer : D
Correct Answer : A
Answer Justification :
Justification: NDRF is India’s elite disaster mitigation combat force established under Disaster
Management Act, 2005
Statement 1: Senior IPS officer Sanjay Kumar was recently appointed as Director General (DG) of
National Disaster Response Force (NDRF).
Statement 3: It functions under Union Ministry of Home Affairs. It is mandated to undertake special
disaster response, relief, rescue operations and combat roles independently in the case of an event
of any disaster (natural or man-made), accident or emergency.
It also assists local authorities in launching a quick rescue and response operation to save life and
property.
Q Source:
http://www.thehindu.com/news/national/andhra-pradesh/ndrf-ready-to-tackle-emergencies-in-monso
on-says-its-commandant/article18958062.ece
TEST - 1 (MOCK TEST - 1)
www.insightsias.com 19
© Insights Active Learning| All rights reserved. You may not reproduce, distribute or exploit the contents in any form without written
permission by copyright owner. Copyright infringers may face civil and criminal liability
16 With reference to Ancient India, Ashtangasamgraha written by Vagbhata deals with
A. Astrology
B. Medicine
C. Law
D. Economics
Your Answer : B
Correct Answer : B
Answer Justification :
Learning: In the field of medicine, Vagbhata lived during this period. He was the last of the great
medical trio of ancient India.
The other two scholars Charaka and Susruta lived before the Gupta age. Vagbhata was the
author Ashtangasamgraha (Summary of the eight branches of medicine).
The Sushruta and Charaka texts differ in one major aspect, with Sushruta Samhita providing
the foundation of surgery, while Charaka Samhita being primarily a foundation of medicine.
Q Source: Page 109: TN 11th Standard History Textbook
TEST - 1 (MOCK TEST - 1)
www.insightsias.com 20
© Insights Active Learning| All rights reserved. You may not reproduce, distribute or exploit the contents in any form without written
permission by copyright owner. Copyright infringers may face civil and criminal liability
17 Which of these is/are mandatory qualification(s) to contest for the office of Vice-President of India?
1. She must be a citizen of India or a person of Indian Origin (PIO) residing in India since last ten
years.
2. She must have completed 35 years of age.
3. She must not hold an office for profit.
4. She must have served as a Member of Parliament (MP) or a member of Legislative Assembly in
India.
Select the correct answer using the codes below.
A. 2 and 3 only
B. 2, 3 and 4 only
C. 1 and 4 only
D. 3 only
Your Answer : A
Correct Answer : A
Answer Justification :
Justification: As per article 66, the candidate contesting for election of Vice-President of India
should be citizen of India, completed age of 35 years and must be qualified to become a member of
Rajya Sabha. He must not hold an office for profit.
Her main function is in the form of ex-officio Chairman of Rajya Sabha. She works as acting
president when president is not available.
Learning: Like Presidential election, Vice Presidential election is also indirect and is held via
system of proportional representation by means of the single transferable vote.
The electoral college of Vice President includes elected by elected members of Lok Sabha and
Rajya Sabha.
Unlike in Presidential election, MLAs or MLCs from state assemblies have no role to play in
Vice Presidential election.
We’ll explore this better when covering Laxmikanth.
Q Source: ECI has issued a notification for the process of filing nominations for the vicepresidential
election - Incumbent Vice President Hamid Ansari’s term comes to an end on August
10, 2017.
TEST - 1 (MOCK TEST - 1)
www.insightsias.com 21
© Insights Active Learning| All rights reserved. You may not reproduce, distribute or exploit the contents in any form without written
permission by copyright owner. Copyright infringers may face civil and criminal liability
18 Under the 2007 India-Bhutan Friendship Treaty
1. Neither Government will allow the use of its territory for activities harmful to the national security
and interest of the other
2. Both governments allow free trade and commerce between the territories of Bhutan and India.
3. Bhutanese subjects residing in Indian territories shall have equal justice with Indian subjects.
4. Government of Bhutan is free to export those arms, for commercial purposes, that it received via
imports through the territory of India.
Select the correct answer using the codes below.
A. 2 and 4 only
B. 1, 2 and 3 only
C. 1 and 3 only
D. 1, 2, 3 and 4
Your Answer : C
Correct Answer : B
Answer Justification :
Background: In June 2017, Chinese Army (People’s Liberation Army) construction party had
entered the Doklam area and attempted to construct a road. Royal Bhutan Army (RBA) patrol had
attempted to dissuade them from this unilateral activity.
Since these developments, India and Bhutan have been in continuous contact, giving a view of
India’s role in Bhutan’s security and matters of mutual interest.
The present standoff between Indian and Chinese troops in Doklam (located at the tri-junction
of India, China and Bhutan) has highlighted India’s very special relationship with Bhutan,
including military responsibilities.
Justification: The 2007 treaty is meant to respect the sensitivities of Bhutan regarding its
sovereignty.
Statement 1: This is to avoid Chinese intrusion from the soil of Bhutan.
Statement 2 and 3: You can read the treaty (hardly 2 pages) here
https://mea.gov.in/Images/pdf/india-bhutan-treaty-07.pdf
Statement 4: Government of Bhutan shall be free to import, from or through India into Bhutan,
whatever arms, ammunition etc as may be required for Bhutan, and that this arrangement shall
hold good for all time as long as the Government of India is satisfied that the intentions of the
Government of Bhutan are friendly and that there is no danger to India from such importations.
However, there shall be no export of such arms, ammunition and materials outside Bhutan either by
the Government of Bhutan or by private individuals. So, 4 is wrong.
Q Source: As mentioned in the background
TEST - 1 (MOCK TEST - 1)
www.insightsias.com 22
© Insights Active Learning| All rights reserved. You may not reproduce, distribute or exploit the contents in any form without written
permission by copyright owner. Copyright infringers may face civil and criminal liability
19 Sunderbans delta is facing potential threats due to
1. Global warming induced sea level rise leading to coastal erosion
2. Freshwater flows from Brahmaputra and Meghna
Which of the above is/are correct?
A. 1 only
B. 2 only
C. Both 1 and 2
D. None
Your Answer : A
Correct Answer : A
Answer Justification :
Justification: Sundarben delta hosts a large population of mangorves (that grow in salt water) - the
largest tidal halophytic mangrove forest in the world.
Statement 1: A study highlights a time series of the erosion of many mangrove forested islands of
the Indian Sunderbans from 1986 to 2012.
These islands include Sagar, Gosaba, Dulibhasani, Dalhousie, Bhangaduni and Jambudwip.
The mean sea level rise is considered a driving factor for coastal erosion, coastal flooding,
increase in the number of tidal creeks and losing land.
Statement 2: A critical minimal inflow of freshwater is necessary for the luxuriant growth of
mangroves.
Decrease in freshwater inflow results change in mangrove succession where freshwater loving
species of mangroves are replaced by salt-water loving ones. So, 2 is wrong.
The loss land, including mangrove forest is mainly due to decrease in fresh water flow and
sediment supply in the western (Indian) part of the delta, and the rate of sea level rise is
higher than sediment supply.
However, the eastern (Bangladesh) side of the Sunderbans delta is gaining land because of
the huge amount of sediment and water flow from the Brahmaputra and Meghna rivers
(which is good for the Mangroves).
This immediate impact of salinity will also have negative impact on the fishing community, as
commercially sought fish species will be replaced by fish that does not have as much market
value.
Q Source:
TEST - 1 (MOCK TEST - 1)
www.insightsias.com 23
© Insights Active Learning| All rights reserved. You may not reproduce, distribute or exploit the contents in any form without written
permission by copyright owner. Copyright infringers may face civil and criminal liability
http://www.thehindu.com/sci-tech/energy-and-environment/climate-change-impact-sunderbans-stea
dily-losing-its-famed-mangroves/article19195229.ece
TEST - 1 (MOCK TEST - 1)
www.insightsias.com 24
© Insights Active Learning| All rights reserved. You may not reproduce, distribute or exploit the contents in any form without written
permission by copyright owner. Copyright infringers may face civil and criminal liability
20 What is the usefulness of Double Asteroid Redirection Test (DART) being developed by NASA?
1. It will help protect earth from potential cosmic body impacts in the future.
2. It will lead to the establishment of the first human space station on an asteroid.
Which of the above is/are correct?
A. 1 only
B. 2 only
C. Both 1 and 2
D. None
Your Answer : D
Correct Answer : A
Answer Justification :
Justification: This is the first-ever mission that will deflect a near-Earth asteroid.
NASA will use kinetic impactor technique to deflect them.
This technique involves sending one or more large, high-speed spacecraft into the path of an
approaching near-earth object to shift its orbit to defend against future impact.
It works by changing the speed of a threatening asteroid by a small fraction of its total
velocity.
This is done well before the predicted impact so that this small velocity adds up over time to a
big shift of the asteroid’s path away from Earth
Q Source: https://www.nasa.gov/planetarydefense/aida/
TEST - 1 (MOCK TEST - 1)
www.insightsias.com 25
© Insights Active Learning| All rights reserved. You may not reproduce, distribute or exploit the contents in any form without written
permission by copyright owner. Copyright infringers may face civil and criminal liability
21 He was a follower of Jainism in the early part of his career and converted to Saivism later. He is
hailed as ‘Vichitrachitta’ who constructed a temple for Brahma, Vishnu and Siva without the use of
bricks, timber, metal and mortar. Who is the ruler?
A. Kadambas of Banavasi
B. Adikavi Pampa
C. Rajaraja I
D. Mahendravarman I
Your Answer :
Correct Answer : D
Answer Justification :
Learning: Mahendravarman I was converted to Saivism which led him to built a Siva temple at
Tiruvadi.
He was a great builder of cave temples. The Mandagappattu inscription hails him as
Vichitrachitta who constructed a temple for Brahma, Vishnu and Siva without the use of
bricks, timber, metal and mortar.
His rock-cut temples are found in a number of places like Tiruchirappalli.
His title Chitrakarapuli reveals his talents in painting. He is also regarded as an expert in
music.
The music inscription at Kudumianmalai is ascribed to him.
Q Source: Page 124: TN 11th Standard History Textbook
TEST - 1 (MOCK TEST - 1)
www.insightsias.com 26
© Insights Active Learning| All rights reserved. You may not reproduce, distribute or exploit the contents in any form without written
permission by copyright owner. Copyright infringers may face civil and criminal liability
22 Which of these justify the robust and benevolent system of administration that Cholas had?
1. Chola kings undertook royal tours to increase the efficiency of the administration.
2. All legal professions were exempt from tax liabilities.
3. All lands were carefully surveyed and classified for assessment of revenue.
4. During the hard times, there were remission of taxes.
Select the correct answer using the codes below.
A. 1 and 4 only
B. 1, 3 and 4 only
C. 2 and 3 only
D. 2 and 4 only
Your Answer : B
Correct Answer : B
Answer Justification :
Justification: Statement 1: There were regular drills with officials.
Statement 2 and 3: There were tolls and customs on goods taken from one place to another, various
kinds of professional taxes, dues levied on ceremonial occasions like marriages and judicial fines.
Moreover, the land revenue department was well organized. It was called as
puravuvarithinaikkalam.
The residential portion of the village was called ur nattam. These and other lands such as the lands
belonging to temples were exempted from tax.
Statement 4: During the hard times, there were remission of taxes and Kulottunga I became famous
by abolishing tolls and earned the title – Sungam Tavirtta Cholan.
The main items of government expenditure were the king and his court, army and navy, roads,
irrigation tanks and canals.
Q Source: Page 146: TN 11th Standard History Textbook
TEST - 1 (MOCK TEST - 1)
www.insightsias.com 27
© Insights Active Learning| All rights reserved. You may not reproduce, distribute or exploit the contents in any form without written
permission by copyright owner. Copyright infringers may face civil and criminal liability
23 What is the significance of Sabarmati Ashram in the modern history of India?
1. It served as the last residence of Mahatma Gandhi.
2. It has been declared as a national monument by the Indian government.
3. Salt Satyagraha was started from this Ashram in 1930.
Select the correct answer using the codes below.
A. 1 only
B. 2 and 3 only
C. 3 only
D. 1 and 3 only
Your Answer : B
Correct Answer : B
Answer Justification :
Justification: The Sabarmati Ashram (also known as Harijan or Satyagraha Ashram) was home to
Mohandas Gandhi from 1917 until 1930 and served as one of the main centres of the Indian
freedom struggle.
Statement 1: This was Sevagram cottage (not Sabarmati Ashram). Gandhiji left to Delhi from
Sevargam Cottage in 1946 and then went to Noakhali. From there he did not return to Sewagram
while engaged in his task of communal unity and peace making. He was martyred in Delhi in 1948.
Statement 2 and 3: It is common knowledge.
Learning: On his return from South Africa, Gandhi’s first Ashram in India was established in the
Kochrab area of Ahmedabad in 1915. The Ashram was then shifted in 1917 to a piece of open
barren land on the banks of the river Sabarmati.
Reasons for this shift included: he wanted to do some experiments in living eg farming, animal
husbandry, cow breeding, Khadi and related constructive activities, for which he was in
search of this kind of barren land.
The Prime Minister has recently released the coin and postal stamp in the honour of Shrimad
Rajchandraji on his 150th birth anniversary at Abhay Ghat. Shrimad Rajchandraji was the
guru to Mahatma Gandhi.
Q Source: PM recently inaugurated the centenary celebrations of Sabarmati Ashram in Ahmedabad
TEST - 1 (MOCK TEST - 1)
www.insightsias.com 28
© Insights Active Learning| All rights reserved. You may not reproduce, distribute or exploit the contents in any form without written
permission by copyright owner. Copyright infringers may face civil and criminal liability
24 Which of these systems were popularly introduced by Alauddin Khalji?
1. Branding of horses
2. Maintaining a descriptive list of soldiers
3. Paying army in kind rather than cash
4. Market based pricing for all products
Select the correct answer using the codes below.
A. 1 and 2 only
B. 3 and 4 only
C. 2 and 4 only
D. 1 and 3 only
Your Answer :
Correct Answer : A
Answer Justification :
Justification: Statement 1, 2 and 3: Alauddin Khalji maintained a large permanent standing army
and paid them in cash from the royal treasury.
He introduced the system of dagh (branding of horses) and prepared huliya (descriptive list of
soldiers).
In order to ensure maximum efficiency, a strict review of army from time to time was carried
out.
Statement 4: The introduction of paying salaries in cash to the soldiers led to price regulations
popularly called as Market Reforms.
Markets were under the control of a high officer called Shahna-i- Mandi. The supply of grain was
ensured by holding stocks in government store-houses.
Regulations were issued to fix the price of all commodities.
Learning: Every merchant was registered under the Market department. There were secret agents
called munhiyans who sent reports to the Sultan regarding the functioning of these markets.
Violation of regulations was severely punished. Harsh punishment was given if any shopkeeper
charged a higher price, or tried to cheat by using false weights and measures. Even during the
famine the same price was maintained
Q Source: Page 180: TN 11th Standard History Textbook
TEST - 1 (MOCK TEST - 1)
www.insightsias.com 29
© Insights Active Learning| All rights reserved. You may not reproduce, distribute or exploit the contents in any form without written
permission by copyright owner. Copyright infringers may face civil and criminal liability
25 Consider the following statements:
1. Under the Participatory Guarantee System for India (PGS-India), FAO bureaucracy certifies organic
products after due inspection.
2. National Programme for Organic Production (NPOP) intends to facilitate certification of organic
products in conformity with the importing countries organic standards.
Which of the above is/are correct?
A. 1 only
B. 2 only
C. Both 1 and 2
D. None
Your Answer :
Correct Answer : B
Answer Justification :
Background: At present, the consumers do not have any method to check the authenticity of the
organic food products due to the absence of a regulatory framework.
FSSAI has come out with draft regulations for organic food products to ensure the
authenticity of organic foods.
FSSAI’s regulation mandates that labelling of organic foods should provide full and accurate
information regarding the organic status of the organic foods.
Organic food products must carry a certification mark or a quality assurance mark provided
by the notified certification bodies.
Justification: Statement 1: Organic foods will have to comply with the government’s provisions
under the National Programme for Organic Production (NPOP) or the Participatory Guarantee
System for India (PGS-India) run by the Agriculture Ministry or any other standards notified by
FSSAI.
The standards of Organic Crop Production have been described in NPOP document.
The production standards include crop production plan, conversion requirement, nutrient
management, disease management etc for agricultural crops.
Details will be covered in I-Learning (new initiative on website).
Statement 2: The PGS is an internationally applicable organic quality assurance system [like ISO
9000] implemented and controlled by the committed organic farmer-producers through active
participation, along with the consumers, in the process based on verifiable trust.
TEST - 1 (MOCK TEST - 1)
www.insightsias.com 30
© Insights Active Learning| All rights reserved. You may not reproduce, distribute or exploit the contents in any form without written
permission by copyright owner. Copyright infringers may face civil and criminal liability
It is not an “inspection raj” certification system but, rather, one that is based on personal integrity
and peer pressure. Integrity is honesty when no one is looking over your shoulder to see what you
are doing. So, 1 is wrong.
http://pgsindia-ncof.gov.in/pgs_india.aspx
Q Source: http://pib.nic.in/newsite/PrintRelease.aspx?relid=155000
http://www.thehindubusinessline.com/news/fssai-issues-draft-regulations-for-organic-food-products/
article9737535.ece
TEST - 1 (MOCK TEST - 1)
www.insightsias.com 31
© Insights Active Learning| All rights reserved. You may not reproduce, distribute or exploit the contents in any form without written
permission by copyright owner. Copyright infringers may face civil and criminal liability
26 Consider the following statements. Assertion (A): Kushanas banned the use of gold coins taken from
their empire on the silk route. Reason (R): Kushanas could not control the Silk route and were at a
loss from its trade.
In the context of the above, which of these is correct?
A. A is correct, and R is an appropriate explanation of A.
B. A is correct, but R is not an appropriate explanation of A.
C. A is correct, but R is incorrect.
D. Both A and R are incorrect.
Your Answer : D
Correct Answer : D
Answer Justification :
Justification: The best-known of the rulers who controlled the Silk Route were the Kushanas, who
ruled over central Asia and north-west India around 2000 years ago.
Their two major centres of power were Peshawar and Mathura.
During their rule, a branch of the Silk Route extended from Central Asia down to the seaports at the
mouth of the river Indus, from where silk was shipped westwards to the Roman Empire.
The Kushanas were amongst the earliest rulers of the subcontinent to issue gold. So, both A and R
are wrong.
Q Source: Page 102: NCERT Class VI Our Pasts - I
TEST - 1 (MOCK TEST - 1)
www.insightsias.com 32
© Insights Active Learning| All rights reserved. You may not reproduce, distribute or exploit the contents in any form without written
permission by copyright owner. Copyright infringers may face civil and criminal liability
27 Consider the following about ‘Know India Programme (KIP)’:
1. KIP is a programme run jointly by IISC, IITs and NITs to inculcate scientific temper and general
knowledge in youth.
2. KIP involves compulsory government financed field visits in public schools to places of national
importance.
Which of the above is/are correct?
A. 1 only
B. 2 only
C. Both 1 and 2
D. None
Your Answer : B
Correct Answer : D
Answer Justification :
Justification: Know India Programme is an initiative of the Ministry of External Affairs for the
Persons of Indian Origin (PIO) (including NRIs) between the age group of 18 to 30 years to provide
them an exposure to the country of their origin (India).
The KIP offers a platform for the young PIOs to visit India share their views, expectations and
experiences and forge closer bonds with the India of present times in a three-week orientation
programme.
External Affairs Minister has recently launched a portal for the Know India Programme (KIP).
Q Source: http://www.mea.gov.in/know-india-programme.htm
TEST - 1 (MOCK TEST - 1)
www.insightsias.com 33
© Insights Active Learning| All rights reserved. You may not reproduce, distribute or exploit the contents in any form without written
permission by copyright owner. Copyright infringers may face civil and criminal liability
28 Which of the two key principles/themes of democracy are inseparable or can’t exist without each
other?
A. Equity and Justice
B. Fraternity and Sovereignty
C. Accountability and transparency
D. Both (a) and (c)
Your Answer : A
Correct Answer : D
Answer Justification :
Justification: Option A: Justice implies fairness, it is a cardinal value of democracy. Equity implies
impartiality. Take Article 14 of constitution that ensures rule of law: it ensures equality between
citizens against law, that leads to justice for all.
Option B: There is no direct correlation between both.
Option C: You can take RTI as an example. If the government isn’t transparent on information, it
can’t be held accountable. On the other hand, if there is no accountability, it will be most likely due
to opacity in functioning of government. Both go hand in hand.
Q Source: Improvisation: Page 40: NCERT Class VI Social and Political Life - I
TEST - 1 (MOCK TEST - 1)
www.insightsias.com 34
© Insights Active Learning| All rights reserved. You may not reproduce, distribute or exploit the contents in any form without written
permission by copyright owner. Copyright infringers may face civil and criminal liability
29 Ramananda, Kabir and Nanak
1. Were not linked with any particular religious creed
2. Did not believe in rituals and ceremonies
3. Condemned monotheism and believed in multiple Gods
4. Denounced all forms of idolatry
Select the correct answer using the codes below.
A. 1, 2 and 3 only
B. 1, 2 and 4 only
C. 3 and 4 only
D. 1 and 3 only
Your Answer : B
Correct Answer : B
Answer Justification :
Justification: Statement 1: In the fourteenth and fifteenth centuries, Ramananda, Kabir and Nanak
remained great apostles of the Bhakti cult.
They drew inspiration from old masters but showed a new path.
They helped the common people to shed age-old superstitions and attain salvation through
Bhakti or pure devotion.
Statement 2, 3 and 4: They condemned polytheism and believed in one god. They also denounced all
forms of idolatry. They strongly believed in Bhakti as the only means of salvation. They also
emphasised the fundamental unity of all religions.
Unlike the early reformers, they were not linked with any particular religious creed and did not
believe in rituals and ceremonies.
Q Source: Page 203: TN 11th Standard History Textbook
TEST - 1 (MOCK TEST - 1)
www.insightsias.com 35
© Insights Active Learning| All rights reserved. You may not reproduce, distribute or exploit the contents in any form without written
permission by copyright owner. Copyright infringers may face civil and criminal liability
30 Over the last decade, the country that is the World’s largest receiver of remittances is
A. USA
B. India
C. Russia
D. Canada
Your Answer : B
Correct Answer : B
Answer Justification :
Learning: According to UN International Fund for Agricultural Development (IFAD), Indians
working across the world had sent home USD 62.7 billion in 2016 making India the top remittancereceiving
country surpassing China.
The study is the first ever study conducted that analysed a 10-year trend in migration and
remittance flows over the period 2007-2016.
In the period between 2007 and 2016, India has surpassed China to become the world’s
largest receiver of remittances.
Asia remains the main remittance-receiving region with 55% of the global flows.
This increase in remittances is expected to lift millions out of poverty and in attaining the
Sustainable Development Goals (SDGs).
Top Remittance Receiving Countries: India (USD 62.7 billion), China (USD 61 billion), the
Philippines (USD 30 billion) and Pakistan (USD 20 billion).
Q Source: IFAD study as mentioned above
TEST - 1 (MOCK TEST - 1)
www.insightsias.com 36
© Insights Active Learning| All rights reserved. You may not reproduce, distribute or exploit the contents in any form without written
permission by copyright owner. Copyright infringers may face civil and criminal liability
31 Bhimbetka, in present-day Madhya Pradesh, is famous for
A. Hosting prehistoric cave paintings
B. Largest Buddha Bronze statues
C. Monolithic temples
D. None of the above
Your Answer : A
Correct Answer : A
Answer Justification :
Learning: Bhimbetka rock shelters are an archaeological site of the Paleolithic, exhibiting the
earliest traces of human life on the Indian Subcontinent, and thus the beginning of the Indian Stone
Age.
Some of the Stone Age rock paintings found among the Bhimbetka rock shelters are some 30,000
years old. The caves also deliver early evidence of dance. They were declared a World Heritage Site
in 2003.
Q Source: Page 15: NCERT Class VI Our Pasts - I
TEST - 1 (MOCK TEST - 1)
www.insightsias.com 37
© Insights Active Learning| All rights reserved. You may not reproduce, distribute or exploit the contents in any form without written
permission by copyright owner. Copyright infringers may face civil and criminal liability
32 Which of these types of trading/agreements in securities markets is NOT permitted in India?
A. Futures contract
B. Options contract
C. Forward contract
D. All are permitted
Your Answer : B
Correct Answer : D
Answer Justification :
Justification: Option A: Futures Contract has been permitted long back. It is a contract between
two parties where both parties agree to purchase or sell a particular commodity or any other
financial instrument at a predetermined price at a specified time in the future.
Option B: Commodity Exchanges have been requesting SEBI for a long time to permit options
trading in commodities be allowed. It was permitted recently.
Options Contract is a derivative product that offers an investor the right to purchase without
any obligation to buy at the specified price/date.
Suppose you want your dream car which as of now costs RM 100,000 (RM is a currency unit).
You don’t have the money to buy the car now, but you will have after 3 months.
A smart move would be to buy an “option” to purchase this car in the future (after 3 months).
You pay the car owner a contract premium (say RM 1,000) to exercise this option.
Remember, you have the option to buy or not buy the car.
See the diagram below now. Suppose after 3 months, car prices increase to RM 150,000. This
is a win case for you, since the options contract was to buy it at RM 100,000. You gain.
Suppose car price reduces. Now, you won’t buy the car, because the contract was not an
obligation (only a right) to buy the car. You would now pay the options premium (RM 1000 to
the owner), and may be buy the car from the market.
TEST - 1 (MOCK TEST - 1)
www.insightsias.com 38
© Insights Active Learning| All rights reserved. You may not reproduce, distribute or exploit the contents in any form without written
permission by copyright owner. Copyright infringers may face civil and criminal liability
Option C: You would heard of the Forward Markets Commission that earlier regulated
forward markets. Now SEBI has subsumed the organization.
Q Source:
http://www.livemint.com/Money/nfnDOuk11PlqWI43mgA1BL/Sebi-permits-options-trading-incommodity-
futures.html
TEST - 1 (MOCK TEST - 1)
www.insightsias.com 39
© Insights Active Learning| All rights reserved. You may not reproduce, distribute or exploit the contents in any form without written
permission by copyright owner. Copyright infringers may face civil and criminal liability
33 Which of these earliest cities was located to the east of Indus river?
1. Dholavira
2. Rakhi Garhi
3. Kalibangan
4. Sotkakoh
Select the correct answer using the codes below.
A. 1, 2 and 3 only
B. 2 and 4 only
C. 1 and 3 only
D. 1, 2, 3 and 4
Your Answer : B
Correct Answer : A
Answer Justification :
Justification:
Learning: These cities were found in the Punjab and Sind in Pakistan, and in Gujarat, Rajasthan,
Haryana and the Punjab in India.
Archaeologists have found a set of unique objects in almost all these cities: red pottery
painted with designs in black, stone weights, seals, special beads, copper tools, and long
stone blades.
TEST - 1 (MOCK TEST - 1)
www.insightsias.com 40
© Insights Active Learning| All rights reserved. You may not reproduce, distribute or exploit the contents in any form without written
permission by copyright owner. Copyright infringers may face civil and criminal liability
Cities such as Kalibangan and Lothal had fire altars, where sacrifices may have been
performed.
In some cities, like Mohenjodaro, a very special tank, which archaeologists call the Great
Bath, was found.
Q Source: Page 33: NCERT Class VI Our Pasts - I
TEST - 1 (MOCK TEST - 1)
www.insightsias.com 41
© Insights Active Learning| All rights reserved. You may not reproduce, distribute or exploit the contents in any form without written
permission by copyright owner. Copyright infringers may face civil and criminal liability
34 The Ministry of Law and Justice has launched the ‘Tele-Law’ initiative in collaboration with the
Ministry of Electronics and Information Technology. Consider the following with reference to it.
1. It would subsume National Legal Service Authority (NALSA) and its State Level authorities to
create a Virtual Legal Service Authority (VLSA).
2. It will provide legal aid services to the marginalized communities and citizens living in rural areas.
3. It would help people to seek legal advice from lawyers with the help of video conferencing facility.
Select the correct answer using the codes below.
A. 1 and 2 only
B. 2 and 3 only
C. 3 only
D. 1, 2 and 3
Your Answer :
Correct Answer : B
Answer Justification :
Justification: Statement 2: Under the initiative, a Para Legal Volunteer (PLV), would be employed
in each Common Service Centres (CSC – which are IT service points in blocks established with
government support and regulation).
The volunteer would serve as the first point of contact for the marginalized communities in
the rural areas who will help them understand the legal issues, explain the advice given by
lawyers and assist in further action as per the advice of the lawyer.
Around 1000 women PLVs will also be trained under this initiative to provide legal aid
services through the CSCs.
Statement 3: The National Legal Service Authority (NALSA) will also provide a panel of lawyers who
will provide advice from the state capitals to the applicants in the CSCs through video conferencing.
Learning: Apart from the lawyers, law school clinics, District Legal Service Authorities, voluntary
service providers and Non-Government Organisations working on legal aid and empowerment can
also be connected through the CSCs.
In the initial phase, the initiative will be tested as a pilot project across 500 Common Service
Centres (CSC) in Uttar Pradesh and Bihar.
The government will also put in place a robust monitoring and evaluation system to assess the
quality of the legal advice delivered to the people.
Q Source:
http://www.thehindu.com/news/national/india-launches-free-legal-aid-via-video-conferencing-to-villa
gers/article18970545.ece
TEST - 1 (MOCK TEST - 1)
www.insightsias.com 42
© Insights Active Learning| All rights reserved. You may not reproduce, distribute or exploit the contents in any form without written
permission by copyright owner. Copyright infringers may face civil and criminal liability
35 The Indian constitution prohibits discrimination between individuals on the basis of
A. Gender and caste only
B. Gender, caste, religion, race and place of birth
C. Gender, education, caste and place of birth
D. Caste, education, religion and race only
Your Answer : B
Correct Answer : B
Answer Justification :
Learning: These provisions come under the Right to Equality under fundamental rights.
The State shall not discriminate against any citizen on grounds only of religion, race, caste, sex,
place of birth or any of them.
No citizen shall, on grounds only of religion, race, caste, sex, place of birth or any of them, be
subject to any disability, liability, restriction or condition with regard to
access to shops, public restaurants, hotels and places of public entertainment; or
the use of wells, tanks, bathing ghats, roads and places of public resort maintained wholly or
partly out of State funds or dedicated to the use of the general public.
Nothing in this article shall prevent the State from making any special provision for women,
children, SCs or STs.
Q Source: Improvisation: Page 20: NCERT Class VI Social and Political Life - I
TEST - 1 (MOCK TEST - 1)
www.insightsias.com 43
© Insights Active Learning| All rights reserved. You may not reproduce, distribute or exploit the contents in any form without written
permission by copyright owner. Copyright infringers may face civil and criminal liability
36 With reference to Ancient Southern India, Tolkappiyam refers to the five-fold division of lands.
Consider the following matches with their description.
1. Kurinji: Desert
2. Mullai: Pastoral
3. Neydal: Agricultural
4. Marudam: Hilly tracks
5. Palai: Coastal
Select the correct matches using the codes below.
A. 1, 3 and 4 only
B. 2 only
C. 1, 4 and 5 only
D. 1, 2, 3, 4 and 5
Your Answer :
Correct Answer : B
Answer Justification :
Justification: This was the description: Kurinji (hilly tracks), Mullai (pastoral), Marudam
(agricultural), Neydal (coastal) and Palai (desert).
Learning: The people living in these five divisions had their respective chief occupations as well as
gods for worship.
Kurinji – chief deity was Murugan – chief occupation, hunting and honey collection.
Mullai – chief deity Mayon (Vishnu) – chief occupation, cattle-rearing and dealing with dairy
products.
Marudam – chief deity Indira – chief occupation, agriculture.
Neydal – chief deity Varunan – chief occupation fishing and salt manufacturing.
Palai – chief deity Korravai – chief occupation robbery.
Q Source: Page 90: TN 11th Standard History Textbook
TEST - 1 (MOCK TEST - 1)
www.insightsias.com 44
© Insights Active Learning| All rights reserved. You may not reproduce, distribute or exploit the contents in any form without written
permission by copyright owner. Copyright infringers may face civil and criminal liability
37 “Kanishka must have conquered the greater part of the Gangetic plain”, the evidence of this
statement comes from
1. Coins of Kanishka are found in places like Mathura, Sravasti and Benares.
2. His coins exhibit only Buddhist images, and not Hindu gods, a cult that was popular then in the
Gangetic plain.
Which of the above is/are correct?
A. 1 only
B. 2 only
C. Both 1 and 2
D. None
Your Answer : A
Correct Answer : A
Answer Justification :
Justification: Statement 1: At the time of his accession his empire included Afghanistan, Gandhara,
Sind and Punjab. Subsequently he conquered Magadha and extended his power as far as Pataliputra
and Bodh Gaya. This evidence comes from his coins that are found in the gangetic plains.
Statement 2: Kanishka embraced Buddhism in the early part of his reign. However, his coins exhibit
the images of not only Buddha but also Greek and Hindu gods. So, 2 is wrong.
Q Source: Page 79: TN 11th Standard History Textbook
TEST - 1 (MOCK TEST - 1)
www.insightsias.com 45
© Insights Active Learning| All rights reserved. You may not reproduce, distribute or exploit the contents in any form without written
permission by copyright owner. Copyright infringers may face civil and criminal liability
38 The Government has issued Public Procurement (Preference to Make in India), Order 2017 recently.
Consider the following with reference to its provisions:
1. The order applies to manufacturing sector only.
2. Ordinarily, all government procurement shall be based on 100% local content sourcing.
3. Import of non-essential commodities shall be severely regulated to promote domestic
manufacturing.
Select the correct answer using the codes below.
A. 1 only
B. 2 and 3 only
C. 3 only
D. None of the above
Your Answer : C
Correct Answer : D
Answer Justification :
Justification: This policy promotes manufacturing and production of goods and services in India
with a view to enhancing income and employment. Since services are also covered, S1 is wrong.
In pursuance of this Order, purchase preference shall be given to local suppliers in all
procurements undertaken by procuring entities in the manner specified in the order.
As per the order the minimum local content shall ordinarily be 50%. The Nodal Ministry may
prescribe a higher or lower percentage in respect of any particular item and may also
prescribe the manner of calculation of local content.
Q Source: http://dipp.nic.in/sites/default/files/publicProcurement_MakeinIndia_15June2017.pdf
TEST - 1 (MOCK TEST - 1)
www.insightsias.com 46
© Insights Active Learning| All rights reserved. You may not reproduce, distribute or exploit the contents in any form without written
permission by copyright owner. Copyright infringers may face civil and criminal liability
39 An important item of trade from Ancient India, it was highly valuable in Roman Empire so much that
it was known as ‘black gold’. What does the item refer to?
A. Crude oil
B. Coal
C. Dark Garnet stones
D. Pepper
Your Answer : A
Correct Answer : D
Answer Justification :
Learning: Frequently referred to as "black gold" in ancient India, on account of its demand and
trade value mainly due to its high trade value, the Zamorin ruler of Malabar coast had a flourishing
trade centre for export of pepper and other spices in Kerala.
The main mercantile traders were Arabs and people from Middle East.
The early Roman Empire got direct access to the Malabar Coast in India and its range of
exotic spices after their conquest of Egypt in 30 BC.
The prices of pepper were extremely high in the Middle Ages and the trade was completely
dominated by the Romans.
Q Source: Page 99: NCERT Class VI Our Pasts - I
TEST - 1 (MOCK TEST - 1)
www.insightsias.com 47
© Insights Active Learning| All rights reserved. You may not reproduce, distribute or exploit the contents in any form without written
permission by copyright owner. Copyright infringers may face civil and criminal liability
40 Consider the following:
1. Temple at Deogarh near Jhansi
2. Giant copper statue of Buddha originally found at Sultanganj
3. Delhi Iron pillar
4. Bagh caves paintings
The above belong to periods of which of these dynasties of India?
A. Peshwas
B. Guptas
C. Paramara
D. Western Ganga
Your Answer :
Correct Answer : B
Answer Justification :
Learning: The Delhi Iron pillar of the Gupta period is still free from rust though completely
exposed to sun and rain for so many centuries.
The paintings of the Gupta period are seen at Bagh caves near Gwalior.
Moreover, the mural paintings of Ajantha mostly illustrate the life of the Buddha as depicted in the
Jataka stories.
There was little influence of Gandhara style on Gupta art. But the beautiful statue of standing
Buddha at Mathura reveals a little Greek style. The Buddha statue unearthed at Saranath was
unique piece of Gupta art.
Q Source: Page 106-107: TN 11th Standard History Textbook
TEST - 1 (MOCK TEST - 1)
www.insightsias.com 48
© Insights Active Learning| All rights reserved. You may not reproduce, distribute or exploit the contents in any form without written
permission by copyright owner. Copyright infringers may face civil and criminal liability
41 Under the SMARTGRAM initiative of Rashtrapati (President) Bhavan, what is considered as a ‘smart
gram (village)’?
A. A village that is on the transition to become a municipal area
B. A village where a majority of the population is digitally literate
C. A village having required physical and social infrastructure governed with a layer of smart
information and communication systems
D. A village that is close to sub-urban areas of smart cities
Your Answer : C
Correct Answer : C
Answer Justification :
Learning: Under the project the selected villages will see new ventures in agriculture, skill
development, energy, education, heath, employment generation, and entrepreneurship.
The model is based on the convergence of resources and effort by the central government, state
government, district administration, PRIs, private sector etc. to enhance the environment,
connectivity and wellbeing of each village.
Q Source: http://pib.nic.in/newsite/PrintRelease.aspx?relid=161641
TEST - 1 (MOCK TEST - 1)
www.insightsias.com 49
© Insights Active Learning| All rights reserved. You may not reproduce, distribute or exploit the contents in any form without written
permission by copyright owner. Copyright infringers may face civil and criminal liability
42 Consider the following statistics.
1. India ranks first among the world’s milk producing Nations.
2. Most of the milk produced in India comes from cooperatives.
Which of the above is/are correct?
A. 1 only
B. 2 only
C. Both 1 and 2
D. None
Your Answer : A
Correct Answer : A
Answer Justification :
Justification: Statement 1: India continued to be the largest milk producing nation (since 1998)
with milk production of nearly 155 million tonnes during 2015-16. India also has the largest bovine
population in the World.
About 54% of the milk produced in the country is surplus for marketing in the domestic market, of
which only about 20% is procured/processed by the organised sector being equally, shared by
Cooperatives and Private dairy organizations.
Statement 2: The dairy cooperatives collectively procure around 16 million tonnes of milk.
Learning: Government is mandated to double milk producers’ income at farm level by providing
rural milk producers with greater access to the organised milk processing sector.
Department of Animal Husbandry, Dairying & Fisheries has formulated a Draft National Action Plan
for Dairy Development recently.
Q Source: PIB SS (Release ID :167289)
TEST - 1 (MOCK TEST - 1)
www.insightsias.com 50
© Insights Active Learning| All rights reserved. You may not reproduce, distribute or exploit the contents in any form without written
permission by copyright owner. Copyright infringers may face civil and criminal liability
43 With reference to the Indus Valley civilization, consider the following statements:
1. Indus valley civilization was the biggest exporter of semi-precious stones and a major importer of
agricultural products.
2. Fishing was a regular occupation while hunting and bull fighting were banned.
3. Linga worship was prevalent and even natural figures like trees were worshipped.
4. Lothal was an important point of trade between the Harappan civilization and the remaining part of
India as well as Mesopotamia.
Select the correct answer using the codes below.
A. 1 and 2 only
B. 3 and 4 only
C. 1, 2 and 3 only
D. 2, 3 and 4 only
Your Answer : D
Correct Answer : B
Answer Justification :
Justification: Statement 1: Gold, copper, tin and several semi-precious stones were imported. Main
exports were several agricultural products such as wheat, barely, peas, oil seeds etc.
Statement 2: Hunting and bull fighting were common pastimes, not banned. Marbles, balls and dice
were also used for games.
Statement 3: The chief female deity was the Mother Goddess represented in terracotta figurines. In
latter times, Linga worship was prevalent. Trees and animals were also worshipped by the
Harappans. They also believed in ghosts and evil forces and used amulets as protection against
them.
Statement 4: Lothal is an important Harappan centre, you should read more about it.
Q Source: Page 21-23: TN 11th Standard History Textbook
TEST - 1 (MOCK TEST - 1)
www.insightsias.com 51
© Insights Active Learning| All rights reserved. You may not reproduce, distribute or exploit the contents in any form without written
permission by copyright owner. Copyright infringers may face civil and criminal liability
44 Many Chinese Buddhist pilgrims visited Indian subcontinent to visit places associated with the life of
the Buddha. Who among the following is NOT one of them?
A. Fa Xian
B. Xuan Zang
C. I-Qing
D. Alan Chen
Your Answer :
Correct Answer : D
Answer Justification :
Learning: Three such Chinese Buddhist pilgrims are best-known.
Fa Xian, who came to the subcontinent about 1600 years ago, Xuan Zang (who came around 1400
years ago) and I-Qing, who came about 50 years after Xuan Zang are the famous one.
They have documented previous socio-economic, political and religious details of Indian life then.
We will cover all travellers in detail in coming tests.
Q Source: Page 105: NCERT Class VI Our Pasts - I
TEST - 1 (MOCK TEST - 1)
www.insightsias.com 52
© Insights Active Learning| All rights reserved. You may not reproduce, distribute or exploit the contents in any form without written
permission by copyright owner. Copyright infringers may face civil and criminal liability
45 Consider the following statements.
1. Anyone who is a resident of the concerned village is a member of the Gram Sabha.
2. All plans for the work of the Gram Panchayat are placed before the people in Gram Sabha.
Which of the above is/are correct?
A. 1 only
B. 2 only
C. Both 1 and 2
D. None
Your Answer : B
Correct Answer : B
Answer Justification :
Justification: Statement 1: Only one who is 18 years old or more and who has the right to vote is a
member of the Gram Sabha.
The Gram Sabha is a meeting of all adults who live in the area covered by a Panchayat. This could
be only one village or a few villages.
Statement 2: The Gram Sabha prevents the Panchayat from doing wrong things like misusing
money or favouring certain people.
It plays an important role in keeping an eye on the elected representatives and in making them
responsible to the persons who elected them.
Q Source: Improvisation: Page 43-45: NCERT Class VI Social and Political Life - I
TEST - 1 (MOCK TEST - 1)
www.insightsias.com 53
© Insights Active Learning| All rights reserved. You may not reproduce, distribute or exploit the contents in any form without written
permission by copyright owner. Copyright infringers may face civil and criminal liability
46 With reference to the Vedic civilization, Rajasuya, Asvamedha and Vajpeya were
A. Rituals and sacrifices performed by the King to strengthen his position
B. Mercenary army contingents that were owned by the local landlords
C. Land tax imposed on areas inside the jurisdiction of the Empire
D. Body of officials who maintained a record of the extent of empire, its defeats and successions
Your Answer : A
Correct Answer : A
Answer Justification :
Context: Larger kingdoms were formed during the later Vedic period as many jana or tribes were
amalgamated to form janapadas or rashtras in the later Vedic period.
Hence the royal power had increased along with the increase in the size of kingdom.
Learning: So, the king performed various rituals and sacrifices to strengthen his position. They
include Rajasuya (consecration ceremony), Asvamedha (horse sacrifice) and Vajpeya (chariot race).
The kings also assumed titles like Rajavisvajanan, Ahilabhuvanapathi, (lord of all earth), Ekrat and
Samrat (sole ruler) to showcase his power.
Q Source: Page 32: TN 11th Standard History Textbook
TEST - 1 (MOCK TEST - 1)
www.insightsias.com 54
© Insights Active Learning| All rights reserved. You may not reproduce, distribute or exploit the contents in any form without written
permission by copyright owner. Copyright infringers may face civil and criminal liability
47 Jainism advocates three principles known as Triratnas (three gems) which are right faith, right
knowledge and right conduct. On the other hand, Buddhism preaches four noble truths and the
eightfold path. What is/are common between the teachings of both religions?
1. Acceptance of the theory that God exists and it has multiple aspects
2. Understanding that no object possesses a soul and is inanimate in the larger flow of life
3. To practice ahimsa and abstain from acquiring precious metals like Gold and Silver
4. Penance and extreme austerity to punish the body
Select the correct answer using the codes below.
A. 1 and 2 only
B. 3 only
C. 2 and 3 only
D. 1 and 4 only
Your Answer : B
Correct Answer : B
Answer Justification :
Justification: We will explain it in the light of teachings of Mahavira.
Justification: Statement 1 and 2: Right Knowledge (as per Mahavira) is the acceptance of the
theory that there is no God and that the world has been existing without a creator and that all
objects possess a soul.
Buddhism sheds any discussion on God and soul as futile. Buddha also taught that the soul does not
exist, so there is no meeting ground in this case. So, 1 and 2 are wrong.
Statement 3: Right conduct (Jainism) refers to the observance of the five great vows such as not to
injure life, not to lie, not to steal, not to acquire property and not to lead an immoral life.
Buddhism also forbids its monks from acquiring property (that may bring a sense of ego and pride
to its monks and nuns) and injuring life. So, 3 is a meeting ground.
Statement 4: Only Jainism advocates extreme penance, Buddhism preaches moderation. So, 4 is
wrong.
Q Source: Page 39: TN 11th Standard History Textbook
TEST - 1 (MOCK TEST - 1)
www.insightsias.com 55
© Insights Active Learning| All rights reserved. You may not reproduce, distribute or exploit the contents in any form without written
permission by copyright owner. Copyright infringers may face civil and criminal liability
48 Which of these is NOT one of the major styles or schools of ancient Indian art?
A. Amravati
B. Mathura
C. Gandhara
D. Sarnath
Your Answer : D
Correct Answer : D
Answer Justification :
Learning: The Amravati school of art had great influence on art in Sri Lanka and South-East Asia
as products from here were carried to those countries. It also had influence over South Indian
sculpture.
Mathura: During the first century AD, Gandhara and Mathura School of Art flourished mainly
during reign of Kushana emperor Kanishka.
Mathura School had developed indigenously.
Apart from Mathura, other important centres of this art were Sarnath and Kosambi.
The material used in this school was the spotted red sandstone. This art reached its peak
during the Gupta period in 6th or 7th century.
Gandhara will be covered later.
Q Source: Improvisation: Page 66: NCERT Class VI Our Pasts - I
TEST - 1 (MOCK TEST - 1)
www.insightsias.com 56
© Insights Active Learning| All rights reserved. You may not reproduce, distribute or exploit the contents in any form without written
permission by copyright owner. Copyright infringers may face civil and criminal liability
49 With reference to ANUGA 2017, consider the following:
1. It is an inter-governmental platform organized by UNSC for promoting defence technology
cooperation.
2. India will be a co-Partner Country in 2017 edition of the event.
3. Agreements signed under the event are mandated to take the place of national laws and impose
regulatory burdens on party nations.
Select the correct answer using the codes below.
A. 1 only
B. 2 and 3 only
C. 2 only
D. 3 only
Your Answer :
Correct Answer : C
Answer Justification :
Justification: Statement 1: ANUGA is the world’s biggest and most important trade fair for Food
and beverage trade held in Germany.
It takes place biennially (every 2 years). ANUGA 2017 is the 34th edition.
ANUGA offers an extensive supporting programme with Lectures, Special exhibition and
attractive industry events
More than fifty per cent of the industry visitors are foreigners.
Statement 2: India is to become a co-Partner Country in ANUGA 2017. It gives India an opportunity
to showcase its strengths and opportunities to an international audience in the food sector and
invite investments.
Statement 3: There is no such provision.
Q Source: http://pib.nic.in/newsite/PrintRelease.aspx?relid=165597
TEST - 1 (MOCK TEST - 1)
www.insightsias.com 57
© Insights Active Learning| All rights reserved. You may not reproduce, distribute or exploit the contents in any form without written
permission by copyright owner. Copyright infringers may face civil and criminal liability
50 Indian Railways Organization for Alternate Fuel (IROAF) has been recently awarded the coveted
National level “Golden Peacock Award for the Year 2017 for Eco-Innovation”. The award was given
for
1. Substitution of Diesel in passenger trains by environment friendly Compressed Natural Gas (CNG).
2. Installation of open source licensing system for electricity generation and procurement
Which of the above is/are correct?
A. 1 only
B. 2 only
C. Both 1 and 2
D. None
Your Answer :
Correct Answer : A
Answer Justification :
Justification: Indian Railways has achieved a major landmark in the field of Eco friendly fuel
technologies by this innovation.
Use of CNG in Train Sets for passenger transportation has been done for the first time in the world.
This innovation (20% substitution of diesel by CNG) will reduce emissions considerably by
bringing down NOx, CO2 and Particulate Matter besides achieving economy in fuel cost by
eight per cent.
It could save up to over thirteen hundred crores annually if implemented over entire fleet of
Diesel Locomotives of Indian Railways.
An improved technology of 40% substitution is at present being developed by IROAF. This will
enhance the potential of saving of fuel cost of IR to about Rs. thirty four hundred crore per annum.
Q Source: PIB Releases: 11th July
TEST - 1 (MOCK TEST - 1)
www.insightsias.com 58
© Insights Active Learning| All rights reserved. You may not reproduce, distribute or exploit the contents in any form without written
permission by copyright owner. Copyright infringers may face civil and criminal liability
51 The Mauryan state had a well-organized civil service, this is evident from
1. Civil servants called Amatyas who looked after the day-to-day administration were selected based
on merit.
2. Adyakshas officers helped control the retail and wholesale prices of goods and ensured their steady
supply in the state.
3. All officials were interviewed and appointed directly by the emperor.
Select the correct answer using the codes below.
A. 1 and 2 only
B. 2 and 3 only
C. 3 only
D. 2 only
Your Answer : A
Correct Answer : A
Answer Justification :
Justification: A council of ministers called Mantriparishad assisted the king in administrative
matters.
It consisted of Purohita, Mahamantri, Senapati and Yuvaraja.
Statement 1: Amatyas were similar to the IAS officers of independent India. The method of selection
of Amatyas was elaborately given by Kautilya.
Statement 2: Department of Commerce and Industry had controlled the retail and wholesale prices
of goods and tried to ensure their steady supply through its officers called Adyakshas. It also
controlled weights and measures, levied custom duties and regulated foreign trade.
Statement 3: Selection process was different for different officials.
Learning: Asoka also appointed Dhamma Mahamatras to supervise the spread of Dhamma.
Even the military service was well organized. The Mauryan army was well organized and it was
under the control of Senapati. The salaries were paid in cash. Kautilya refers to the salaries of
different ranks of military officers.
Q Source: Page 66-67: TN 11th Standard History Textbook
TEST - 1 (MOCK TEST - 1)
www.insightsias.com 59
© Insights Active Learning| All rights reserved. You may not reproduce, distribute or exploit the contents in any form without written
permission by copyright owner. Copyright infringers may face civil and criminal liability
52 Ptolemy, a Greek writer, mentions many important ports in the Deccan. Among them, the greatest
port of the Satavahanas in Western Deccan was
A. Kalyani
B. Ganjam
C. Satakarni
D. Pulamayi
Your Answer : C
Correct Answer : A
Answer Justification :
Learning: Vashishtaputra Pulamayi extended the Satavahana power up to the mouth of the Krishna
river.
He issued coins on which the image of ships was inscribed. They reveal the naval power and
maritime trade of the Satavahanas.
Their greatest port was Kalyani on the west Deccan. Gandakasela and Ganjam on the east coast
were the other important seaports.
The last great ruler of Satavahanas was Yajna Sri Satakarni.
Q Source: Page 76: TN 11th Standard History Textbook
TEST - 1 (MOCK TEST - 1)
www.insightsias.com 60
© Insights Active Learning| All rights reserved. You may not reproduce, distribute or exploit the contents in any form without written
permission by copyright owner. Copyright infringers may face civil and criminal liability
53 The National Mission on Cultural Mapping of India intends to
1. Create repository of information about cultural assets
2. Launch massive cultural awareness campaigns
3. Open a direct channel of communication of artists with the Government
4. Hold talent hunt competitions from Block level to National level
Select the correct answer using the codes below.
A. 1 and 4 only
B. 2 and 3 only
C. 1, 2 and 3 only
D. 1, 2, 3 and 4
Your Answer : C
Correct Answer : D
Answer Justification :
Learning: The mission aims at converting the vast and widespread cultural canvas of India into an
objective cultural map to preserve our cultural heritage.
The Mission also seeks to open a direct channel of communication of artists with the
Government and peer to peer communication among artists for talent honing and handholding
of each other.
With a vision that no cultural talent or heritage should remain unattended (so that it could be
nurtured), the Ministry of Culture has planned to hold talent hunt competitions with various
rounds from Block level to National level.
The Artist/Organisations also may register online at www.culturalmapping.nic.in.
The Cultural Mapping of India portal will serve as a repository/consolidated database of
information about cultural assets and resources i.e. a cultural inventory at one place for
essential planning, sustainable economic development and for preserving the scattered and
near extinct art forms.
Q Source: http://pib.nic.in/newsite/PrintRelease.aspx?relid=165655
TEST - 1 (MOCK TEST - 1)
www.insightsias.com 61
© Insights Active Learning| All rights reserved. You may not reproduce, distribute or exploit the contents in any form without written
permission by copyright owner. Copyright infringers may face civil and criminal liability
54 This council held near Srinagar prepared an authoritative commentary on the Buddhist Tripitakas
and gave the Mahayana doctrine a final shape. The council was convened by
A. Kanishka
B. Samprati
C. Bindusara Amitraghata
D. Devavarman
Your Answer :
Correct Answer : A
Answer Justification :
Learning: He convened this Fourth Buddhist Council that discussed matters relating to Buddhist
theology and doctrine.
It was held at the Kundalavana monastery near Srinagar in Kashmir under the presidentship of
Vasumitra. About 500 monks attended the Council.
Learning: Asvagosha was a great philosopher, poet and dramatist and was particularly close to
Kanishka. He became his religious advisor in his later years and composed Buddhacharita.
Nagarjuna from south India also adorned the court of Kanishka. The famous physician of ancient
India Charaka was also patronized by him.
Q Source: Page: TN 11th Standard History Textbook
TEST - 1 (MOCK TEST - 1)
www.insightsias.com 62
© Insights Active Learning| All rights reserved. You may not reproduce, distribute or exploit the contents in any form without written
permission by copyright owner. Copyright infringers may face civil and criminal liability
55 Consider the following about The Energy Resources Institute (TERI).
1. It is a non-profit policy research organisation.
2. It is an attached agency of the International Centre for Climate Governance (ICCG).
Which of the above is/are correct?
A. 1 only
B. 2 only
C. Both 1 and 2
D. None
Your Answer : C
Correct Answer : A
Answer Justification :
Justification: Statement 1: TERI conducts research work in the fields of energy, environment and
sustainable development.
Statement 2: The Energy Resources Institute (TERI) was ranked second among world’s best climate
think tank by the International Centre for Climate Governance (ICCG).
ICCG is an internationally renowned centre founded in 2009.
The main focus of ICCG activities is the design of climate policy and related governance issues.
Q Source: www.insightsonindia.com/2017/07/06/insights-daily-current-affairs-06-july-2017/
TEST - 1 (MOCK TEST - 1)
www.insightsias.com 63
© Insights Active Learning| All rights reserved. You may not reproduce, distribute or exploit the contents in any form without written
permission by copyright owner. Copyright infringers may face civil and criminal liability
56 Which of these ancient scripts generally writes in left to right direction?
A. Brahmi
B. Kharosthi
C. Harappan
D. All of the above
Your Answer : B
Correct Answer : A
Answer Justification :
Learning: Brahmi: is one of the oldest writing systems used in South and Central Asia from the 1st
millennium BCE.
Most modern Indian scripts have developed from the Brahmi script over hundreds of years.
Harappan: Iravatham Mahadevan established that the Harappan script is from right to left.
Kharosthi: is an ancient script used in ancient Gandhara (primarily modern-day Afghanistan and
Pakistan) to write the Gandhari Prakrit and Sanskrit.
Q Source: Improvisation: Page 81: NCERT Class VI Our Pasts - I
TEST - 1 (MOCK TEST - 1)
www.insightsias.com 64
© Insights Active Learning| All rights reserved. You may not reproduce, distribute or exploit the contents in any form without written
permission by copyright owner. Copyright infringers may face civil and criminal liability
57 Consider the following statements.
1. Innovate in India (i3) program offers young entrepreneurs an avenue to engage with the best
practices in the domestic financial and banking industry.
2. National Biopharma Mission will concentrate, inter alia, on development of medical devices and
biotherapeutics to upgrade India’s product development capabilities to globally competitive levels.
Which of the above is/are correct?
A. 1 only
B. 2 only
C. Both 1 and 2
D. None
Your Answer : B
Correct Answer : B
Answer Justification :
Justification: Statement 1: i3 is a flagship program of the Government of India in collaboration
with World Bank.
It is committed to make India a hub for design and development of novel, affordable and
effective biopharmaceutical products and solutions by engaging with entrepreneurs.
It will strive to increase India’s global biopharmaceutical market share from current 2.8% to
5% and generate additional business opportunity of $16 billion.
Statement 2 (S2): It will bring together expertise from national and international corridors to
provide strategic direction and guidance to move promising solutions through the product
development value chain.
The mission will be implemented by Biotechnology Industry Research Assistance Council (BIRAC), a
PSU of Department of Biotechnology.
Besides, it will also work on establishment of shared infrastructure and facilities and at the same
time creating and enhancing technology transfer capabilities in public and private sector.
Q Source: http://pib.nic.in/newsite/PrintRelease.aspx?relid=166951
TEST - 1 (MOCK TEST - 1)
www.insightsias.com 65
© Insights Active Learning| All rights reserved. You may not reproduce, distribute or exploit the contents in any form without written
permission by copyright owner. Copyright infringers may face civil and criminal liability
58 In a democratic state, government derives its ultimate sovereignty from
A. Citizens of the state
B. Territory of the State
C. Laws governing the state
D. Parliament
Your Answer : D
Correct Answer : A
Answer Justification :
Justification: The chain is straightforward.
People elect representatives, representatives form Parliament, Parliament chooses the government,
government makes laws, and those laws regulate the territory of the state.
So, the root of sovereignty can be easily traced to people.
Q Source: Improvisation: Page 31: NCERT Class VI Social and Political Life - I
TEST - 1 (MOCK TEST - 1)
www.insightsias.com 66
© Insights Active Learning| All rights reserved. You may not reproduce, distribute or exploit the contents in any form without written
permission by copyright owner. Copyright infringers may face civil and criminal liability
59 Chandragupta II, a ruler of the Gupta Empire, assumed the famous title of ‘Sakari’. What did the title
imply?
A. Destroyer of Sakas
B. Lord of the world
C. Well Wisher of masses
D. Devotee of Shankara
Your Answer :
Correct Answer : A
Answer Justification :
Learning: The greatest of the military achievements of Chandragupta II was his war against the
Saka satraps of western India.
Rudrasimha III, the last ruler of the Saka satrap was defeated, dethroned and killed.
His territories in western Malwa and the Kathiawar Peninsula were annexed into the Gupta
Empire.
After this victory he performed the horse sacrifice and assumed the title Sakari, meaning,
‘destroyer of Sakas’. He also called himself Vikramaditya.
Q Source: Page 101: TN 11th Standard History Textbook
TEST - 1 (MOCK TEST - 1)
www.insightsias.com 67
© Insights Active Learning| All rights reserved. You may not reproduce, distribute or exploit the contents in any form without written
permission by copyright owner. Copyright infringers may face civil and criminal liability
60 Consider the following statements about the recently established Global Foreign Exchange
Committee (GFXC).
1. It was established as a research arm of International Monetary Fund (IMF).
2. GFXC will enforce a FX Global Code that will legally regulate the manipulation of floating foreign
exchange by Central Banks.
3. It will promote collaboration and communication among the local foreign exchange committees
(FXCs) of member nations.
4. It will replace the institution of Bank of International Settlements (BIS).
5. Indian government has boycotted GFXC due to its non-democratic governance structure.
Select the correct answer using the codes below.
A. 2, 3 and 4 only
B. 1, 2 and 5 only
C. 3 only
D. 1, 2, 3 and 4 only
Your Answer : C
Correct Answer : C
Answer Justification :
Justification: This is a very important development, hence the long explanation.
Statement 1: GFXC was established in 2017 as a forum bringing together central banks and private
sector participants with the aim to promote a robust, liquid, open, and appropriately transparent FX
market.
Statement 2: FX Global Code is a set of global principles of good practice in the foreign exchange
market, developed to provide a common set of guidelines to promote the integrity and effective
functioning of the wholesale foreign exchange market.
The Global Code does not impose legal or regulatory obligations on Market Participants, nor
does it substitute for regulation.
It is rather intended to serve as a supplement to any and all local laws, rules and regulations
by identifying global good practices and processes.
Statement 3: The objectives of the GFXC are:
to promote collaboration and communication among the local foreign exchange committees
(FXCs) and non-GFXC jurisdictions with significant FX markets;
to exchange views on trends and developments in global FX markets, including on the
structure and functioning of those markets, drawing on information gathered at the various
FXCs; and
TEST - 1 (MOCK TEST - 1)
www.insightsias.com 68
© Insights Active Learning| All rights reserved. You may not reproduce, distribute or exploit the contents in any form without written
permission by copyright owner. Copyright infringers may face civil and criminal liability
to promote, maintain and update on a regular basis the FX Global Code (the Code) and to
consider good practices regarding effective mechanisms to support adherence.
Statement 4: The GFXC committee has been set up under guidance of the Bank for International
Settlements (BIS), an international financial organisation.
Statement 5: India will soon join Global Foreign Exchange Committee (GFXC).
Q Source:
http://www.thehindubusinessline.com/markets/forex/india-to-join-new-global-foreign-exchange-com
mittee/article9756342.ece
TEST - 1 (MOCK TEST - 1)
www.insightsias.com 69
© Insights Active Learning| All rights reserved. You may not reproduce, distribute or exploit the contents in any form without written
permission by copyright owner. Copyright infringers may face civil and criminal liability
61 Why the sale of Oxytocin, despite being a natural hormone, is heavily regulated?
1. It was used as a depressant by a large segment of population.
2. Its indiscriminate use by dairy farmers was causing irreversible hormone damage in milch animals.
3. Consumption of food laced with Oxytocin can cause cardiac problems in humans.
4. It is abused to speed up child birth in women.
Select the correct answer using the codes below.
A. 1 and 3 only
B. 2, 3 and 4 only
C. 2 and 4 only
D. 1, 2, 3 and 4
Your Answer : B
Correct Answer : B
Answer Justification :
Justification: Statement 1: Oxytocin has also been dubbed the love or the bliss hormone due to its
effects on behaviour. It isn’t a depressant. So, 1 is wrong.
Statement 2: The drug Oxytocin is banned in India under Prevention of Cruelty to Animals Act and
Food and Drug Adulteration Prevention Act, 1960.
In mammals, Oxytocin stimulates ejection of milk from the mammary glands.
But, its use can cause reproductive damage in animals and even their death.
Statement 3: Overdose of the drug in humans can lead to haemorrhage, palpitation, low blood
pressure and in some cases even death.
Statement 4: The release of oxytocin by the pituitary gland acts to regulate two female reproductive
functions: Childbirth and Breast-feeding. So, it is used to speed up child birth which is wrong and
should be shunned.
So, after being banned in retail markets, the Indian Drug Controller will soon restrict
manufacturing of controversial hormone drug Oxytocin to public sector undertakings (PSUs).
Q Source:
http://www.thehindu.com/news/national/only-psus-may-be-let-to-make-oxytocin/article18868286.ece
TEST - 1 (MOCK TEST - 1)
www.insightsias.com 70
© Insights Active Learning| All rights reserved. You may not reproduce, distribute or exploit the contents in any form without written
permission by copyright owner. Copyright infringers may face civil and criminal liability
62 Consider the following statements.
1. Manimegalai is a Tamil epic that criticizes Buddhism while investigating the strengths of
contemporary Hindu traditions.
2. Silappatikaram, produced at Sangam, is a poetic rendition with details of Tamil culture.
Which of the above is/are correct?
A. 1 only
B. 2 only
C. Both 1 and 2
D. None
Your Answer :
Correct Answer : B
Answer Justification :
Justification: The corpus of Sangam literature includes Tolkappiyam, Ettutogai, Pattuppattu,
Pathinenkilkanakku, and the two epic – Silappathigaram and Manimegalai.
Statement 1: The aim of the author of Manimegalai, Seethalai Saathanar, was to compare Buddhism
favourably with the other prevailing religions in South India in order to propagate Buddhism.
He criticizes Hinduism, and exposes the weaknesses of the other contemporary Indian
religions, while praising the Buddha's Teaching, the Dharma, as the most perfect religion. So,
1 is wrong.
Statement 2: Regarded as one of the great works of Tamil literature, the Silappatikaram is a poetic
rendition with details of Tamil culture; its varied religions; its town plans and city types; the
mingling of different people; and the arts of dance and music.
Q Source: Improvisation: Page 85-86: TN 11th Standard History Textbook
TEST - 1 (MOCK TEST - 1)
www.insightsias.com 71
© Insights Active Learning| All rights reserved. You may not reproduce, distribute or exploit the contents in any form without written
permission by copyright owner. Copyright infringers may face civil and criminal liability
63 Consider the following statements.
1. Harsha prohibited the use of animal food in his kingdom and punished those who killed any living
being.
2. Harsha gave away his enormous wealth as gifts to the members of all religious sects during the
Allahabad Conference.
Which of the above is/are correct?
A. 1 only
B. 2 only
C. Both 1 and 2
D. None
Your Answer : C
Correct Answer : C
Answer Justification :
Justification: Statement 1: In his early life, Harsha was a devout Saiva but later he became an
ardent Hinayana Buddhist. Hiuen Tsang converted him to Mahayana Buddhism. This led him to
shed eating meat and enforce the same as well.
Statement 2: Hiuen Tsang mentions in his account about the conference held at Allahabad, known
as Prayag.
It was the one among the conferences routinely convened by Harsha once in five years. Harsha gave
away his enormous wealth as gifts to the members of all religious sects.
According to Hiuen Tsang, Harsha was so lavish that he emptied the treasury and even gave away
the clothes and jewels he was wearing. His statement might be one of admiring exaggeration.
Q Source: Page 116: TN 11th Standard History Textbook
TEST - 1 (MOCK TEST - 1)
www.insightsias.com 72
© Insights Active Learning| All rights reserved. You may not reproduce, distribute or exploit the contents in any form without written
permission by copyright owner. Copyright infringers may face civil and criminal liability
64 With reference to the rights and responsibilities of the Attorney General of India, consider the
following:
1. He has right of audience in all courts within the territory of India.
2. He has the right to speak and take part in the proceeding of a joint sitting of the Parliament.
3. He is entitled to vote on crucial national resolutions and motions in Rajya Sabha, excluding
legislations.
4. He can be made a member of any parliamentary committee with a right to discuss and vote.
5. He is not allowed to take up private practice as long as he is an incumbent.
Select the correct answer using the codes below.
A. 1 and 2 only
B. 1, 3 and 4 only
C. 1, 4 and 5 only
D. 2, 3 and 5 only
Your Answer : C
Correct Answer : A
Answer Justification :
Justification: He is the first law officer of the government of India and acts as top advocate for
Union Government. He is responsible for giving advice to President/ Union Government upon legal
matters.
Statement 2 and 3: He has the right to speak and take part in proceedings of both the houses of
parliament including joint sittings. But cannot vote in parliament.
Statement 4: He can also be made a member of any parliamentary committee but in the committee
also, he has no power to vote. He has all the powers and privileges that of a member of parliament.
Statement 5: Attorney General of India is not a full time Government servant. He is an advocate of
the government and is allowed to take up private practice, provided the other party is not the state.
Further, without permission of the government he cannot defend the accused persons in criminal
matters.
Q Source:
http://www.thehindu.com/news/national/attorney-general-rohatgi-will-defend-indias-record-on-rights
-abuses-at-unhrcs-review/article18312468.ece
TEST - 1 (MOCK TEST - 1)
www.insightsias.com 73
© Insights Active Learning| All rights reserved. You may not reproduce, distribute or exploit the contents in any form without written
permission by copyright owner. Copyright infringers may face civil and criminal liability
65 In the context of the rule of South Indian dynasties, Devadhana and Brahmadeya referred to?
A. Pristine land
B. Divine king rule
C. Treasure appropriated to the Gods
D. Land grants to the temples
Your Answer : C
Correct Answer : D
Answer Justification :
Justification: Land tax was the primary source of the government revenue. The Brahmadeya and
Devadhana lands were exempted from tax.
Learning: This is a lateral explanation and deals with aspects other than those mentioned in the
question, because detailing the topic at hand is not of much use.
The Pallavas had a well organized administrative system. The Pallava state was divided into
Kottams. The Kottam was administered by officers appointed by the king.
It was also the responsibility of the central government to provide irrigation facilities to the
lands.
Traders and artisans such as carpenters, goldsmiths, washer-men, oil-pressers and weavers
paid taxes to the government.
The Pallava inscriptions throw much light on the village assemblies called sabhas and their
committees. They maintained records of all village lands, looked after local affairs and
managed temples.
Q Source: Page 126: TN 11th Standard History Textbook
TEST - 1 (MOCK TEST - 1)
www.insightsias.com 74
© Insights Active Learning| All rights reserved. You may not reproduce, distribute or exploit the contents in any form without written
permission by copyright owner. Copyright infringers may face civil and criminal liability
66 They introduced the art of excavating temples from the rock, and the Dravidian style of temple
architecture began with their rule. They were?
A. Cheras
B. Pallavas
C. Eastern Chalukyas
D. Jaffna
Your Answer : B
Correct Answer : B
Answer Justification :
Learning: Pallava age was a great age of temple building.
It was a gradual evolution starting from the cave temples to monolithic rathas and culminated in
structural temples.
The development of temple architecture under the Pallavas can be seen in several stages, which
started with Mahendravarman I introducing rock-cut temples.
This style of Pallava temples are seen at places like Mandagappattu, Mahendravadi, Mamandur,
Dalavanur, Tiruchirappalli, Vallam, Siyamangalam and Tirukalukkunram.
Q Source: Page 127: TN 11th Standard History Textbook
TEST - 1 (MOCK TEST - 1)
www.insightsias.com 75
© Insights Active Learning| All rights reserved. You may not reproduce, distribute or exploit the contents in any form without written
permission by copyright owner. Copyright infringers may face civil and criminal liability
67 In Delhi Sultanate, the Sultan was assisted by a number of departments and officials in his
administration. Consider the following with reference to these departments:
1. Diwani Insha dealt with the correspondence between the ruler and the officials.
2. Diwani Ariz gave grants for the construction and maintenance of mosques, tombs and madrasas.
Which of the above is/are correct?
A. 1 only
B. 2 only
C. Both 1 and 2
D. None
Your Answer :
Correct Answer : A
Answer Justification :
Justification: Statement 1: It is correct.
Statement 2: The military department was called Diwani Ariz. It was headed by Ariz-i-mumalik. He
was responsible for recruiting the soldiers and administering the military department.
Learning: Under the Delhi sultanate, the post of Naib was the most powerful one. The Naib
practically enjoyed all the powers of the Sultan and exercised general control over all the
departments.
Next to him was the Wazir who was heading the finance department called Diwani Wizarat.
Diwani Rasalat was the department of religious affairs. It was headed by chief Sadr. Grants
were made by this department for religious construction.
Q Source: Page 192: TN 11th Standard History Textbook
TEST - 1 (MOCK TEST - 1)
www.insightsias.com 76
© Insights Active Learning| All rights reserved. You may not reproduce, distribute or exploit the contents in any form without written
permission by copyright owner. Copyright infringers may face civil and criminal liability
68 In the context of Ancient India, Amogavarsha’s work Kavirajamarga is significant because?
A. It was the first poetic work in Kannada language.
B. The largest repository of royal assets was commissioned in this work.
C. All artists of the empire contributed to this work.
D. It was a prose written in the middle of a battle ground.
Your Answer :
Correct Answer : A
Answer Justification :
Learning: The Kannada literature saw its beginning during the period of the Rashtrakutas.
Amogavarsha’s Kavirajamarga was the first poetic work in Kannada language.
The name literally means "Royal Path for Poets" and was written as a guide book for poets and
scholars (Kavishiksha).
Q Source: Page 137: TN 11th Standard History Textbook
TEST - 1 (MOCK TEST - 1)
www.insightsias.com 77
© Insights Active Learning| All rights reserved. You may not reproduce, distribute or exploit the contents in any form without written
permission by copyright owner. Copyright infringers may face civil and criminal liability
69 Several accords have been signed between the Government of India (GoI) and different groups to
bring peace and prosperity to the North-eastern states. Consider the following with reference to
them.
1. Mizo accord promised statehood to Mizoram.
2. Assam Accord was reached between GoI and ULFA calming the Assam agitation.
3. Nagaland peace accord between GoI and the National Socialist Council of Nagaland (NSCN) was
signed to end the Naga insurgency.
Select the correct answer using the codes below.
A. 1 and 3 only
B. 3 only
C. 2 and 3 only
D. 1 and 2 only
Your Answer : A
Correct Answer : A
Answer Justification :
Justification: Statement 1: Under the Mizo accord, the Mizo National Front (MNF) fighters
surrendered en masse and government fulfilled its promise by giving full-fledged state status to
Mizoram, along with its own High Court.
Mizo was notified as an official Indian language. Further, Mizo religious and social customs
were promised Constitutional protection.
As per the accord, the MNF agreed to forgo all contacts with other insurgent groups in the
Northeast. Mizo Accord is considered to be the most successful agreement with insurgents.
Statement 2: ULFA is a banned organization. So, 2 is wrong.
The accord brought an end to the Assam Agitation and paved the way for the leaders of the
agitation to form a political party and form a government in the state of Assam soon after. However,
not all promises have been fulfilled that remains a bone of contention.
Statement 3: This article explains the history and issues with Naga insurgency very clearly and
chronologically: A must read – This is an important issue.
http://indianexpress.com/article/explained/simply-put-towards-accord-step-by-step/
Q Source:
http://www.thehindu.com/news/national/mizoram-celebrates-31st-anniversary-of-mizo-peace-accord/
article19187194.ece
TEST - 1 (MOCK TEST - 1)
www.insightsias.com 78
© Insights Active Learning| All rights reserved. You may not reproduce, distribute or exploit the contents in any form without written
permission by copyright owner. Copyright infringers may face civil and criminal liability
70 The system of village autonomy with sabhas and their committees developed through the ages and
reached its culmination during the Chola rule. A village was divided in several wards, what was/were
the qualifications to become a ward member?
1. A woman could not become a ward member.
2. Only those who have completed seventy years of age
3. Have knowledge of Vedas
4. Owns a residence
Select the correct answer using the codes below.
A. 3 and 4 only
B. 1, 2 and 3 only
C. 1 and 4 only
D. 1, 2, 3 and 4
Your Answer : C
Correct Answer : A
Answer Justification :
Learning: That village was divided into thirty wards and each was to nominate its members to the
village council.
The qualifications to become a ward member were:
Ownership of at least one fourth veli of land.
Own residence.
Above thirty years and below seventy years of age.
Knowledge of Vedas.
However, certain norms of disqualification were also mentioned in the inscriptions. They were:
Those who had been members of the committees for the past three years.
Those who had failed to submit accounts as committee members.
Those who had committed sins.
Those who had stolen the property of others
Two inscriptions belonging to the period of Parantaka I found at Uttiramerur provide details of the
TEST - 1 (MOCK TEST - 1)
www.insightsias.com 79
© Insights Active Learning| All rights reserved. You may not reproduce, distribute or exploit the contents in any form without written
permission by copyright owner. Copyright infringers may face civil and criminal liability
formation and functions of village councils.
Q Source: Page 148: TN 11th Standard History Textbook
TEST - 1 (MOCK TEST - 1)
www.insightsias.com 80
© Insights Active Learning| All rights reserved. You may not reproduce, distribute or exploit the contents in any form without written
permission by copyright owner. Copyright infringers may face civil and criminal liability
71 In the ancient traditions, traders’ voyages often refer which of these lands to Suvarnabhumi (the land
of gold)?
A. Countries of the East Asia
B. African nations
C. Sri Lankan islands
D. European countries
Your Answer :
Correct Answer : A
Answer Justification :
Learning: The east coast of India is studded with numerous ports and Indians undertook frequent
voyages to these lands.
Also, Indians began to colonize the East Asia in the Gupta period. It was further encouraged
by the Pallavas.
The Indian colonists established great kingdoms (with Indian names) and some of them lasted
for more than a thousand years.
Till the arrival of Islam in the fifteenth century, Indian culture dominated this region.
Q Source: Page 156: TN 11th Standard History Textbook
TEST - 1 (MOCK TEST - 1)
www.insightsias.com 81
© Insights Active Learning| All rights reserved. You may not reproduce, distribute or exploit the contents in any form without written
permission by copyright owner. Copyright infringers may face civil and criminal liability
72 The system of four ashrams is advocated in
A. Buddhism
B. Jainism
C. Lokayatas
D. Brahmanism
Your Answer : C
Correct Answer : D
Answer Justification :
Learning: Around the time when Jainism and Buddhism were becoming popular, brahmins
developed the system of ashramas.
Ashramas refer to stage of life: brahmacharya, grihastha, vanaprastha and samnyasa.
Brahmin, kshatriya and vaishya men were expected to lead simple lives and study the Vedas
during the early years of their life (brahmacharya).
Then they had to marry and live as householders (grihastha).
Then they had to live in the forest and meditate (vanaprastha). Finally, they had to give up
everything and become samnyasins.
The system of ashramas allowed men to spend some part of their lives in meditation.
Generally, women were not allowed to study the Vedas, and they had to follow the ashramas
chosen by their husbands.
Q Source: Page 173: TN 11th Standard History Textbook
TEST - 1 (MOCK TEST - 1)
www.insightsias.com 82
© Insights Active Learning| All rights reserved. You may not reproduce, distribute or exploit the contents in any form without written
permission by copyright owner. Copyright infringers may face civil and criminal liability
73 ‘Petya’ recently seen in news was a/an
A. Laser beaming innovation
B. Form of cyber-attack
C. Alternate to fossil fuels
D. Emergency helpline launched by PETA
Your Answer : B
Correct Answer : B
Answer Justification :
Learning: It is a ransomware, similar to the Wannacry ransomware.
In India, this ransomware crippled the operations at one of the terminals of the Jawaharlal Nehru
Port Trust and impacted the external trade by affecting the systems dealing with the cargo and
ships.
Ransomware is a type of malicious software that threatens to publish the victim's data or
perpetually block access to it unless a ransom is paid.
Q Source:
https://www.theguardian.com/technology/2017/jun/27/petya-ransomware-cyber-attack-who-what-wh
y-how
TEST - 1 (MOCK TEST - 1)
www.insightsias.com 83
© Insights Active Learning| All rights reserved. You may not reproduce, distribute or exploit the contents in any form without written
permission by copyright owner. Copyright infringers may face civil and criminal liability
74 In 1329-30 Muhammad bin Tughlaq introduced a token copper currency in place of silver, but banned
it later and promised to exchange silver coins for these copper coins. This was done because
A. Copper was scarce in the empire, but had huge demand in the world.
B. Token currency was trampled and disrespected by people.
C. Goldsmiths began to forge the token coins on a large scale
D. Treasury became empty with issue of token coins.
Your Answer : A
Correct Answer : C
Answer Justification :
Learning: There was a shortage of silver through out the world in the fourteenth century.
Kublai Khan issued paper money in China. In the same manner, Muhammad bin Tughlaq
issued copper coins at par with the value of the silver tanka coins.
But he was not able to prevent forging the new coins. The goldsmiths began to forge the
token coins on a large scale. Soon the new coins were not accepted in the markets due to its
widescale duplication and thus credibility.
Finally, Muhammad bin Tughlaq stopped the circulation of token currency and promised to
exchange silver coins for the copper coins.
Many people exchanged the new coins but the treasury became empty.
According the Barani, the heap of copper coins remained lying on roadside in Tughlaqabad.
Q Source: Page 184: TN 11th Standard History Textbook
TEST - 1 (MOCK TEST - 1)
www.insightsias.com 84
© Insights Active Learning| All rights reserved. You may not reproduce, distribute or exploit the contents in any form without written
permission by copyright owner. Copyright infringers may face civil and criminal liability
75 Writing in the journal Young India in 1931, who said, "I cannot possibly bear the idea that a man who
has got wealth should get the vote but a man who has got character but no wealth or literacy should
have no vote…. for the crime of being a poor man…"?
A. Jawahar Lal Nehru
B. M.K. Gandhi
C. Rajendra Prasad
D. Maulana Abul Kalam Azad
Your Answer :
Correct Answer : B
Answer Justification :
Learning: Young India, Harijan and Hind Swaraj were some of the most famous works of Gandhi.
Young India was a weekshed - a weekly paper or journal.
He used Young India to spread his unique ideology and thoughts regarding the use of nonviolence
in organising movements and to urge readers to consider, organise, and plan for India's eventual
independence from Britain.
The journal was reprinted in USA by the India Home Rule League of America.
Q Source: Improvisation: Page 33: NCERT Class VI Social and Political Life - I
TEST - 1 (MOCK TEST - 1)
www.insightsias.com 85
© Insights Active Learning| All rights reserved. You may not reproduce, distribute or exploit the contents in any form without written
permission by copyright owner. Copyright infringers may face civil and criminal liability
76 In the context of Pallava’s history, what was Dakshinachitra?
A. A compendium of temples
B. A spiritual treatise on Pallava practices
C. An autobiography of a Pallava ruler
D. An instruction manual for painters
Your Answer :
Correct Answer : D
Answer Justification :
Learning: Mahendravarman, one of the greatest of the rulers of the Pallava dynasty, was found of
painting and is said to have commissioned a scholar at his court to pen down an instruction manual
for painters, which is a treatise on south Indian paintings called Dakshinachitra.
Unfortunately, it has been lost in the course of time.
Mahendravarman I gave himself the title of “Vchitrachitta” (the man with new fangled ideas).
Q Source: Page 129: TN 11th Standard History Textbook
TEST - 1 (MOCK TEST - 1)
www.insightsias.com 86
© Insights Active Learning| All rights reserved. You may not reproduce, distribute or exploit the contents in any form without written
permission by copyright owner. Copyright infringers may face civil and criminal liability
77 In medieval India, takkavi loans were given in order to
A. Improve agriculture
B. Build religious shrines
C. Construct flood saving embankments in cities
D. Serve sovereign debt
Your Answer :
Correct Answer : A
Answer Justification :
Learning: Takkavi loans were a measure of reprieve taken after few suppressing moves of Tughlaq.
In order to overcome financial difficulties in his empire, Muhammad bin Tughlaq increased
the land revenue on the farmers of Doab (land between Ganges and Yamuna rivers).
It was an excessive and arbitrary step on the farmers. A severe famine was also ravaging that
region at that time. It had resulted in a serious peasant revolts. The revolts were crushed.
However, the Sultan realized later that adequate relief measures and the promotion of
agriculture were the real solution to the problem.
He launched a scheme by which takkavi loans (loans for cultivation) were given to the farmers
to buy seed and to extend agriculture.
Q Source: Page 193: TN 11th Standard History Textbook
TEST - 1 (MOCK TEST - 1)
www.insightsias.com 87
© Insights Active Learning| All rights reserved. You may not reproduce, distribute or exploit the contents in any form without written
permission by copyright owner. Copyright infringers may face civil and criminal liability
78 Consider the following statements.
1. Sankara denounced Hindu revivalist movement citing them as going against rationality and
modernity.
2. Ramanuja advocated prabattimarga or the path of self-surrender to God for salvation.
Which of the above is/are correct?
A. 1 only
B. 2 only
C. Both 1 and 2
D. None
Your Answer :
Correct Answer : B
Answer Justification :
Learning: Statement 1: In the ninth century Sankara started a Hindu revivalist movement giving a
new orientation to Hinduism. So, 1 is wrong.
However, his doctrine of Advaita or Monism was too abstract to appeal to the common man.
Moreover, there was a reaction against the Advaita concept of Nirgunabrahman (God without
attributes) with the emergence of the idea of Sagunabrahman (God with attributes).
Statement 2: In the twelfth century, Ramanuja, preached Visishtadvaita.
According to him God is Sagunabrahman. The creative process and all the objects in creation
are real but not illusory as was held by Sankaracharya.
Therefore, God, soul, matter are real. But God is inner substance and the rest are his
attributes.
He invited the downtrodden to Vaishnavism
Q Source: Page 202: TN 11th Standard History Textbook
TEST - 1 (MOCK TEST - 1)
www.insightsias.com 88
© Insights Active Learning| All rights reserved. You may not reproduce, distribute or exploit the contents in any form without written
permission by copyright owner. Copyright infringers may face civil and criminal liability
79 Consider the following statements.
1. A Supernova helps us know about the Universe since they are the longest existing phenomenon in
space.
2. Pulsars are de-magnetized neutron stars that neither rotate nor emit electromagnetic radiation.
3. Black hole refers to the remains of a supergiant star that has collapsed into itself.
4. The effects of microgravity can be seen when astronauts and objects float in space.
Select the correct answer using the codes below.
A. 1 and 3 only
B. 3 and 4 only
C. 2, 3 and 4 only
D. 1, 2 and 4 only
Your Answer : D
Correct Answer : B
Answer Justification :
Background: China has launched its first X-ray space telescope to study black holes, pulsars and
gamma-ray bursts.
Justification: Statement 1: A supernova happens where there is a change in the core, or centre, of
a star.
A supernova burns for only a short period of time, but it can tell scientists a lot about the universe.
One kind of supernova has shown scientists that we live in an expanding universe, one that is
growing at an ever increasing rate.
Statement 2: Pulsars are highly magnetized, rotating neutron stars that emit a beam of
electromagnetic radiation.
They are spherical, compact objects that are about the size of a large city but contain more
mass than that of the sun.
Scientists are using pulsars to study extreme states of matter, search for planets beyond
Earth’s solar system and measure cosmic distance.
Statement 3: A black hole cannot be seen because of the strong gravity that is pulling all of the light
into the black hole's centre.
However, scientists can see the effects of its strong gravity on the stars and gases around it.
If a star is orbiting a certain point in space, scientists can study the star's motion to find out if
it is orbiting a black hole.
TEST - 1 (MOCK TEST - 1)
www.insightsias.com 89
© Insights Active Learning| All rights reserved. You may not reproduce, distribute or exploit the contents in any form without written
permission by copyright owner. Copyright infringers may face civil and criminal liability
Statement 4: Microgravity is the condition in which people or objects appear to be weightless. This
occurs when gravitation force around is weak (but not zero) or when objects are in a state of free
fall (as in a satellite).
For example, astronauts can move equipment weighing hundreds of pounds with their fingertips
due to microgravity.
Q Source: NASA Website and China’s telescope launch
TEST - 1 (MOCK TEST - 1)
www.insightsias.com 90
© Insights Active Learning| All rights reserved. You may not reproduce, distribute or exploit the contents in any form without written
permission by copyright owner. Copyright infringers may face civil and criminal liability
80 The highest court of appeal in the Vijayanagar Empire was
A. Gram Sabha
B. Mantri Parishad
C. Qazi’s Court
D. The King
Your Answer :
Correct Answer : D
Answer Justification :
Learning: The administration under the Vijayanagar Empire was organized in the way that the king
enjoyed absolute authority in executive, judicial and legislative matters. He was the highest court of
appeal.
The king was assisted by a council of ministers in his day to day administration. But, word of the
king was final.
Moreover, the succession to the throne was on the principle of hereditary.
Q Source: Page 212: TN 11th Standard History Textbook
TEST - 1 (MOCK TEST - 1)
www.insightsias.com 91
© Insights Active Learning| All rights reserved. You may not reproduce, distribute or exploit the contents in any form without written
permission by copyright owner. Copyright infringers may face civil and criminal liability
81 The beginning of agriculture can be ascribed most suitably to which of these times?
A. 2,500 years ago with the onset of Magadha empire
B. 4,700 years ago with the appearance of the first cities on the Indus
C. Nearly 8,000-10,000 years ago
D. About 25,000 years ago in the first Penistone glacial period
Your Answer : B
Correct Answer : C
Answer Justification :
Justification & Learning: Option D: People evolved into their current form some 200,000 years
ago (see human evolution), yet they did not begin to engage in agriculture until about
15,000–10,000 years before the present.
Wild grains were collected and eaten from at least 20,000 BC. So, D is wrong.
Option C: The first agriculture appears to have developed at the closing of the last Pleistocene
glacial period, or Ice Age (about 11,700 years ago).
From around 9,500 BC, the Neolithic founder crops such as emmer wheat, einkorn wheat, hulled
barley, peas etc were cultivated in the Levant. Rice was domesticated in China between 11,500 and
6,200 BC.
Q Source: Page 10: NCERT Class VI Our Pasts - I
TEST - 1 (MOCK TEST - 1)
www.insightsias.com 92
© Insights Active Learning| All rights reserved. You may not reproduce, distribute or exploit the contents in any form without written
permission by copyright owner. Copyright infringers may face civil and criminal liability
82 With reference to the International Tribunal for the Law of the Sea (ITLOS), consider the following:
1. It is an organ of the United Nations adjudicating disputes arising out of the interpretation and
application of the UN Convention on the Law of the Seas (UNCLOS).
2. Each state party of the ITLOS nominates a representative to become a member of ITLOS.
Which of the above is/are correct?
A. 1 only
B. 2 only
C. Both 1 and 2
D. None
Your Answer :
Correct Answer : D
Answer Justification :
Justification: Statement 1: The Tribunal is an independent judicial body but maintains close links
with the United Nations.
The Tribunal and the UN have entered into agreement concerning cooperation and relationship.
UN has granted the Tribunal observer status in the General Assembly.
Statement 2: The Tribunal is composed of 21 independent members, elected from among persons
enjoying the highest reputation for fairness and integrity and of recognized competence in the field
of the law of the sea.
Recently, law expert Neeru Chadha has become the first Indian women to elected to the
International Tribunal for the Law of the Seas (ITLOS).
Neeru has also earned a distinction of becoming the second Indian woman to occupy a top
position at the United Nations after Lakshmi Pandit who served as the President of the United
Nations General Assembly.
Q Source:
http://www.thehindu.com/news/international/neeru-chadha-becomes-1st-indian-woman-as-member-o
f-itlos/article19053500.ece
TEST - 1 (MOCK TEST - 1)
www.insightsias.com 93
© Insights Active Learning| All rights reserved. You may not reproduce, distribute or exploit the contents in any form without written
permission by copyright owner. Copyright infringers may face civil and criminal liability
83 This archaeological site consist of a number of low archaeological mounds created by generations of
superimposed mudbrick structures. Located close to the mouth of the Bolan Pass, it was abandoned
by the time of the emergence of the literate urbanised phase of the Indus Civilisation. An entry in the
UNESCO tentative list it is?
A. Lothal
B. Mehrgarh
C. Inamgaon
D. Surkotada
Your Answer :
Correct Answer : B
Answer Justification :
Learning: It was a small farming village. It is one of the earliest sites with evidence of farming and
herding in South Asia.
Mehrgarh is now seen as a precursor to the Indus Valley Civilization, displaying the whole sequence
from earliest settlement and the start of agriculture, to the mature Harappan Civilisation (based on
the above Q description of site abandonment).
Most of its archaeological deposits are buried deep beneath accumulations of alluvium.
Q Source: Improvisation: Page 29 (image): NCERT Class VI Our Pasts - I
TEST - 1 (MOCK TEST - 1)
www.insightsias.com 94
© Insights Active Learning| All rights reserved. You may not reproduce, distribute or exploit the contents in any form without written
permission by copyright owner. Copyright infringers may face civil and criminal liability
84 Integrated Child Development Services (ICDS) Scheme has a bearing on
1. Infant Mortality rate (IMR)
2. Maternal Mortality rate (MMR)
3. Reduction of malnutrition in children
4. Pre-school non-formal education
Select the correct answer using the codes below.
A. 1 and 3 only
B. 2 and 4 only
C. 1, 2 and 3 only
D. 1, 2, 3 and 4
Your Answer : C
Correct Answer : D
Answer Justification :
Justification: It is a centrally sponsored scheme implemented by the states/UTs. The Scheme was
launched in 1975.
Statement 1 and 3: The scheme has the following objectives:
to improve the nutritional and health status of children in the age-group 0-6 years; this leads
to an improvement in IMR.
to reduce mortality, morbidity, malnutrition and school dropout;
to promote policy coordination and implementation for promoting child development;
to lay an adequate foundation for the psychological, physical and social well-being of the
children etc.
Statement 2 and 4: Under the scheme, the services such as Supplementary nutrition; Pre-school
non-formal education; nutrition and health education, immunization, health check-ups; and Referral
services through Anganwadi Centres for children below 6 years of age as well as to pregnant
women& lactating mothers are offered. These improve MMR.
Learning: Infant mortality rate (IMR) is the number of deaths per 1,000 live births of children
under one year of age. It is presently around 40 in India.
The Maternal mortality rate (MMR) is the annual number of female deaths per 100,000 live births
from any cause related to or aggravated by pregnancy or its management (excluding accidental or
incidental causes). The recent World Bank data puts the MMR for India reported in 2015 at 174.
Q Source: CRY report on children in India
TEST - 1 (MOCK TEST - 1)
www.insightsias.com 95
© Insights Active Learning| All rights reserved. You may not reproduce, distribute or exploit the contents in any form without written
permission by copyright owner. Copyright infringers may face civil and criminal liability
85 The river(s) that do NOT find mention in Rigveda is?
A. Ganga and Yamuna
B. Indus and its other tributaries
C. Saraswati
D. All of the above find mention.
Your Answer : D
Correct Answer : D
Answer Justification :
Learning: Option B: There is a hymn in the form of a dialogue between sage Vishvamitra, and two
rivers, (Beas and Sutlej) that were worshipped as goddesses.
Option C: The most prominent river of the Rigveda is the Sarasvati, next to the Indus.
Option A: The Ganga and Yamuna are named only once.
Q Source: Page 45: NCERT Class VI Our Pasts - I
TEST - 1 (MOCK TEST - 1)
www.insightsias.com 96
© Insights Active Learning| All rights reserved. You may not reproduce, distribute or exploit the contents in any form without written
permission by copyright owner. Copyright infringers may face civil and criminal liability
86 There are plenty of source materials to reconstruct the history of the Gupta period. Some of the
renowned material, in this context, includes
1. Puranas throw light on the royal genealogy of the Gupta kings.
2. Devichandraguptam and Mudhrakshasam provide information regarding the rise of the Guptas
3. Meherauli Iron Pillar Inscription mention their conquests.
4. Fahien’s writings gives an account of the social, economic and religious condition of the empire.
Select the correct answer using the codes below.
A. 1 and 4 only
B. 2, 3 and 4 only
C. 1 and 3 only
D. 1, 2, 3 and 4
Your Answer : D
Correct Answer : D
Answer Justification :
Learning: S1 and S2 mention commonly known sources.
Statement 3: The Meherauli Iron Pillar inscription mentions the extensive conquests of
Chandragupta I. He is considered to be the founder of the Gupta era which starts with his accession
in A.D. 320.
The Allahabad Pillar inscription mentions that Samudragupta defeated twelve rulers in his South
Indian Expedition.
Statement 4: We will cover a separate question on Fahien’s accounts. It is important to note that his
main interest of visiting India was religious.
Q Source: Page 96-97: TN 11th Standard History Textbook
TEST - 1 (MOCK TEST - 1)
www.insightsias.com 97
© Insights Active Learning| All rights reserved. You may not reproduce, distribute or exploit the contents in any form without written
permission by copyright owner. Copyright infringers may face civil and criminal liability
87 Buddhist Viharas are
A. A monument to meditate and chant for the entire Sangha
B. Permanent shelters or monasteries for dwelling and rest
C. Divine extensions of stupas
D. Sites containing relics of Buddha
Your Answer : B
Correct Answer : B
Answer Justification :
Learning: To begin with, both Jaina and Buddhist monks went from place to place throughout the
year, teaching people.
The only time they stayed in one place was during the rainy season, when it was very difficult
to travel.
Then, their supporters built temporary shelters for them in gardens, or they lived in natural
caves in hilly areas.
As time went on, many supporters of the monks and nuns, and they themselves, felt the need
for more permanent shelters and so monasteries were built.
These were known as viharas.
Q Source: Page 71: NCERT Class VI Our Pasts - I
TEST - 1 (MOCK TEST - 1)
www.insightsias.com 98
© Insights Active Learning| All rights reserved. You may not reproduce, distribute or exploit the contents in any form without written
permission by copyright owner. Copyright infringers may face civil and criminal liability
88 PRAGATI is a unique integrating and interactive platform to
1. Address the grievances of citizens
2. Monitoring and reviewing important government projects
Which of the above is/are correct?
A. 1 only
B. 2 only
C. Both 1 and 2
D. None
Your Answer : B
Correct Answer : C
Answer Justification :
Justification: PRAGATI stands for Pro-Active Governance And Timely Implementation.
Key features of the PRAGATI application are as follows:
It is a three-tier system (PMO, Union Government Secretaries, and Chief Secretaries of the
States);
Prime Minister will hold a monthly programme where he will interact with the Government of
India Secretaries, and Chief Secretaries through Video-conferencing enabled by data and geoinformatics
visuals;
Issues to be flagged before the PM are picked up from the available database regarding
Public Grievances, on-going Programmes and pending Projects;
It will also take into consideration various correspondences to PM’s office by the common
people or from high dignitaries of States and/or developers of public projects;
Q Source: PIB AKT/NT: Release ID :167366
TEST - 1 (MOCK TEST - 1)
www.insightsias.com 99
© Insights Active Learning| All rights reserved. You may not reproduce, distribute or exploit the contents in any form without written
permission by copyright owner. Copyright infringers may face civil and criminal liability
89 In ancient Southern India, kadaisiyar and adimai were used to refer to
A. Ordinary ploughmen
B. Landless labourers, including slaves
C. Tax officials
D. Large landowners
Your Answer :
Correct Answer : B
Answer Justification :
Learning: Society was segregated even in ancient India.
There were at least three different kinds of people living in most villages in the southern and
northern parts of the ancient Indian subcontinent.
In the Tamil region, large landowners were known as vellalar, ordinary ploughmen were known as
uzhavar, and landless labourers, including slaves, were known as kadaisiyar and adimai.
In the northern region, this distinction was made in terms of land, political position and assets held.
Q Source: Page 88: NCERT Class VI Our Pasts - I
TEST - 1 (MOCK TEST - 1)
www.insightsias.com 100
© Insights Active Learning| All rights reserved. You may not reproduce, distribute or exploit the contents in any form without written
permission by copyright owner. Copyright infringers may face civil and criminal liability
90 Popularly known Jatakas were
A. Stories of previous lives of the Buddha
B. Objects of concentration
C. Donation passed from student to teacher
D. Arahants who sacrificed their lives for the service of Sangha
Your Answer : A
Correct Answer : A
Answer Justification :
Learning: They are a voluminous body of literature native to India concerning the previous births
of Gautama Buddha in both human and animal form.
The future Buddha may appear as a king, an outcast, a god, an elephant—but, in whatever
form, he exhibits some virtue that the tale thereby inculcates.
In Theravada Buddhism, the Jatakas are a textual division of the Pali Canon, included in the
Khuddaka Nikaya of the Sutta Pitaka.
The term Jataka may also refer to a traditional commentary on this book. Q Source: Page 89:
NCERT Class VI Our Pasts – I
TEST - 1 (MOCK TEST - 1)
www.insightsias.com 101
© Insights Active Learning| All rights reserved. You may not reproduce, distribute or exploit the contents in any form without written
permission by copyright owner. Copyright infringers may face civil and criminal liability
91 Consider the following statements.
1. The Anguttara Nikaya, a part of Abhidhamma Pitaka, contains thematically linked discourses of
Buddha.
2. The Majjhima Nikaya, a segment of Vinaya Pitaka, contains the smallest discourses given by the
Buddha to his close disciples.
Which of the above is/are correct?
A. 1 only
B. 2 only
C. Both 1 and 2
D. None
Your Answer :
Correct Answer : D
Answer Justification :
Justification: Both are a part of Sutta Pitaka. The word Nikaya is most commonly used in reference
to the Buddhist texts of the Sutta Piáš­aka.
Statement 1: This is incorrect because Abhidhamma Pitaka is a detailed scholastic reworking of
material appearing in the Suttas, not a collection of original discourses by the Buddha himself.
Statement 2: This is incorrect because Vinaya pitaka contains rules for the Sangha, it isn’t known
for hosting discourses.
Q Source: Improvisation: Page 47: TN 11th Standard History Textbook
TEST - 1 (MOCK TEST - 1)
www.insightsias.com 102
© Insights Active Learning| All rights reserved. You may not reproduce, distribute or exploit the contents in any form without written
permission by copyright owner. Copyright infringers may face civil and criminal liability
92 Changthangi goat of Northern Himalayan region is known for yielding
A. Pashmina wool
B. Shahtoosh
C. Himayalan Yew
D. Pitas and Humas
Your Answer : A
Correct Answer : A
Answer Justification :
Learning: They are a breed of goat inhabiting the plateaus in Tibet and neighbouring areas of
Ladakh in Jammu & Kashmir.
They are raised for ultra-fine cashmere wool Pashmina, which literally translates to "Soft Gold" in
Kashmiri.
The Changthangi goats have revitalized the poor economy of Changthang, Leh and Ladakh region.
Q Source: Page 9: NCERT Class VI Social and Political Life - I
TEST - 1 (MOCK TEST - 1)
www.insightsias.com 103
© Insights Active Learning| All rights reserved. You may not reproduce, distribute or exploit the contents in any form without written
permission by copyright owner. Copyright infringers may face civil and criminal liability
93 Consider the following about the role and contributions of B.R. Ambedkar in modern India:
1. He inspired the Dalit Buddhist Movement.
2. He was Independent India's first law minister.
3. He served as a Governor of Reserve Bank of India.
Select the correct answer using the codes below.
A. 1 and 2 only
B. 2 and 3 only
C. 3 only
D. 1 and 3 only
Your Answer :
Correct Answer : A
Answer Justification :
Justification: Statement 1: He spent his life advocating political rights and social freedom for
Dalits. In 1956 he converted to Buddhism, initiating mass conversions of Dalits.
Statement 3: The Reserve Bank of India (RBI), was based on the ideas that Ambedkar presented to
the Hilton Young Commission. He, however, did not serve as its Governor.
Ambedkar was the first Indian to pursue a doctorate in economics abroad. He argued that
industrialisation and agricultural growth could enhance the Indian economy.
Q Source: Improvisation: Page 20: NCERT Class VI Social and Political Life - I
TEST - 1 (MOCK TEST - 1)
www.insightsias.com 104
© Insights Active Learning| All rights reserved. You may not reproduce, distribute or exploit the contents in any form without written
permission by copyright owner. Copyright infringers may face civil and criminal liability
94 SATH - ‘Sustainable Action for Transforming Human capital’ initiative of NITI Aayog focuses on
A. Creating ‘role model’ states for health systems
B. Revamping higher education
C. Building workforce for the future in areas of cutting edge technology
D. Eradicating manual scavenging
Your Answer : C
Correct Answer : A
Answer Justification :
Learning: This was launched with the State Governments for furthering the agenda of cooperative
federalism.
NITI Aayog under the SATH initiative plans to identify and build three future ‘role model’
states for health systems.
After identification of three states, NITI Aayog will work in close collaboration with the state
machinery of these three states.
It will help in designing a robust roadmap, develop a program governance structure, establish
monitoring and tracking mechanisms, and provide support to the state institutions to achieve
the end objectives.
SATH program will be implemented by NITI Aayog along with McKinsey & Company and IPE
Global consortium.
Parameters such as MMR, IMR, incidence of malaria will be considered for determining
potential impact.
Q Source:
http://www.thehindu.com/business/niti-aayog-unveils-sath-programme/article18956584.ece
TEST - 1 (MOCK TEST - 1)
www.insightsias.com 105
© Insights Active Learning| All rights reserved. You may not reproduce, distribute or exploit the contents in any form without written
permission by copyright owner. Copyright infringers may face civil and criminal liability
95 The decision to locate a common well in a particular area of the village can be taken by
A. Central Government only
B. State Government only
C. District bureaucracy only
D. Panchayati Raj Institutions (PRIs)
Your Answer : D
Correct Answer : D
Answer Justification :
Learning: PRIs were established with an objective of democratic decentralization.
State legislatures can devolve powers to PRIs under 73rd constitutional amendment. Such powers
can involve management of local markets, public works, sanitation, public health etc.
Under these provisions, a public well can be established by PRIs.
Q Source: Improvisation: Page 30: NCERT Class VI Social and Political Life - I
TEST - 1 (MOCK TEST - 1)
www.insightsias.com 106
© Insights Active Learning| All rights reserved. You may not reproduce, distribute or exploit the contents in any form without written
permission by copyright owner. Copyright infringers may face civil and criminal liability
96 With reference to Buddhist disciples, consider the following.
1. Sariputta was a chief female disciple of Gautama Buddha.
2. Khema, who joined the Buddhist Sangha, was one of the queens of King Bimbisara.
3. Moggallana, a disciple of Buddha was known for his psychic powers.
4. King Ajatasatru of Magadha and King Prasenajit of Kosala became Buddha’s disciples.
Select the correct answer using the codes below.
A. 1 and 2 only
B. 3 and 4 only
C. 2, 3 and 4 only
D. 1 and 3 only
Your Answer :
Correct Answer : C
Answer Justification :
Justification: Statement 1: Sariputta was one of two chief male disciples of Gautama Buddha along
with Moggallana, counterparts to the bhikkhunis Khema and Uppalavanna, his two chief female
disciples. They were to maintain the order of monks and nuns.
Statement 2: The conversion of Khema was one of the rare cases where the Buddha used his
psychic powers to make a change in the heart of another.
Statement 3: Moggallana attained enlightenment shortly after joining the Sangha. As a teacher, he
became known for his psychic powers, which he used extensively in his teaching methods.
Statement 4: Kings like Prasenajit of Kosala and Bimbisara and Ajatasatru of Magadha too accepted
the doctrines of the Buddha and became his disciples.
Q Source: Improvisation: Page 41: TN 11th Standard History Textbook
TEST - 1 (MOCK TEST - 1)
www.insightsias.com 107
© Insights Active Learning| All rights reserved. You may not reproduce, distribute or exploit the contents in any form without written
permission by copyright owner. Copyright infringers may face civil and criminal liability
97 With reference to the International Telecommunication Union (ITU), consider the following
statements.
1. It is a non-profit non-governmental organization comprising private sector ISP representatives.
2. It releases the Global Cybersecurity Index (GCI).
3. It is responsible for allocating global radio spectrum and satellite orbits.
4. It develops the technical standards that ensure networks and technologies to seamlessly
interconnect.
5. NASSCOM is the nodal agency for coordinating with ITU from India, as a member country.
6. It strives to improve access to Information and Communication Technology among the underserved
communities worldwide.
Select the correct answer using the codes below.
A. 1, 4 and 5 only
B. 2, 3, 4 and 6 only
C. 3, 5 and 6 only
D. 1, 2, 3, 4, 5 and 6
Your Answer : D
Correct Answer : B
Answer Justification :
Justification: Statement 1: ITU is the United Nations specialised agency for information and
communication technologies (ICTs). ITU has 193 countries as its members and also 800 privatesector
entities and academic institutions.
Statement 2: India has been ranked 23rd (“maturing category” of the index ) out of 165 nations in
the second Global Cybersecurity Index (GCI) that measures the commitment of nations across the
world to cyber security.
ITU has ranked the countries based on the countries’ legal, technical and organisational
institutions, educational and research capabilities, as well as their cooperation in informationsharing
networks.
Statement 3 and 4: There must be a global agency that coordinates these standards (lie ICANN),
else telecommunication between different nations would have been difficult.
Statement 6: India is a member country since 1869 (british times), but it is the Department of
Telecommunications that is the nodal agency for such coordination.
Q Source: Release of Global Cybersecurity Index (GCI) 2017
TEST - 1 (MOCK TEST - 1)
www.insightsias.com 108
© Insights Active Learning| All rights reserved. You may not reproduce, distribute or exploit the contents in any form without written
permission by copyright owner. Copyright infringers may face civil and criminal liability
98 Firoz Tughlaq’s welfare state included which of these administrative measures?
1. Treating Hindus and Muslims alike in terms of political and economic rights
2. Abolition of the practice of slavery
3. Establishment of government department to take care of orphans and widows
Select the correct answer using the codes below.
A. 1 and 3 only
B. 2 and 3 only
C. 3 only
D. 1 and 2 only
Your Answer :
Correct Answer : C
Answer Justification :
Justification: Statement 1: As Firoz was guided by the ulemas, he was intolerant towards Shia
Muslims and Sufis. He treated Hindus as second grade citizens and imposed Jiziya. In this respect
he was the precursor of Sikandar Lodi and Aurangazeb.
Statement 2: Also he increased the number of slaves by capturing the defeated soldiers and young
persons. So, 2 is wrong.
Statement 3: A new department called Diwan-i-Khairat was created to take care of orphans and
widows. Free hospitals and marriage bureaus for poor Muslims were also established.
Q Source: Page 185-186: TN 11th Standard History Textbook
TEST - 1 (MOCK TEST - 1)
www.insightsias.com 109
© Insights Active Learning| All rights reserved. You may not reproduce, distribute or exploit the contents in any form without written
permission by copyright owner. Copyright infringers may face civil and criminal liability
99 With reference to the National Mission on Education through ICT (NMEICT), consider the following:
1. Certification of attainments of any kind at any level acquired through formal or non-formal means in
conventional or non-conventional fields
2. Platform for sharing of ideas and techniques and pooling of knowledge resources
3. Scholarship and Talent management including identification, nurturing and disbursement
electronically
Which of the above is/are the objectives of NMEICT?
A. 1 and 2 only
B. 2 only
C. 1 and 3 only
D. 1, 2 and 3
Your Answer :
Correct Answer : D
Answer Justification :
Learning: These are the guiding philosophies for NMECIT –
(a) no talent of the country should be allowed to go waste,
(b) all the services available through the content delivery portal Sakshat (NMEICT) should be free
and freely available material on the web should be used so as to avoid reinventing the wheel.
Scheme: It has been envisaged as a Centrally Sponsored Scheme to leverage the potential of ICT,
in teaching and learning process for the benefit of all the learners in Higher Education Institutions
in any time anywhere mode.
The Mission has two major components:
providing connectivity, along with provision for access devices, to institutions and learners;
Content generation.
The content portion of this Mission would have an ambitious vision of catering to the learning needs
of more than 50 crore Indians (working population) and of providing a one stop solution to all the
requirements of the learning community.
The objectives and details can be read at
http://mhrd.gov.in/sites/upload_files/mhrd/files/upload_document/MissionDocument.pdf
TEST - 1 (MOCK TEST - 1)
www.insightsias.com 110
© Insights Active Learning| All rights reserved. You may not reproduce, distribute or exploit the contents in any form without written
permission by copyright owner. Copyright infringers may face civil and criminal liability
100 India Infrastructure Fund (IIF)
1. is a SEBI-registered domestic venture capital fund
2. sponsored entirely by foreign governments
3. invests only in greenfield projects
Select the correct answer using the codes below.
A. 1 only
B. 2 and 3 only
C. 1 and 3 only
D. None of the above
Your Answer :TEST - 1 (MOCK TEST - 1)
www.insightsias.com 1
© Insights Active Learning| All rights reserved. You may not reproduce, distribute or exploit the contents in any form without written
permission by copyright owner. Copyright infringers may face civil and criminal liability
1 Social Entrepreneurship is running
A. Any non-profit business
B. An ecologically sustainable business
C. An inclusive business that has participation from major social groups and local community
D. A business that makes its money in a socially responsible way
Your Answer :
Correct Answer : D
Answer Justification :
Learning: It is the use of the techniques by start-up companies and other entrepreneurs to develop,
fund and implement solutions to social, cultural, or environmental issues.
Muhammad Yunus (Grameen Bank) was a famous social entrepreneur. Another example can be a
crowdfunding internet platform like https://milaap.org
These ventures are not necessarily formed to reinvest all profits into the communities.
So, these firms apply commercial strategies to maximize improvements in human and
environmental well-being.
Q Source: Often seen in news: A recent speech by the PM
TEST - 1 (MOCK TEST - 1)
www.insightsias.com 2
© Insights Active Learning| All rights reserved. You may not reproduce, distribute or exploit the contents in any form without written
permission by copyright owner. Copyright infringers may face civil and criminal liability
2 Which of these are the chief difference(s) between Mesolithic and Neolithic cultures?
1. Hunting-gathering pattern of life began not before the Neolithic period.
2. Mesolithic cultures did not practice domestication of animals unlike Neolithic people.
3. The use of bow and arrow cannot be found in Mesolithic culture, unlike Neolithic culture.
4. Evidence of pottery is absent from the Neolithic culture but is markedly found in Mesolithic culture.
Select the correct answer using the codes below.
A. 3 and 4 only
B. 2 only
C. 1 and 3 only
D. None of the above
Your Answer : B
Correct Answer : D
Answer Justification :
Justification: Statement 1 and 3: Hunting-gathering pattern of life was prevalent since Old stone
age to Mesolithic and Neolithic.
However, there seems to have been a shift from big animal hunting to small animal hunting and
fishing. This is how bows and arrows were used in this period. Both 1 and 3 are thus wrong.
Statement 2: Domestication of animals, horticulture and primitive cultivation started during
Mesolithic period itself.
However, during Neolithic period, domestication of sheep, goats and cattle was widely prevalent.
Cattle were used for cultivation and for transport.
Statement 4: During Neolithic age, wheels were used to make pottery. Pottery was used for cooking
as well as storage of food grains. So, 4 is wrong.
Learning: During the Neolithic phase, the cultivation of plants and domestication of animals led to
the emergence of village communities based on sedentary life.
There was a great improvement in technology of making tools and other equipments used by
man. Stone tools were now polished.
Mud brick houses were built instead of grass huts.
Large urns were used as coffins for the burial of the dead.
Wheat, barely, rice, millets were cultivated in different areas at different points of time. Rice
cultivation was extensive in eastern India.
The people of Neolithic Age used clothes made of cotton and wool
TEST - 1 (MOCK TEST - 1)
www.insightsias.com 3
© Insights Active Learning| All rights reserved. You may not reproduce, distribute or exploit the contents in any form without written
permission by copyright owner. Copyright infringers may face civil and criminal liability
Q Source: Page 13-14: TN 11th Standard History Textbook
56154 - ROHIT YADAV - rohit.kshaurya@gmail.com - 9410878797
TEST - 1 (MOCK TEST - 1)
Total Marks : 200
Mark Scored :
www.insightsias.com 4
© Insights Active Learning| All rights reserved. You may not reproduce, distribute or exploit the contents in any form without written
permission by copyright owner. Copyright infringers may face civil and criminal liability
3 The Government of India has recently announced the sale of “Government of India Floating Rate Bonds
2024”. With reference to government securities, consider the following:
1. Floating Rate Bonds are sold by the Reserve Bank of India.
2. RBI issues Cash Management Bills (CMBs) to meet long-term borrowing needs of the Government
of India.
3. Treasury bills are short term debt instruments issued by the Government of India.
Select the correct answer using the codes below.
A. 1 and 2 only
B. 1 and 3 only
C. 2 only
D. 2 and 3 only
Your Answer : B
Correct Answer : B
Answer Justification :
Concept: A Government security is a tradable instrument issued by the Central Government or the
State Governments.
It acknowledges the Government’s debt obligation (to raise money from the market for
various purposes).
Such securities are short term (usually called treasury bills, with original maturities of less
than one year) or long term (usually called Government bonds or dated securities with
original maturity of one year or more).
Besides providing a return in the form of coupons (interest), Government securities offer the
maximum safety as they carry the Sovereign’s commitment for payment of interest and
repayment of principal.
Justification: Statement 1: There are two types of bonds:
Fixed Rate Bonds – These are bonds on which the coupon rate is fixed for the entire life of the
bond. Most Government bonds are issued as fixed rate bonds.
Floating Rate Bonds – Floating Rate Bonds are securities which do not have a fixed coupon rate.
The coupon is re-set at pre-announced intervals (say, every six months or one year) by adding a
spread over a base rate. These are issued by RBI.
Statement 2: Cash Management Bills (CMBs), are a relatively new short-term instrument to meet
the temporary mismatches in the cash flow of the Government. The CMBs have the generic
character of T-bills but are issued for maturities less than 91 days.
Statement 3: Treasury bills or T-bills, which are money market instruments, are short term debt
instruments issued by the Government of India and are presently issued in three tenors, namely, 91
day, 182 day and 364 day.
TEST - 1 (MOCK TEST - 1)
www.insightsias.com 5
© Insights Active Learning| All rights reserved. You may not reproduce, distribute or exploit the contents in any form without written
permission by copyright owner. Copyright infringers may face civil and criminal liability
Q Source: RBI Bond sales
TEST - 1 (MOCK TEST - 1)
www.insightsias.com 6
© Insights Active Learning| All rights reserved. You may not reproduce, distribute or exploit the contents in any form without written
permission by copyright owner. Copyright infringers may face civil and criminal liability
4 Ashokan inscriptions remain valuable sources for the study of Asoka and the Mauryan Empire. This is
because
1. His wooden pillars heralded a new architectural era because most of the monuments before his
period were made of stone.
2. Pillar edicts give a summary of his efforts to promote the Dhamma within his kingdom and outside.
3. They often deal with instructions given to his officials which is informative of Mauryan polity.
Select the correct answer using the codes below.
A. 2 and 3 only
B. 1 and 3 only
C. 1 and 2 only
D. 2 only
Your Answer : A
Correct Answer : A
Answer Justification :
Justification: Statement 1: The monuments before the period of Asoka were mostly made of wood
and therefore perished.
The use of stone started from the time of Asoka. Even of the numerous monuments of Asoka, only a
few have remained.
His palace and monasteries and most of his stupas have disappeared. The only remaining stupa is at
Sanchi.
Statement 2: The state and spread of Dhamma gives crucial information about the social life at the
time of Ashoka.
Statement 3: Ashoka often gave instructions to his Dhamma mahamahattas to spread Dhamma.
These instructions, their style etc were recorded on the edicts.
Learning: Ashokan inscriptions (first deciphered by James Princep) are written in Pali language
and in some places Prakrit was used.
The Brahmi script was employed for writing. In the northwestern India Asokan inscriptions were
found in Karoshti script.
There are fourteen Major Rock Edicts. The two Kaling Edicts are found in the newly conquered
territory. The major pillar Edicts were erected in important cities. There are minor Rock Edicts and
minor pillar Edicts.
Q Source: Page 60-61: TN 11th Standard History Textbook
TEST - 1 (MOCK TEST - 1)
www.insightsias.com 7
© Insights Active Learning| All rights reserved. You may not reproduce, distribute or exploit the contents in any form without written
permission by copyright owner. Copyright infringers may face civil and criminal liability
5 Which of the following is the closest analogy of the Indian Constitution?
A. Rulebook
B. Anthology
C. Dictionary
D. Social Register
Your Answer : A
Correct Answer : A
Answer Justification :
Justification: Option A: The Indian Constitution lays down the basic rules or laws that have to be
followed by everyone. These laws are for both the government and the people. Hence, A is
appropriate.
Option B: An anthology is a collection of literary works, originally of poems, which constitution is
not.
Option C: Constitution is not a mere dictionary of political and legal terms, it is much more than
that.
Option D: Social Register (SR) is a directory of people from certain social classes. Constitution is for
everyone. It isn’t a directory.
Q Source: Improvisation: Page 39: NCERT Class VI Social and Political Life - I
TEST - 1 (MOCK TEST - 1)
www.insightsias.com 8
© Insights Active Learning| All rights reserved. You may not reproduce, distribute or exploit the contents in any form without written
permission by copyright owner. Copyright infringers may face civil and criminal liability
6 ‘JIGYASA’, recently seen in news is a
A. Interstellar exploration programme of ISRO
B. Student- scientist connect programme
C. Earth core drilling programme of Ministry of Earth Sciences
D. Biodiversity documentation programme in the Western Ghats
Your Answer : B
Correct Answer : B
Answer Justification :
Learning: Council of Scientific and Industrial Research (CSIR), has joined hands with Kendriya
Vidyalaya Sangathan (KVS) to implement this programme.
The program will also enable the students and teachers to practically live the theoretical concepts
taught in science by visiting CSIR laboratories and by participating in mini-science projects. The
model of engagement includes:
Student Residential Programmes;
Scientists as Teachers and Teachers as Scientists;
Lab specific activities / Onsite Experiments;
The focus is on connecting school students and scientists so as to extend student’s classroom
learning with that of a very well-planned research laboratory based learning.
Q Source: http://pib.nic.in/newsite/PrintRelease.aspx?relid=167194
TEST - 1 (MOCK TEST - 1)
www.insightsias.com 9
© Insights Active Learning| All rights reserved. You may not reproduce, distribute or exploit the contents in any form without written
permission by copyright owner. Copyright infringers may face civil and criminal liability
7 Consider the following statements. Assertion (A): Every elector at the Presidential election is bound by
the voting instructions issued by the whip of the respective political party. Reason (R): Anti-defection
provisions are enforced by the constitution under the Tenth Schedule.
In the context of the above, which of these is correct?
A. A is correct, and R is an appropriate explanation of A.
B. A is correct, but R is not an appropriate explanation of A.
C. A is correct, but R is incorrect.
D. A is incorrect, but R is correct.
Your Answer : C
Correct Answer : D
Answer Justification :
Concept: Defection is defined as “to abandon a position or association, often to join an opposing
group” which essentially describes a situation when a member of a particular party abandons his
loyalty towards that party and provide his support (in the form of his vote or otherwise) to another
party.
To avoid such a situation (about which you’ll read later in Laxmikanth and 11th NCERT), an antidefection
provision was added in the constitution. If a member of a political party disobeys party
directions concerning matters such as voting on a bill or for a candidate, this amounts to defection.
The member is then liable to be punished by the party.
Justification: However, the ECI has clarified recently that voting or not voting as per his/her own
free will at the Presidential election and the electors are at liberty to vote or not to vote at the
Presidential election as per their own free will and choice.
This will equally apply to the political parties and they are free to canvas or seek votes of
electors for any candidate or requesting or appealing to them to refrain from voting.
However, the political parties cannot issue any direction or whip to their members to vote in a
particular manner or not to vote at the election leaving them with no choice, as that would
tantamount to the offence of undue influence within the meaning of section 171C of the IPC.
This is because, as per the ECI, voting at election to the office of President is different from
voting by a member of Parliament or State Legislature inside the House.
Q Source: RM/ RS: Release ID :167193
TEST - 1 (MOCK TEST - 1)
www.insightsias.com 10
© Insights Active Learning| All rights reserved. You may not reproduce, distribute or exploit the contents in any form without written
permission by copyright owner. Copyright infringers may face civil and criminal liability
8 With reference to unique BRICS countries initiatives, consider the following statements:
1. BRICS Network University (NU) is an open educational project and does not involve establishment
of rigid organizational forms such as a secretariat.
2. BRICS Think Tanks Council (BTTC) comprises research organizations from BRICS countries
including Observer Research Foundation (ORF) from India.
Which of the above is/are correct?
A. 1 only
B. 2 only
C. Both 1 and 2
D. None
Your Answer :
Correct Answer : C
Answer Justification :
Justification: Recently at the 5th BRICS Education ministers meeting, Beijing declaration on
education was adopted which advocated support for both these organizations.
Statement 1: NU is a consortium of the universities, elaborating Master and PhD programmes in
different disciplines. It was established in 2015 in BRICS meeting at Moscow.
NU follows principles of:
Openness, which enables various universities to join the network respecting national criteria.
Focus on educational programmes that can be supplemented with network research and
innovation projects.
There is also a BRICS Universities League which is a consortium of leading research universities
from BRICS countries.
Statement 2: The BTTC (established in 2013) comprises the Institute for Applied Economic
Research (IPEA) (Brazil), the National Committee for BRICS Research (NRC/BRICS) (Russia), the
Observer Research Foundation (ORF) etc.
The main purposes of its establishment was to form a platform for the exchange of ideas and
present policy recommendations for BRICS leaders.
Q Source: Beijing declaration on Education: 5th July 2017:
http://pib.nic.in/newsite/PrintRelease.aspx?relid=167154
TEST - 1 (MOCK TEST - 1)
www.insightsias.com 11
© Insights Active Learning| All rights reserved. You may not reproduce, distribute or exploit the contents in any form without written
permission by copyright owner. Copyright infringers may face civil and criminal liability
9 With reference to the Sangam age, Panar and Viraliyar were
A. Temple guards
B. Bull fighters
C. Singing bards
D. Charity donors
Your Answer :
Correct Answer : C
Answer Justification :
Learning: Poetry, music and dancing were popular among the people of the Sangam age. Liberal
donations were given to poets by the kings, chieftains and nobles.
The royal courts were crowded with singing bards called Panar and Viraliyar. They were
experts in folk songs and folk dances.
The arts of music and dancing were highly developed. A variety of Yazhs and drums are
referred to in the Sangam literature.
Dancing was performed by Kanigaiyar. Koothu was the most popular entertainment of the
people.
Q Source: Page 91: TN 11th Standard History Textbook
TEST - 1 (MOCK TEST - 1)
www.insightsias.com 12
© Insights Active Learning| All rights reserved. You may not reproduce, distribute or exploit the contents in any form without written
permission by copyright owner. Copyright infringers may face civil and criminal liability
10 The 600-year-old Walled City of Ahmedabad was recently declared a World Heritage City. With
reference to the World Heritage Cities Programme and associated initiatives, consider the following:
1. It is a thematic programmes formally approved and monitored by the World Heritage Committee.
2. The programme has provisions for technical assistance to States Parties for the implementation of
new approaches and schemes for urban heritage conservation.
3. IUCN has set up the ‘Historic Urban Landscape initiative’ which is an international working group
comprising, inter alia, UNESCO as an advisory body to the World Heritage Convention.
4. The Organization of World Heritage Cities (OWHC) is an inter-governmental organization of the
municipal corporations of cities in which sites of the UNESCO World Heritage list are located.
Select the correct answer using the codes below.
A. 1 and 2 only
B. 3 and 4 only
C. 1, 2 and 3 only
D. 1, 2, 3 and 4
Your Answer :
Correct Answer : A
Answer Justification :
Justification: Statement 1 and 2: It aims to assist States Parties in the challenges of protecting and
managing their urban heritage. The programme is structured along a two-way process, with 1) the
development of a theoretical framework for urban heritage conservation, and 2) the provision of
technical assistance to States Parties for the implementation of new approaches and schemes.
Statement 3: As part of this policy process of conservation, UNESCO (not IUCN) set up the Historic
Urban Landscape initiative, an international working group comprising ICOMOS, IUCN and
ICCROM (as Advisory Bodies to the 1972 World Heritage Convention) and other partner
organizations, including UN-Habitat etc.
Statement 4: OWHC is an international non-profit, non-governmental organization of 250 cities in
which sites of the UNESCO World Heritage list are located.
Learning: It is first Indian city get the honour. The Walled City of Ahmedabad is located on the
eastern banks of Sabarmati river.
It was founded by Sultan Ahmed Shah I of Gujarat Sultanate in in 1411.
It remained the capital of the Gujarat Sultanate and later important political and commercial
centre of Gujarat.
It presents a rich architectural heritage from the sultanate period, notably the Bhadra citadel,
walls and gates of the Fort city and numerous mosques and tombs, as well as important Hindu
and Jain temples of later periods.
Q Source: Improvisation:
http://www.thehindu.com/news/national/other-states/ahmedabad-declared-indias-first-heritage-cityTEST
- 1 (MOCK TEST - 1)
www.insightsias.com 13
© Insights Active Learning| All rights reserved. You may not reproduce, distribute or exploit the contents in any form without written
permission by copyright owner. Copyright infringers may face civil and criminal liability
by-unesco/article19244478.ece
TEST - 1 (MOCK TEST - 1)
www.insightsias.com 14
© Insights Active Learning| All rights reserved. You may not reproduce, distribute or exploit the contents in any form without written
permission by copyright owner. Copyright infringers may face civil and criminal liability
11 What argument(s) would you advance to justify that the position of women had become miserable
during the Gupta period?
1. Women were prohibited from studying religious texts.
2. The practice of Swyamvara was given up and Manusmriti suggested early marriage for girls.
Which of the above is/are correct?
A. 1 only
B. 2 only
C. Both 1 and 2
D. None
Your Answer : A
Correct Answer : C
Answer Justification :
Justification: Statement 1: This ensured the domination of men over women.
Statement 2: Swaymvara was the practice of women choosing their own partners by way of a
competition/selection. When this was given up, women lost an important right. Manusmriti (law
book) further entrenched patriarchal rules to worsen the position of women.
Learning: During the Gupta dynasty period the male members of a family were entitled to inherit
property left behind by their fathers. However, such right of inheritance was not given to women or
girls.
The power of inheritance enabled the male members of a family to exercise dominance over the
women.
Q Source: Page 104: TN 11th Standard History Textbook
TEST - 1 (MOCK TEST - 1)
www.insightsias.com 15
© Insights Active Learning| All rights reserved. You may not reproduce, distribute or exploit the contents in any form without written
permission by copyright owner. Copyright infringers may face civil and criminal liability
12 The year of 2017 marks 25 years of dialogue partnership between India and ASEAN and 15 years of
summit-level interaction. With reference to India-ASEAN relations, consider the following:
1. India's relationship with ASEAN is a key pillar of our Act East Policy.
2. AIFTA is a free trade area among the member states of the ASEAN and India.
3. India and ASEAN are yet to upgrade their relationship to a ‘strategic partnership’.
Select the correct answer using the codes below.
A. 2 only
B. 1 and 3 only
C. 1 and 2 only
D. 3 only
Your Answer : C
Correct Answer : C
Answer Justification :
Justification: Statement 1: India’s commitment to strengthen its ties with ASEAN countries is
reflected by its ‘Look East Policy’ becoming ‘Act East Policy.’ Three Cs are the focus -Commerce,
Connectivity and Culture.
Statement 2: ASEAN became ASEAN Plus Six with additional countries: Australia, New Zealand and
India. Codification of the relations between these nations has seen progress through the
development of the Regional Comprehensive Economic Partnership (RCEP), a proposed free-trade
agreement involving the 16 countries of ASEAN plus six.
Statement 3: India and ASEAN had upgraded their relationship to a strategic partnership in 2012.
ASEAN consists of 10-member countries- Brunei, Cambodia, Indonesia, Laos, Malaysia, Myanmar,
Philippines, Singapore, Thailand and Vietnam.
It is a regional organisation that aims to promote intergovernmental cooperation and economic
integration amongst its members.
Q Source: 25 years of Dialogue partnership between India-ASEAN
TEST - 1 (MOCK TEST - 1)
www.insightsias.com 16
© Insights Active Learning| All rights reserved. You may not reproduce, distribute or exploit the contents in any form without written
permission by copyright owner. Copyright infringers may face civil and criminal liability
13 Universal suffrage (voting rights to all eligible) was granted to women all over India in
A. 1950 once Indian republic was formed
B. Morley-Minto Act
C. Government of India Act, 1935
D. 1939 General elections
Your Answer : B
Correct Answer : A
Answer Justification :
Justification & Learning: First local effort was made in Madras, which granted women’s suffrage
in 1921, but only to those men and women who owned land property according to British
administration's records. This was neither Universal nor equitable.
In 1950, universal suffrage granted voting rights to all women. This is enshrined in Article 326 in
our constitution.
Q Source: Improvisation: Page 32: NCERT Class VI Social and Political Life - I
TEST - 1 (MOCK TEST - 1)
www.insightsias.com 17
© Insights Active Learning| All rights reserved. You may not reproduce, distribute or exploit the contents in any form without written
permission by copyright owner. Copyright infringers may face civil and criminal liability
14 Karla is one of the most famous centres of early rock-cut architecture and has received much
attention of scholars and commoners alike. What is important about Karla caves?
1. The grand Chaitya griha of Karla is the largest of all the chaita-grihas of India.
2. The rock-cut cave was completed without accepting any financial donation from individuals apart
from their voluntary labour.
Which of the above is/are correct?
A. 1 only
B. 2 only
C. Both 1 and 2
D. None
Your Answer :
Correct Answer : A
Answer Justification :
Justification: Statement 1: The chaityagriha at Karla is the biggest of its type in the whole of India.
The hall measures around thirty eight m deep from door to back. The object of worship is the stupa
at the rear end of the chaityagriha.
Statement 2: The caves were caused from the donations and support of a group of assorted
individuals. This includes a prince of Maharathi family; merchants, monks etc.
The inscriptions of Usavadata and Vasisthiputra Pulumavi mention donation of land to the Veluraka
Samgha for this.
Q Source: Page 105: NCERT Class VI Our Pasts - I
TEST - 1 (MOCK TEST - 1)
www.insightsias.com 18
© Insights Active Learning| All rights reserved. You may not reproduce, distribute or exploit the contents in any form without written
permission by copyright owner. Copyright infringers may face civil and criminal liability
15 With reference to the National Disaster Response Force (NDRF), consider the following statements:
1. President of India serves as the Director General (DG) of NDRF.
2. NDRF was established under the Disaster Management Act, 2005.
3. It is housed directly under the Prime Minister’s Office and functions from there.
Select the correct answer using the codes below.
A. 2 only
B. 1 and 3 only
C. 1 only
D. 2 and 3 only
Your Answer : D
Correct Answer : A
Answer Justification :
Justification: NDRF is India’s elite disaster mitigation combat force established under Disaster
Management Act, 2005
Statement 1: Senior IPS officer Sanjay Kumar was recently appointed as Director General (DG) of
National Disaster Response Force (NDRF).
Statement 3: It functions under Union Ministry of Home Affairs. It is mandated to undertake special
disaster response, relief, rescue operations and combat roles independently in the case of an event
of any disaster (natural or man-made), accident or emergency.
It also assists local authorities in launching a quick rescue and response operation to save life and
property.
Q Source:
http://www.thehindu.com/news/national/andhra-pradesh/ndrf-ready-to-tackle-emergencies-in-monso
on-says-its-commandant/article18958062.ece
TEST - 1 (MOCK TEST - 1)
www.insightsias.com 19
© Insights Active Learning| All rights reserved. You may not reproduce, distribute or exploit the contents in any form without written
permission by copyright owner. Copyright infringers may face civil and criminal liability
16 With reference to Ancient India, Ashtangasamgraha written by Vagbhata deals with
A. Astrology
B. Medicine
C. Law
D. Economics
Your Answer : B
Correct Answer : B
Answer Justification :
Learning: In the field of medicine, Vagbhata lived during this period. He was the last of the great
medical trio of ancient India.
The other two scholars Charaka and Susruta lived before the Gupta age. Vagbhata was the
author Ashtangasamgraha (Summary of the eight branches of medicine).
The Sushruta and Charaka texts differ in one major aspect, with Sushruta Samhita providing
the foundation of surgery, while Charaka Samhita being primarily a foundation of medicine.
Q Source: Page 109: TN 11th Standard History Textbook
TEST - 1 (MOCK TEST - 1)
www.insightsias.com 20
© Insights Active Learning| All rights reserved. You may not reproduce, distribute or exploit the contents in any form without written
permission by copyright owner. Copyright infringers may face civil and criminal liability
17 Which of these is/are mandatory qualification(s) to contest for the office of Vice-President of India?
1. She must be a citizen of India or a person of Indian Origin (PIO) residing in India since last ten
years.
2. She must have completed 35 years of age.
3. She must not hold an office for profit.
4. She must have served as a Member of Parliament (MP) or a member of Legislative Assembly in
India.
Select the correct answer using the codes below.
A. 2 and 3 only
B. 2, 3 and 4 only
C. 1 and 4 only
D. 3 only
Your Answer : A
Correct Answer : A
Answer Justification :
Justification: As per article 66, the candidate contesting for election of Vice-President of India
should be citizen of India, completed age of 35 years and must be qualified to become a member of
Rajya Sabha. He must not hold an office for profit.
Her main function is in the form of ex-officio Chairman of Rajya Sabha. She works as acting
president when president is not available.
Learning: Like Presidential election, Vice Presidential election is also indirect and is held via
system of proportional representation by means of the single transferable vote.
The electoral college of Vice President includes elected by elected members of Lok Sabha and
Rajya Sabha.
Unlike in Presidential election, MLAs or MLCs from state assemblies have no role to play in
Vice Presidential election.
We’ll explore this better when covering Laxmikanth.
Q Source: ECI has issued a notification for the process of filing nominations for the vicepresidential
election - Incumbent Vice President Hamid Ansari’s term comes to an end on August
10, 2017.
TEST - 1 (MOCK TEST - 1)
www.insightsias.com 21
© Insights Active Learning| All rights reserved. You may not reproduce, distribute or exploit the contents in any form without written
permission by copyright owner. Copyright infringers may face civil and criminal liability
18 Under the 2007 India-Bhutan Friendship Treaty
1. Neither Government will allow the use of its territory for activities harmful to the national security
and interest of the other
2. Both governments allow free trade and commerce between the territories of Bhutan and India.
3. Bhutanese subjects residing in Indian territories shall have equal justice with Indian subjects.
4. Government of Bhutan is free to export those arms, for commercial purposes, that it received via
imports through the territory of India.
Select the correct answer using the codes below.
A. 2 and 4 only
B. 1, 2 and 3 only
C. 1 and 3 only
D. 1, 2, 3 and 4
Your Answer : C
Correct Answer : B
Answer Justification :
Background: In June 2017, Chinese Army (People’s Liberation Army) construction party had
entered the Doklam area and attempted to construct a road. Royal Bhutan Army (RBA) patrol had
attempted to dissuade them from this unilateral activity.
Since these developments, India and Bhutan have been in continuous contact, giving a view of
India’s role in Bhutan’s security and matters of mutual interest.
The present standoff between Indian and Chinese troops in Doklam (located at the tri-junction
of India, China and Bhutan) has highlighted India’s very special relationship with Bhutan,
including military responsibilities.
Justification: The 2007 treaty is meant to respect the sensitivities of Bhutan regarding its
sovereignty.
Statement 1: This is to avoid Chinese intrusion from the soil of Bhutan.
Statement 2 and 3: You can read the treaty (hardly 2 pages) here
https://mea.gov.in/Images/pdf/india-bhutan-treaty-07.pdf
Statement 4: Government of Bhutan shall be free to import, from or through India into Bhutan,
whatever arms, ammunition etc as may be required for Bhutan, and that this arrangement shall
hold good for all time as long as the Government of India is satisfied that the intentions of the
Government of Bhutan are friendly and that there is no danger to India from such importations.
However, there shall be no export of such arms, ammunition and materials outside Bhutan either by
the Government of Bhutan or by private individuals. So, 4 is wrong.
Q Source: As mentioned in the background
TEST - 1 (MOCK TEST - 1)
www.insightsias.com 22
© Insights Active Learning| All rights reserved. You may not reproduce, distribute or exploit the contents in any form without written
permission by copyright owner. Copyright infringers may face civil and criminal liability
19 Sunderbans delta is facing potential threats due to
1. Global warming induced sea level rise leading to coastal erosion
2. Freshwater flows from Brahmaputra and Meghna
Which of the above is/are correct?
A. 1 only
B. 2 only
C. Both 1 and 2
D. None
Your Answer : A
Correct Answer : A
Answer Justification :
Justification: Sundarben delta hosts a large population of mangorves (that grow in salt water) - the
largest tidal halophytic mangrove forest in the world.
Statement 1: A study highlights a time series of the erosion of many mangrove forested islands of
the Indian Sunderbans from 1986 to 2012.
These islands include Sagar, Gosaba, Dulibhasani, Dalhousie, Bhangaduni and Jambudwip.
The mean sea level rise is considered a driving factor for coastal erosion, coastal flooding,
increase in the number of tidal creeks and losing land.
Statement 2: A critical minimal inflow of freshwater is necessary for the luxuriant growth of
mangroves.
Decrease in freshwater inflow results change in mangrove succession where freshwater loving
species of mangroves are replaced by salt-water loving ones. So, 2 is wrong.
The loss land, including mangrove forest is mainly due to decrease in fresh water flow and
sediment supply in the western (Indian) part of the delta, and the rate of sea level rise is
higher than sediment supply.
However, the eastern (Bangladesh) side of the Sunderbans delta is gaining land because of
the huge amount of sediment and water flow from the Brahmaputra and Meghna rivers
(which is good for the Mangroves).
This immediate impact of salinity will also have negative impact on the fishing community, as
commercially sought fish species will be replaced by fish that does not have as much market
value.
Q Source:
TEST - 1 (MOCK TEST - 1)
www.insightsias.com 23
© Insights Active Learning| All rights reserved. You may not reproduce, distribute or exploit the contents in any form without written
permission by copyright owner. Copyright infringers may face civil and criminal liability
http://www.thehindu.com/sci-tech/energy-and-environment/climate-change-impact-sunderbans-stea
dily-losing-its-famed-mangroves/article19195229.ece
TEST - 1 (MOCK TEST - 1)
www.insightsias.com 24
© Insights Active Learning| All rights reserved. You may not reproduce, distribute or exploit the contents in any form without written
permission by copyright owner. Copyright infringers may face civil and criminal liability
20 What is the usefulness of Double Asteroid Redirection Test (DART) being developed by NASA?
1. It will help protect earth from potential cosmic body impacts in the future.
2. It will lead to the establishment of the first human space station on an asteroid.
Which of the above is/are correct?
A. 1 only
B. 2 only
C. Both 1 and 2
D. None
Your Answer : D
Correct Answer : A
Answer Justification :
Justification: This is the first-ever mission that will deflect a near-Earth asteroid.
NASA will use kinetic impactor technique to deflect them.
This technique involves sending one or more large, high-speed spacecraft into the path of an
approaching near-earth object to shift its orbit to defend against future impact.
It works by changing the speed of a threatening asteroid by a small fraction of its total
velocity.
This is done well before the predicted impact so that this small velocity adds up over time to a
big shift of the asteroid’s path away from Earth
Q Source: https://www.nasa.gov/planetarydefense/aida/
TEST - 1 (MOCK TEST - 1)
www.insightsias.com 25
© Insights Active Learning| All rights reserved. You may not reproduce, distribute or exploit the contents in any form without written
permission by copyright owner. Copyright infringers may face civil and criminal liability
21 He was a follower of Jainism in the early part of his career and converted to Saivism later. He is
hailed as ‘Vichitrachitta’ who constructed a temple for Brahma, Vishnu and Siva without the use of
bricks, timber, metal and mortar. Who is the ruler?
A. Kadambas of Banavasi
B. Adikavi Pampa
C. Rajaraja I
D. Mahendravarman I
Your Answer :
Correct Answer : D
Answer Justification :
Learning: Mahendravarman I was converted to Saivism which led him to built a Siva temple at
Tiruvadi.
He was a great builder of cave temples. The Mandagappattu inscription hails him as
Vichitrachitta who constructed a temple for Brahma, Vishnu and Siva without the use of
bricks, timber, metal and mortar.
His rock-cut temples are found in a number of places like Tiruchirappalli.
His title Chitrakarapuli reveals his talents in painting. He is also regarded as an expert in
music.
The music inscription at Kudumianmalai is ascribed to him.
Q Source: Page 124: TN 11th Standard History Textbook
TEST - 1 (MOCK TEST - 1)
www.insightsias.com 26
© Insights Active Learning| All rights reserved. You may not reproduce, distribute or exploit the contents in any form without written
permission by copyright owner. Copyright infringers may face civil and criminal liability
22 Which of these justify the robust and benevolent system of administration that Cholas had?
1. Chola kings undertook royal tours to increase the efficiency of the administration.
2. All legal professions were exempt from tax liabilities.
3. All lands were carefully surveyed and classified for assessment of revenue.
4. During the hard times, there were remission of taxes.
Select the correct answer using the codes below.
A. 1 and 4 only
B. 1, 3 and 4 only
C. 2 and 3 only
D. 2 and 4 only
Your Answer : B
Correct Answer : B
Answer Justification :
Justification: Statement 1: There were regular drills with officials.
Statement 2 and 3: There were tolls and customs on goods taken from one place to another, various
kinds of professional taxes, dues levied on ceremonial occasions like marriages and judicial fines.
Moreover, the land revenue department was well organized. It was called as
puravuvarithinaikkalam.
The residential portion of the village was called ur nattam. These and other lands such as the lands
belonging to temples were exempted from tax.
Statement 4: During the hard times, there were remission of taxes and Kulottunga I became famous
by abolishing tolls and earned the title – Sungam Tavirtta Cholan.
The main items of government expenditure were the king and his court, army and navy, roads,
irrigation tanks and canals.
Q Source: Page 146: TN 11th Standard History Textbook
TEST - 1 (MOCK TEST - 1)
www.insightsias.com 27
© Insights Active Learning| All rights reserved. You may not reproduce, distribute or exploit the contents in any form without written
permission by copyright owner. Copyright infringers may face civil and criminal liability
23 What is the significance of Sabarmati Ashram in the modern history of India?
1. It served as the last residence of Mahatma Gandhi.
2. It has been declared as a national monument by the Indian government.
3. Salt Satyagraha was started from this Ashram in 1930.
Select the correct answer using the codes below.
A. 1 only
B. 2 and 3 only
C. 3 only
D. 1 and 3 only
Your Answer : B
Correct Answer : B
Answer Justification :
Justification: The Sabarmati Ashram (also known as Harijan or Satyagraha Ashram) was home to
Mohandas Gandhi from 1917 until 1930 and served as one of the main centres of the Indian
freedom struggle.
Statement 1: This was Sevagram cottage (not Sabarmati Ashram). Gandhiji left to Delhi from
Sevargam Cottage in 1946 and then went to Noakhali. From there he did not return to Sewagram
while engaged in his task of communal unity and peace making. He was martyred in Delhi in 1948.
Statement 2 and 3: It is common knowledge.
Learning: On his return from South Africa, Gandhi’s first Ashram in India was established in the
Kochrab area of Ahmedabad in 1915. The Ashram was then shifted in 1917 to a piece of open
barren land on the banks of the river Sabarmati.
Reasons for this shift included: he wanted to do some experiments in living eg farming, animal
husbandry, cow breeding, Khadi and related constructive activities, for which he was in
search of this kind of barren land.
The Prime Minister has recently released the coin and postal stamp in the honour of Shrimad
Rajchandraji on his 150th birth anniversary at Abhay Ghat. Shrimad Rajchandraji was the
guru to Mahatma Gandhi.
Q Source: PM recently inaugurated the centenary celebrations of Sabarmati Ashram in Ahmedabad
TEST - 1 (MOCK TEST - 1)
www.insightsias.com 28
© Insights Active Learning| All rights reserved. You may not reproduce, distribute or exploit the contents in any form without written
permission by copyright owner. Copyright infringers may face civil and criminal liability
24 Which of these systems were popularly introduced by Alauddin Khalji?
1. Branding of horses
2. Maintaining a descriptive list of soldiers
3. Paying army in kind rather than cash
4. Market based pricing for all products
Select the correct answer using the codes below.
A. 1 and 2 only
B. 3 and 4 only
C. 2 and 4 only
D. 1 and 3 only
Your Answer :
Correct Answer : A
Answer Justification :
Justification: Statement 1, 2 and 3: Alauddin Khalji maintained a large permanent standing army
and paid them in cash from the royal treasury.
He introduced the system of dagh (branding of horses) and prepared huliya (descriptive list of
soldiers).
In order to ensure maximum efficiency, a strict review of army from time to time was carried
out.
Statement 4: The introduction of paying salaries in cash to the soldiers led to price regulations
popularly called as Market Reforms.
Markets were under the control of a high officer called Shahna-i- Mandi. The supply of grain was
ensured by holding stocks in government store-houses.
Regulations were issued to fix the price of all commodities.
Learning: Every merchant was registered under the Market department. There were secret agents
called munhiyans who sent reports to the Sultan regarding the functioning of these markets.
Violation of regulations was severely punished. Harsh punishment was given if any shopkeeper
charged a higher price, or tried to cheat by using false weights and measures. Even during the
famine the same price was maintained
Q Source: Page 180: TN 11th Standard History Textbook
TEST - 1 (MOCK TEST - 1)
www.insightsias.com 29
© Insights Active Learning| All rights reserved. You may not reproduce, distribute or exploit the contents in any form without written
permission by copyright owner. Copyright infringers may face civil and criminal liability
25 Consider the following statements:
1. Under the Participatory Guarantee System for India (PGS-India), FAO bureaucracy certifies organic
products after due inspection.
2. National Programme for Organic Production (NPOP) intends to facilitate certification of organic
products in conformity with the importing countries organic standards.
Which of the above is/are correct?
A. 1 only
B. 2 only
C. Both 1 and 2
D. None
Your Answer :
Correct Answer : B
Answer Justification :
Background: At present, the consumers do not have any method to check the authenticity of the
organic food products due to the absence of a regulatory framework.
FSSAI has come out with draft regulations for organic food products to ensure the
authenticity of organic foods.
FSSAI’s regulation mandates that labelling of organic foods should provide full and accurate
information regarding the organic status of the organic foods.
Organic food products must carry a certification mark or a quality assurance mark provided
by the notified certification bodies.
Justification: Statement 1: Organic foods will have to comply with the government’s provisions
under the National Programme for Organic Production (NPOP) or the Participatory Guarantee
System for India (PGS-India) run by the Agriculture Ministry or any other standards notified by
FSSAI.
The standards of Organic Crop Production have been described in NPOP document.
The production standards include crop production plan, conversion requirement, nutrient
management, disease management etc for agricultural crops.
Details will be covered in I-Learning (new initiative on website).
Statement 2: The PGS is an internationally applicable organic quality assurance system [like ISO
9000] implemented and controlled by the committed organic farmer-producers through active
participation, along with the consumers, in the process based on verifiable trust.
TEST - 1 (MOCK TEST - 1)
www.insightsias.com 30
© Insights Active Learning| All rights reserved. You may not reproduce, distribute or exploit the contents in any form without written
permission by copyright owner. Copyright infringers may face civil and criminal liability
It is not an “inspection raj” certification system but, rather, one that is based on personal integrity
and peer pressure. Integrity is honesty when no one is looking over your shoulder to see what you
are doing. So, 1 is wrong.
http://pgsindia-ncof.gov.in/pgs_india.aspx
Q Source: http://pib.nic.in/newsite/PrintRelease.aspx?relid=155000
http://www.thehindubusinessline.com/news/fssai-issues-draft-regulations-for-organic-food-products/
article9737535.ece
TEST - 1 (MOCK TEST - 1)
www.insightsias.com 31
© Insights Active Learning| All rights reserved. You may not reproduce, distribute or exploit the contents in any form without written
permission by copyright owner. Copyright infringers may face civil and criminal liability
26 Consider the following statements. Assertion (A): Kushanas banned the use of gold coins taken from
their empire on the silk route. Reason (R): Kushanas could not control the Silk route and were at a
loss from its trade.
In the context of the above, which of these is correct?
A. A is correct, and R is an appropriate explanation of A.
B. A is correct, but R is not an appropriate explanation of A.
C. A is correct, but R is incorrect.
D. Both A and R are incorrect.
Your Answer : D
Correct Answer : D
Answer Justification :
Justification: The best-known of the rulers who controlled the Silk Route were the Kushanas, who
ruled over central Asia and north-west India around 2000 years ago.
Their two major centres of power were Peshawar and Mathura.
During their rule, a branch of the Silk Route extended from Central Asia down to the seaports at the
mouth of the river Indus, from where silk was shipped westwards to the Roman Empire.
The Kushanas were amongst the earliest rulers of the subcontinent to issue gold. So, both A and R
are wrong.
Q Source: Page 102: NCERT Class VI Our Pasts - I
TEST - 1 (MOCK TEST - 1)
www.insightsias.com 32
© Insights Active Learning| All rights reserved. You may not reproduce, distribute or exploit the contents in any form without written
permission by copyright owner. Copyright infringers may face civil and criminal liability
27 Consider the following about ‘Know India Programme (KIP)’:
1. KIP is a programme run jointly by IISC, IITs and NITs to inculcate scientific temper and general
knowledge in youth.
2. KIP involves compulsory government financed field visits in public schools to places of national
importance.
Which of the above is/are correct?
A. 1 only
B. 2 only
C. Both 1 and 2
D. None
Your Answer : B
Correct Answer : D
Answer Justification :
Justification: Know India Programme is an initiative of the Ministry of External Affairs for the
Persons of Indian Origin (PIO) (including NRIs) between the age group of 18 to 30 years to provide
them an exposure to the country of their origin (India).
The KIP offers a platform for the young PIOs to visit India share their views, expectations and
experiences and forge closer bonds with the India of present times in a three-week orientation
programme.
External Affairs Minister has recently launched a portal for the Know India Programme (KIP).
Q Source: http://www.mea.gov.in/know-india-programme.htm
TEST - 1 (MOCK TEST - 1)
www.insightsias.com 33
© Insights Active Learning| All rights reserved. You may not reproduce, distribute or exploit the contents in any form without written
permission by copyright owner. Copyright infringers may face civil and criminal liability
28 Which of the two key principles/themes of democracy are inseparable or can’t exist without each
other?
A. Equity and Justice
B. Fraternity and Sovereignty
C. Accountability and transparency
D. Both (a) and (c)
Your Answer : A
Correct Answer : D
Answer Justification :
Justification: Option A: Justice implies fairness, it is a cardinal value of democracy. Equity implies
impartiality. Take Article 14 of constitution that ensures rule of law: it ensures equality between
citizens against law, that leads to justice for all.
Option B: There is no direct correlation between both.
Option C: You can take RTI as an example. If the government isn’t transparent on information, it
can’t be held accountable. On the other hand, if there is no accountability, it will be most likely due
to opacity in functioning of government. Both go hand in hand.
Q Source: Improvisation: Page 40: NCERT Class VI Social and Political Life - I
TEST - 1 (MOCK TEST - 1)
www.insightsias.com 34
© Insights Active Learning| All rights reserved. You may not reproduce, distribute or exploit the contents in any form without written
permission by copyright owner. Copyright infringers may face civil and criminal liability
29 Ramananda, Kabir and Nanak
1. Were not linked with any particular religious creed
2. Did not believe in rituals and ceremonies
3. Condemned monotheism and believed in multiple Gods
4. Denounced all forms of idolatry
Select the correct answer using the codes below.
A. 1, 2 and 3 only
B. 1, 2 and 4 only
C. 3 and 4 only
D. 1 and 3 only
Your Answer : B
Correct Answer : B
Answer Justification :
Justification: Statement 1: In the fourteenth and fifteenth centuries, Ramananda, Kabir and Nanak
remained great apostles of the Bhakti cult.
They drew inspiration from old masters but showed a new path.
They helped the common people to shed age-old superstitions and attain salvation through
Bhakti or pure devotion.
Statement 2, 3 and 4: They condemned polytheism and believed in one god. They also denounced all
forms of idolatry. They strongly believed in Bhakti as the only means of salvation. They also
emphasised the fundamental unity of all religions.
Unlike the early reformers, they were not linked with any particular religious creed and did not
believe in rituals and ceremonies.
Q Source: Page 203: TN 11th Standard History Textbook
TEST - 1 (MOCK TEST - 1)
www.insightsias.com 35
© Insights Active Learning| All rights reserved. You may not reproduce, distribute or exploit the contents in any form without written
permission by copyright owner. Copyright infringers may face civil and criminal liability
30 Over the last decade, the country that is the World’s largest receiver of remittances is
A. USA
B. India
C. Russia
D. Canada
Your Answer : B
Correct Answer : B
Answer Justification :
Learning: According to UN International Fund for Agricultural Development (IFAD), Indians
working across the world had sent home USD 62.7 billion in 2016 making India the top remittancereceiving
country surpassing China.
The study is the first ever study conducted that analysed a 10-year trend in migration and
remittance flows over the period 2007-2016.
In the period between 2007 and 2016, India has surpassed China to become the world’s
largest receiver of remittances.
Asia remains the main remittance-receiving region with 55% of the global flows.
This increase in remittances is expected to lift millions out of poverty and in attaining the
Sustainable Development Goals (SDGs).
Top Remittance Receiving Countries: India (USD 62.7 billion), China (USD 61 billion), the
Philippines (USD 30 billion) and Pakistan (USD 20 billion).
Q Source: IFAD study as mentioned above
TEST - 1 (MOCK TEST - 1)
www.insightsias.com 36
© Insights Active Learning| All rights reserved. You may not reproduce, distribute or exploit the contents in any form without written
permission by copyright owner. Copyright infringers may face civil and criminal liability
31 Bhimbetka, in present-day Madhya Pradesh, is famous for
A. Hosting prehistoric cave paintings
B. Largest Buddha Bronze statues
C. Monolithic temples
D. None of the above
Your Answer : A
Correct Answer : A
Answer Justification :
Learning: Bhimbetka rock shelters are an archaeological site of the Paleolithic, exhibiting the
earliest traces of human life on the Indian Subcontinent, and thus the beginning of the Indian Stone
Age.
Some of the Stone Age rock paintings found among the Bhimbetka rock shelters are some 30,000
years old. The caves also deliver early evidence of dance. They were declared a World Heritage Site
in 2003.
Q Source: Page 15: NCERT Class VI Our Pasts - I
TEST - 1 (MOCK TEST - 1)
www.insightsias.com 37
© Insights Active Learning| All rights reserved. You may not reproduce, distribute or exploit the contents in any form without written
permission by copyright owner. Copyright infringers may face civil and criminal liability
32 Which of these types of trading/agreements in securities markets is NOT permitted in India?
A. Futures contract
B. Options contract
C. Forward contract
D. All are permitted
Your Answer : B
Correct Answer : D
Answer Justification :
Justification: Option A: Futures Contract has been permitted long back. It is a contract between
two parties where both parties agree to purchase or sell a particular commodity or any other
financial instrument at a predetermined price at a specified time in the future.
Option B: Commodity Exchanges have been requesting SEBI for a long time to permit options
trading in commodities be allowed. It was permitted recently.
Options Contract is a derivative product that offers an investor the right to purchase without
any obligation to buy at the specified price/date.
Suppose you want your dream car which as of now costs RM 100,000 (RM is a currency unit).
You don’t have the money to buy the car now, but you will have after 3 months.
A smart move would be to buy an “option” to purchase this car in the future (after 3 months).
You pay the car owner a contract premium (say RM 1,000) to exercise this option.
Remember, you have the option to buy or not buy the car.
See the diagram below now. Suppose after 3 months, car prices increase to RM 150,000. This
is a win case for you, since the options contract was to buy it at RM 100,000. You gain.
Suppose car price reduces. Now, you won’t buy the car, because the contract was not an
obligation (only a right) to buy the car. You would now pay the options premium (RM 1000 to
the owner), and may be buy the car from the market.
TEST - 1 (MOCK TEST - 1)
www.insightsias.com 38
© Insights Active Learning| All rights reserved. You may not reproduce, distribute or exploit the contents in any form without written
permission by copyright owner. Copyright infringers may face civil and criminal liability
Option C: You would heard of the Forward Markets Commission that earlier regulated
forward markets. Now SEBI has subsumed the organization.
Q Source:
http://www.livemint.com/Money/nfnDOuk11PlqWI43mgA1BL/Sebi-permits-options-trading-incommodity-
futures.html
TEST - 1 (MOCK TEST - 1)
www.insightsias.com 39
© Insights Active Learning| All rights reserved. You may not reproduce, distribute or exploit the contents in any form without written
permission by copyright owner. Copyright infringers may face civil and criminal liability
33 Which of these earliest cities was located to the east of Indus river?
1. Dholavira
2. Rakhi Garhi
3. Kalibangan
4. Sotkakoh
Select the correct answer using the codes below.
A. 1, 2 and 3 only
B. 2 and 4 only
C. 1 and 3 only
D. 1, 2, 3 and 4
Your Answer : B
Correct Answer : A
Answer Justification :
Justification:
Learning: These cities were found in the Punjab and Sind in Pakistan, and in Gujarat, Rajasthan,
Haryana and the Punjab in India.
Archaeologists have found a set of unique objects in almost all these cities: red pottery
painted with designs in black, stone weights, seals, special beads, copper tools, and long
stone blades.
TEST - 1 (MOCK TEST - 1)
www.insightsias.com 40
© Insights Active Learning| All rights reserved. You may not reproduce, distribute or exploit the contents in any form without written
permission by copyright owner. Copyright infringers may face civil and criminal liability
Cities such as Kalibangan and Lothal had fire altars, where sacrifices may have been
performed.
In some cities, like Mohenjodaro, a very special tank, which archaeologists call the Great
Bath, was found.
Q Source: Page 33: NCERT Class VI Our Pasts - I
TEST - 1 (MOCK TEST - 1)
www.insightsias.com 41
© Insights Active Learning| All rights reserved. You may not reproduce, distribute or exploit the contents in any form without written
permission by copyright owner. Copyright infringers may face civil and criminal liability
34 The Ministry of Law and Justice has launched the ‘Tele-Law’ initiative in collaboration with the
Ministry of Electronics and Information Technology. Consider the following with reference to it.
1. It would subsume National Legal Service Authority (NALSA) and its State Level authorities to
create a Virtual Legal Service Authority (VLSA).
2. It will provide legal aid services to the marginalized communities and citizens living in rural areas.
3. It would help people to seek legal advice from lawyers with the help of video conferencing facility.
Select the correct answer using the codes below.
A. 1 and 2 only
B. 2 and 3 only
C. 3 only
D. 1, 2 and 3
Your Answer :
Correct Answer : B
Answer Justification :
Justification: Statement 2: Under the initiative, a Para Legal Volunteer (PLV), would be employed
in each Common Service Centres (CSC – which are IT service points in blocks established with
government support and regulation).
The volunteer would serve as the first point of contact for the marginalized communities in
the rural areas who will help them understand the legal issues, explain the advice given by
lawyers and assist in further action as per the advice of the lawyer.
Around 1000 women PLVs will also be trained under this initiative to provide legal aid
services through the CSCs.
Statement 3: The National Legal Service Authority (NALSA) will also provide a panel of lawyers who
will provide advice from the state capitals to the applicants in the CSCs through video conferencing.
Learning: Apart from the lawyers, law school clinics, District Legal Service Authorities, voluntary
service providers and Non-Government Organisations working on legal aid and empowerment can
also be connected through the CSCs.
In the initial phase, the initiative will be tested as a pilot project across 500 Common Service
Centres (CSC) in Uttar Pradesh and Bihar.
The government will also put in place a robust monitoring and evaluation system to assess the
quality of the legal advice delivered to the people.
Q Source:
http://www.thehindu.com/news/national/india-launches-free-legal-aid-via-video-conferencing-to-villa
gers/article18970545.ece
TEST - 1 (MOCK TEST - 1)
www.insightsias.com 42
© Insights Active Learning| All rights reserved. You may not reproduce, distribute or exploit the contents in any form without written
permission by copyright owner. Copyright infringers may face civil and criminal liability
35 The Indian constitution prohibits discrimination between individuals on the basis of
A. Gender and caste only
B. Gender, caste, religion, race and place of birth
C. Gender, education, caste and place of birth
D. Caste, education, religion and race only
Your Answer : B
Correct Answer : B
Answer Justification :
Learning: These provisions come under the Right to Equality under fundamental rights.
The State shall not discriminate against any citizen on grounds only of religion, race, caste, sex,
place of birth or any of them.
No citizen shall, on grounds only of religion, race, caste, sex, place of birth or any of them, be
subject to any disability, liability, restriction or condition with regard to
access to shops, public restaurants, hotels and places of public entertainment; or
the use of wells, tanks, bathing ghats, roads and places of public resort maintained wholly or
partly out of State funds or dedicated to the use of the general public.
Nothing in this article shall prevent the State from making any special provision for women,
children, SCs or STs.
Q Source: Improvisation: Page 20: NCERT Class VI Social and Political Life - I
TEST - 1 (MOCK TEST - 1)
www.insightsias.com 43
© Insights Active Learning| All rights reserved. You may not reproduce, distribute or exploit the contents in any form without written
permission by copyright owner. Copyright infringers may face civil and criminal liability
36 With reference to Ancient Southern India, Tolkappiyam refers to the five-fold division of lands.
Consider the following matches with their description.
1. Kurinji: Desert
2. Mullai: Pastoral
3. Neydal: Agricultural
4. Marudam: Hilly tracks
5. Palai: Coastal
Select the correct matches using the codes below.
A. 1, 3 and 4 only
B. 2 only
C. 1, 4 and 5 only
D. 1, 2, 3, 4 and 5
Your Answer :
Correct Answer : B
Answer Justification :
Justification: This was the description: Kurinji (hilly tracks), Mullai (pastoral), Marudam
(agricultural), Neydal (coastal) and Palai (desert).
Learning: The people living in these five divisions had their respective chief occupations as well as
gods for worship.
Kurinji – chief deity was Murugan – chief occupation, hunting and honey collection.
Mullai – chief deity Mayon (Vishnu) – chief occupation, cattle-rearing and dealing with dairy
products.
Marudam – chief deity Indira – chief occupation, agriculture.
Neydal – chief deity Varunan – chief occupation fishing and salt manufacturing.
Palai – chief deity Korravai – chief occupation robbery.
Q Source: Page 90: TN 11th Standard History Textbook
TEST - 1 (MOCK TEST - 1)
www.insightsias.com 44
© Insights Active Learning| All rights reserved. You may not reproduce, distribute or exploit the contents in any form without written
permission by copyright owner. Copyright infringers may face civil and criminal liability
37 “Kanishka must have conquered the greater part of the Gangetic plain”, the evidence of this
statement comes from
1. Coins of Kanishka are found in places like Mathura, Sravasti and Benares.
2. His coins exhibit only Buddhist images, and not Hindu gods, a cult that was popular then in the
Gangetic plain.
Which of the above is/are correct?
A. 1 only
B. 2 only
C. Both 1 and 2
D. None
Your Answer : A
Correct Answer : A
Answer Justification :
Justification: Statement 1: At the time of his accession his empire included Afghanistan, Gandhara,
Sind and Punjab. Subsequently he conquered Magadha and extended his power as far as Pataliputra
and Bodh Gaya. This evidence comes from his coins that are found in the gangetic plains.
Statement 2: Kanishka embraced Buddhism in the early part of his reign. However, his coins exhibit
the images of not only Buddha but also Greek and Hindu gods. So, 2 is wrong.
Q Source: Page 79: TN 11th Standard History Textbook
TEST - 1 (MOCK TEST - 1)
www.insightsias.com 45
© Insights Active Learning| All rights reserved. You may not reproduce, distribute or exploit the contents in any form without written
permission by copyright owner. Copyright infringers may face civil and criminal liability
38 The Government has issued Public Procurement (Preference to Make in India), Order 2017 recently.
Consider the following with reference to its provisions:
1. The order applies to manufacturing sector only.
2. Ordinarily, all government procurement shall be based on 100% local content sourcing.
3. Import of non-essential commodities shall be severely regulated to promote domestic
manufacturing.
Select the correct answer using the codes below.
A. 1 only
B. 2 and 3 only
C. 3 only
D. None of the above
Your Answer : C
Correct Answer : D
Answer Justification :
Justification: This policy promotes manufacturing and production of goods and services in India
with a view to enhancing income and employment. Since services are also covered, S1 is wrong.
In pursuance of this Order, purchase preference shall be given to local suppliers in all
procurements undertaken by procuring entities in the manner specified in the order.
As per the order the minimum local content shall ordinarily be 50%. The Nodal Ministry may
prescribe a higher or lower percentage in respect of any particular item and may also
prescribe the manner of calculation of local content.
Q Source: http://dipp.nic.in/sites/default/files/publicProcurement_MakeinIndia_15June2017.pdf
TEST - 1 (MOCK TEST - 1)
www.insightsias.com 46
© Insights Active Learning| All rights reserved. You may not reproduce, distribute or exploit the contents in any form without written
permission by copyright owner. Copyright infringers may face civil and criminal liability
39 An important item of trade from Ancient India, it was highly valuable in Roman Empire so much that
it was known as ‘black gold’. What does the item refer to?
A. Crude oil
B. Coal
C. Dark Garnet stones
D. Pepper
Your Answer : A
Correct Answer : D
Answer Justification :
Learning: Frequently referred to as "black gold" in ancient India, on account of its demand and
trade value mainly due to its high trade value, the Zamorin ruler of Malabar coast had a flourishing
trade centre for export of pepper and other spices in Kerala.
The main mercantile traders were Arabs and people from Middle East.
The early Roman Empire got direct access to the Malabar Coast in India and its range of
exotic spices after their conquest of Egypt in 30 BC.
The prices of pepper were extremely high in the Middle Ages and the trade was completely
dominated by the Romans.
Q Source: Page 99: NCERT Class VI Our Pasts - I
TEST - 1 (MOCK TEST - 1)
www.insightsias.com 47
© Insights Active Learning| All rights reserved. You may not reproduce, distribute or exploit the contents in any form without written
permission by copyright owner. Copyright infringers may face civil and criminal liability
40 Consider the following:
1. Temple at Deogarh near Jhansi
2. Giant copper statue of Buddha originally found at Sultanganj
3. Delhi Iron pillar
4. Bagh caves paintings
The above belong to periods of which of these dynasties of India?
A. Peshwas
B. Guptas
C. Paramara
D. Western Ganga
Your Answer :
Correct Answer : B
Answer Justification :
Learning: The Delhi Iron pillar of the Gupta period is still free from rust though completely
exposed to sun and rain for so many centuries.
The paintings of the Gupta period are seen at Bagh caves near Gwalior.
Moreover, the mural paintings of Ajantha mostly illustrate the life of the Buddha as depicted in the
Jataka stories.
There was little influence of Gandhara style on Gupta art. But the beautiful statue of standing
Buddha at Mathura reveals a little Greek style. The Buddha statue unearthed at Saranath was
unique piece of Gupta art.
Q Source: Page 106-107: TN 11th Standard History Textbook
TEST - 1 (MOCK TEST - 1)
www.insightsias.com 48
© Insights Active Learning| All rights reserved. You may not reproduce, distribute or exploit the contents in any form without written
permission by copyright owner. Copyright infringers may face civil and criminal liability
41 Under the SMARTGRAM initiative of Rashtrapati (President) Bhavan, what is considered as a ‘smart
gram (village)’?
A. A village that is on the transition to become a municipal area
B. A village where a majority of the population is digitally literate
C. A village having required physical and social infrastructure governed with a layer of smart
information and communication systems
D. A village that is close to sub-urban areas of smart cities
Your Answer : C
Correct Answer : C
Answer Justification :
Learning: Under the project the selected villages will see new ventures in agriculture, skill
development, energy, education, heath, employment generation, and entrepreneurship.
The model is based on the convergence of resources and effort by the central government, state
government, district administration, PRIs, private sector etc. to enhance the environment,
connectivity and wellbeing of each village.
Q Source: http://pib.nic.in/newsite/PrintRelease.aspx?relid=161641
TEST - 1 (MOCK TEST - 1)
www.insightsias.com 49
© Insights Active Learning| All rights reserved. You may not reproduce, distribute or exploit the contents in any form without written
permission by copyright owner. Copyright infringers may face civil and criminal liability
42 Consider the following statistics.
1. India ranks first among the world’s milk producing Nations.
2. Most of the milk produced in India comes from cooperatives.
Which of the above is/are correct?
A. 1 only
B. 2 only
C. Both 1 and 2
D. None
Your Answer : A
Correct Answer : A
Answer Justification :
Justification: Statement 1: India continued to be the largest milk producing nation (since 1998)
with milk production of nearly 155 million tonnes during 2015-16. India also has the largest bovine
population in the World.
About 54% of the milk produced in the country is surplus for marketing in the domestic market, of
which only about 20% is procured/processed by the organised sector being equally, shared by
Cooperatives and Private dairy organizations.
Statement 2: The dairy cooperatives collectively procure around 16 million tonnes of milk.
Learning: Government is mandated to double milk producers’ income at farm level by providing
rural milk producers with greater access to the organised milk processing sector.
Department of Animal Husbandry, Dairying & Fisheries has formulated a Draft National Action Plan
for Dairy Development recently.
Q Source: PIB SS (Release ID :167289)
TEST - 1 (MOCK TEST - 1)
www.insightsias.com 50
© Insights Active Learning| All rights reserved. You may not reproduce, distribute or exploit the contents in any form without written
permission by copyright owner. Copyright infringers may face civil and criminal liability
43 With reference to the Indus Valley civilization, consider the following statements:
1. Indus valley civilization was the biggest exporter of semi-precious stones and a major importer of
agricultural products.
2. Fishing was a regular occupation while hunting and bull fighting were banned.
3. Linga worship was prevalent and even natural figures like trees were worshipped.
4. Lothal was an important point of trade between the Harappan civilization and the remaining part of
India as well as Mesopotamia.
Select the correct answer using the codes below.
A. 1 and 2 only
B. 3 and 4 only
C. 1, 2 and 3 only
D. 2, 3 and 4 only
Your Answer : D
Correct Answer : B
Answer Justification :
Justification: Statement 1: Gold, copper, tin and several semi-precious stones were imported. Main
exports were several agricultural products such as wheat, barely, peas, oil seeds etc.
Statement 2: Hunting and bull fighting were common pastimes, not banned. Marbles, balls and dice
were also used for games.
Statement 3: The chief female deity was the Mother Goddess represented in terracotta figurines. In
latter times, Linga worship was prevalent. Trees and animals were also worshipped by the
Harappans. They also believed in ghosts and evil forces and used amulets as protection against
them.
Statement 4: Lothal is an important Harappan centre, you should read more about it.
Q Source: Page 21-23: TN 11th Standard History Textbook
TEST - 1 (MOCK TEST - 1)
www.insightsias.com 51
© Insights Active Learning| All rights reserved. You may not reproduce, distribute or exploit the contents in any form without written
permission by copyright owner. Copyright infringers may face civil and criminal liability
44 Many Chinese Buddhist pilgrims visited Indian subcontinent to visit places associated with the life of
the Buddha. Who among the following is NOT one of them?
A. Fa Xian
B. Xuan Zang
C. I-Qing
D. Alan Chen
Your Answer :
Correct Answer : D
Answer Justification :
Learning: Three such Chinese Buddhist pilgrims are best-known.
Fa Xian, who came to the subcontinent about 1600 years ago, Xuan Zang (who came around 1400
years ago) and I-Qing, who came about 50 years after Xuan Zang are the famous one.
They have documented previous socio-economic, political and religious details of Indian life then.
We will cover all travellers in detail in coming tests.
Q Source: Page 105: NCERT Class VI Our Pasts - I
TEST - 1 (MOCK TEST - 1)
www.insightsias.com 52
© Insights Active Learning| All rights reserved. You may not reproduce, distribute or exploit the contents in any form without written
permission by copyright owner. Copyright infringers may face civil and criminal liability
45 Consider the following statements.
1. Anyone who is a resident of the concerned village is a member of the Gram Sabha.
2. All plans for the work of the Gram Panchayat are placed before the people in Gram Sabha.
Which of the above is/are correct?
A. 1 only
B. 2 only
C. Both 1 and 2
D. None
Your Answer : B
Correct Answer : B
Answer Justification :
Justification: Statement 1: Only one who is 18 years old or more and who has the right to vote is a
member of the Gram Sabha.
The Gram Sabha is a meeting of all adults who live in the area covered by a Panchayat. This could
be only one village or a few villages.
Statement 2: The Gram Sabha prevents the Panchayat from doing wrong things like misusing
money or favouring certain people.
It plays an important role in keeping an eye on the elected representatives and in making them
responsible to the persons who elected them.
Q Source: Improvisation: Page 43-45: NCERT Class VI Social and Political Life - I
TEST - 1 (MOCK TEST - 1)
www.insightsias.com 53
© Insights Active Learning| All rights reserved. You may not reproduce, distribute or exploit the contents in any form without written
permission by copyright owner. Copyright infringers may face civil and criminal liability
46 With reference to the Vedic civilization, Rajasuya, Asvamedha and Vajpeya were
A. Rituals and sacrifices performed by the King to strengthen his position
B. Mercenary army contingents that were owned by the local landlords
C. Land tax imposed on areas inside the jurisdiction of the Empire
D. Body of officials who maintained a record of the extent of empire, its defeats and successions
Your Answer : A
Correct Answer : A
Answer Justification :
Context: Larger kingdoms were formed during the later Vedic period as many jana or tribes were
amalgamated to form janapadas or rashtras in the later Vedic period.
Hence the royal power had increased along with the increase in the size of kingdom.
Learning: So, the king performed various rituals and sacrifices to strengthen his position. They
include Rajasuya (consecration ceremony), Asvamedha (horse sacrifice) and Vajpeya (chariot race).
The kings also assumed titles like Rajavisvajanan, Ahilabhuvanapathi, (lord of all earth), Ekrat and
Samrat (sole ruler) to showcase his power.
Q Source: Page 32: TN 11th Standard History Textbook
TEST - 1 (MOCK TEST - 1)
www.insightsias.com 54
© Insights Active Learning| All rights reserved. You may not reproduce, distribute or exploit the contents in any form without written
permission by copyright owner. Copyright infringers may face civil and criminal liability
47 Jainism advocates three principles known as Triratnas (three gems) which are right faith, right
knowledge and right conduct. On the other hand, Buddhism preaches four noble truths and the
eightfold path. What is/are common between the teachings of both religions?
1. Acceptance of the theory that God exists and it has multiple aspects
2. Understanding that no object possesses a soul and is inanimate in the larger flow of life
3. To practice ahimsa and abstain from acquiring precious metals like Gold and Silver
4. Penance and extreme austerity to punish the body
Select the correct answer using the codes below.
A. 1 and 2 only
B. 3 only
C. 2 and 3 only
D. 1 and 4 only
Your Answer : B
Correct Answer : B
Answer Justification :
Justification: We will explain it in the light of teachings of Mahavira.
Justification: Statement 1 and 2: Right Knowledge (as per Mahavira) is the acceptance of the
theory that there is no God and that the world has been existing without a creator and that all
objects possess a soul.
Buddhism sheds any discussion on God and soul as futile. Buddha also taught that the soul does not
exist, so there is no meeting ground in this case. So, 1 and 2 are wrong.
Statement 3: Right conduct (Jainism) refers to the observance of the five great vows such as not to
injure life, not to lie, not to steal, not to acquire property and not to lead an immoral life.
Buddhism also forbids its monks from acquiring property (that may bring a sense of ego and pride
to its monks and nuns) and injuring life. So, 3 is a meeting ground.
Statement 4: Only Jainism advocates extreme penance, Buddhism preaches moderation. So, 4 is
wrong.
Q Source: Page 39: TN 11th Standard History Textbook
TEST - 1 (MOCK TEST - 1)
www.insightsias.com 55
© Insights Active Learning| All rights reserved. You may not reproduce, distribute or exploit the contents in any form without written
permission by copyright owner. Copyright infringers may face civil and criminal liability
48 Which of these is NOT one of the major styles or schools of ancient Indian art?
A. Amravati
B. Mathura
C. Gandhara
D. Sarnath
Your Answer : D
Correct Answer : D
Answer Justification :
Learning: The Amravati school of art had great influence on art in Sri Lanka and South-East Asia
as products from here were carried to those countries. It also had influence over South Indian
sculpture.
Mathura: During the first century AD, Gandhara and Mathura School of Art flourished mainly
during reign of Kushana emperor Kanishka.
Mathura School had developed indigenously.
Apart from Mathura, other important centres of this art were Sarnath and Kosambi.
The material used in this school was the spotted red sandstone. This art reached its peak
during the Gupta period in 6th or 7th century.
Gandhara will be covered later.
Q Source: Improvisation: Page 66: NCERT Class VI Our Pasts - I
TEST - 1 (MOCK TEST - 1)
www.insightsias.com 56
© Insights Active Learning| All rights reserved. You may not reproduce, distribute or exploit the contents in any form without written
permission by copyright owner. Copyright infringers may face civil and criminal liability
49 With reference to ANUGA 2017, consider the following:
1. It is an inter-governmental platform organized by UNSC for promoting defence technology
cooperation.
2. India will be a co-Partner Country in 2017 edition of the event.
3. Agreements signed under the event are mandated to take the place of national laws and impose
regulatory burdens on party nations.
Select the correct answer using the codes below.
A. 1 only
B. 2 and 3 only
C. 2 only
D. 3 only
Your Answer :
Correct Answer : C
Answer Justification :
Justification: Statement 1: ANUGA is the world’s biggest and most important trade fair for Food
and beverage trade held in Germany.
It takes place biennially (every 2 years). ANUGA 2017 is the 34th edition.
ANUGA offers an extensive supporting programme with Lectures, Special exhibition and
attractive industry events
More than fifty per cent of the industry visitors are foreigners.
Statement 2: India is to become a co-Partner Country in ANUGA 2017. It gives India an opportunity
to showcase its strengths and opportunities to an international audience in the food sector and
invite investments.
Statement 3: There is no such provision.
Q Source: http://pib.nic.in/newsite/PrintRelease.aspx?relid=165597
TEST - 1 (MOCK TEST - 1)
www.insightsias.com 57
© Insights Active Learning| All rights reserved. You may not reproduce, distribute or exploit the contents in any form without written
permission by copyright owner. Copyright infringers may face civil and criminal liability
50 Indian Railways Organization for Alternate Fuel (IROAF) has been recently awarded the coveted
National level “Golden Peacock Award for the Year 2017 for Eco-Innovation”. The award was given
for
1. Substitution of Diesel in passenger trains by environment friendly Compressed Natural Gas (CNG).
2. Installation of open source licensing system for electricity generation and procurement
Which of the above is/are correct?
A. 1 only
B. 2 only
C. Both 1 and 2
D. None
Your Answer :
Correct Answer : A
Answer Justification :
Justification: Indian Railways has achieved a major landmark in the field of Eco friendly fuel
technologies by this innovation.
Use of CNG in Train Sets for passenger transportation has been done for the first time in the world.
This innovation (20% substitution of diesel by CNG) will reduce emissions considerably by
bringing down NOx, CO2 and Particulate Matter besides achieving economy in fuel cost by
eight per cent.
It could save up to over thirteen hundred crores annually if implemented over entire fleet of
Diesel Locomotives of Indian Railways.
An improved technology of 40% substitution is at present being developed by IROAF. This will
enhance the potential of saving of fuel cost of IR to about Rs. thirty four hundred crore per annum.
Q Source: PIB Releases: 11th July
TEST - 1 (MOCK TEST - 1)
www.insightsias.com 58
© Insights Active Learning| All rights reserved. You may not reproduce, distribute or exploit the contents in any form without written
permission by copyright owner. Copyright infringers may face civil and criminal liability
51 The Mauryan state had a well-organized civil service, this is evident from
1. Civil servants called Amatyas who looked after the day-to-day administration were selected based
on merit.
2. Adyakshas officers helped control the retail and wholesale prices of goods and ensured their steady
supply in the state.
3. All officials were interviewed and appointed directly by the emperor.
Select the correct answer using the codes below.
A. 1 and 2 only
B. 2 and 3 only
C. 3 only
D. 2 only
Your Answer : A
Correct Answer : A
Answer Justification :
Justification: A council of ministers called Mantriparishad assisted the king in administrative
matters.
It consisted of Purohita, Mahamantri, Senapati and Yuvaraja.
Statement 1: Amatyas were similar to the IAS officers of independent India. The method of selection
of Amatyas was elaborately given by Kautilya.
Statement 2: Department of Commerce and Industry had controlled the retail and wholesale prices
of goods and tried to ensure their steady supply through its officers called Adyakshas. It also
controlled weights and measures, levied custom duties and regulated foreign trade.
Statement 3: Selection process was different for different officials.
Learning: Asoka also appointed Dhamma Mahamatras to supervise the spread of Dhamma.
Even the military service was well organized. The Mauryan army was well organized and it was
under the control of Senapati. The salaries were paid in cash. Kautilya refers to the salaries of
different ranks of military officers.
Q Source: Page 66-67: TN 11th Standard History Textbook
TEST - 1 (MOCK TEST - 1)
www.insightsias.com 59
© Insights Active Learning| All rights reserved. You may not reproduce, distribute or exploit the contents in any form without written
permission by copyright owner. Copyright infringers may face civil and criminal liability
52 Ptolemy, a Greek writer, mentions many important ports in the Deccan. Among them, the greatest
port of the Satavahanas in Western Deccan was
A. Kalyani
B. Ganjam
C. Satakarni
D. Pulamayi
Your Answer : C
Correct Answer : A
Answer Justification :
Learning: Vashishtaputra Pulamayi extended the Satavahana power up to the mouth of the Krishna
river.
He issued coins on which the image of ships was inscribed. They reveal the naval power and
maritime trade of the Satavahanas.
Their greatest port was Kalyani on the west Deccan. Gandakasela and Ganjam on the east coast
were the other important seaports.
The last great ruler of Satavahanas was Yajna Sri Satakarni.
Q Source: Page 76: TN 11th Standard History Textbook
TEST - 1 (MOCK TEST - 1)
www.insightsias.com 60
© Insights Active Learning| All rights reserved. You may not reproduce, distribute or exploit the contents in any form without written
permission by copyright owner. Copyright infringers may face civil and criminal liability
53 The National Mission on Cultural Mapping of India intends to
1. Create repository of information about cultural assets
2. Launch massive cultural awareness campaigns
3. Open a direct channel of communication of artists with the Government
4. Hold talent hunt competitions from Block level to National level
Select the correct answer using the codes below.
A. 1 and 4 only
B. 2 and 3 only
C. 1, 2 and 3 only
D. 1, 2, 3 and 4
Your Answer : C
Correct Answer : D
Answer Justification :
Learning: The mission aims at converting the vast and widespread cultural canvas of India into an
objective cultural map to preserve our cultural heritage.
The Mission also seeks to open a direct channel of communication of artists with the
Government and peer to peer communication among artists for talent honing and handholding
of each other.
With a vision that no cultural talent or heritage should remain unattended (so that it could be
nurtured), the Ministry of Culture has planned to hold talent hunt competitions with various
rounds from Block level to National level.
The Artist/Organisations also may register online at www.culturalmapping.nic.in.
The Cultural Mapping of India portal will serve as a repository/consolidated database of
information about cultural assets and resources i.e. a cultural inventory at one place for
essential planning, sustainable economic development and for preserving the scattered and
near extinct art forms.
Q Source: http://pib.nic.in/newsite/PrintRelease.aspx?relid=165655
TEST - 1 (MOCK TEST - 1)
www.insightsias.com 61
© Insights Active Learning| All rights reserved. You may not reproduce, distribute or exploit the contents in any form without written
permission by copyright owner. Copyright infringers may face civil and criminal liability
54 This council held near Srinagar prepared an authoritative commentary on the Buddhist Tripitakas
and gave the Mahayana doctrine a final shape. The council was convened by
A. Kanishka
B. Samprati
C. Bindusara Amitraghata
D. Devavarman
Your Answer :
Correct Answer : A
Answer Justification :
Learning: He convened this Fourth Buddhist Council that discussed matters relating to Buddhist
theology and doctrine.
It was held at the Kundalavana monastery near Srinagar in Kashmir under the presidentship of
Vasumitra. About 500 monks attended the Council.
Learning: Asvagosha was a great philosopher, poet and dramatist and was particularly close to
Kanishka. He became his religious advisor in his later years and composed Buddhacharita.
Nagarjuna from south India also adorned the court of Kanishka. The famous physician of ancient
India Charaka was also patronized by him.
Q Source: Page: TN 11th Standard History Textbook
TEST - 1 (MOCK TEST - 1)
www.insightsias.com 62
© Insights Active Learning| All rights reserved. You may not reproduce, distribute or exploit the contents in any form without written
permission by copyright owner. Copyright infringers may face civil and criminal liability
55 Consider the following about The Energy Resources Institute (TERI).
1. It is a non-profit policy research organisation.
2. It is an attached agency of the International Centre for Climate Governance (ICCG).
Which of the above is/are correct?
A. 1 only
B. 2 only
C. Both 1 and 2
D. None
Your Answer : C
Correct Answer : A
Answer Justification :
Justification: Statement 1: TERI conducts research work in the fields of energy, environment and
sustainable development.
Statement 2: The Energy Resources Institute (TERI) was ranked second among world’s best climate
think tank by the International Centre for Climate Governance (ICCG).
ICCG is an internationally renowned centre founded in 2009.
The main focus of ICCG activities is the design of climate policy and related governance issues.
Q Source: www.insightsonindia.com/2017/07/06/insights-daily-current-affairs-06-july-2017/
TEST - 1 (MOCK TEST - 1)
www.insightsias.com 63
© Insights Active Learning| All rights reserved. You may not reproduce, distribute or exploit the contents in any form without written
permission by copyright owner. Copyright infringers may face civil and criminal liability
56 Which of these ancient scripts generally writes in left to right direction?
A. Brahmi
B. Kharosthi
C. Harappan
D. All of the above
Your Answer : B
Correct Answer : A
Answer Justification :
Learning: Brahmi: is one of the oldest writing systems used in South and Central Asia from the 1st
millennium BCE.
Most modern Indian scripts have developed from the Brahmi script over hundreds of years.
Harappan: Iravatham Mahadevan established that the Harappan script is from right to left.
Kharosthi: is an ancient script used in ancient Gandhara (primarily modern-day Afghanistan and
Pakistan) to write the Gandhari Prakrit and Sanskrit.
Q Source: Improvisation: Page 81: NCERT Class VI Our Pasts - I
TEST - 1 (MOCK TEST - 1)
www.insightsias.com 64
© Insights Active Learning| All rights reserved. You may not reproduce, distribute or exploit the contents in any form without written
permission by copyright owner. Copyright infringers may face civil and criminal liability
57 Consider the following statements.
1. Innovate in India (i3) program offers young entrepreneurs an avenue to engage with the best
practices in the domestic financial and banking industry.
2. National Biopharma Mission will concentrate, inter alia, on development of medical devices and
biotherapeutics to upgrade India’s product development capabilities to globally competitive levels.
Which of the above is/are correct?
A. 1 only
B. 2 only
C. Both 1 and 2
D. None
Your Answer : B
Correct Answer : B
Answer Justification :
Justification: Statement 1: i3 is a flagship program of the Government of India in collaboration
with World Bank.
It is committed to make India a hub for design and development of novel, affordable and
effective biopharmaceutical products and solutions by engaging with entrepreneurs.
It will strive to increase India’s global biopharmaceutical market share from current 2.8% to
5% and generate additional business opportunity of $16 billion.
Statement 2 (S2): It will bring together expertise from national and international corridors to
provide strategic direction and guidance to move promising solutions through the product
development value chain.
The mission will be implemented by Biotechnology Industry Research Assistance Council (BIRAC), a
PSU of Department of Biotechnology.
Besides, it will also work on establishment of shared infrastructure and facilities and at the same
time creating and enhancing technology transfer capabilities in public and private sector.
Q Source: http://pib.nic.in/newsite/PrintRelease.aspx?relid=166951
TEST - 1 (MOCK TEST - 1)
www.insightsias.com 65
© Insights Active Learning| All rights reserved. You may not reproduce, distribute or exploit the contents in any form without written
permission by copyright owner. Copyright infringers may face civil and criminal liability
58 In a democratic state, government derives its ultimate sovereignty from
A. Citizens of the state
B. Territory of the State
C. Laws governing the state
D. Parliament
Your Answer : D
Correct Answer : A
Answer Justification :
Justification: The chain is straightforward.
People elect representatives, representatives form Parliament, Parliament chooses the government,
government makes laws, and those laws regulate the territory of the state.
So, the root of sovereignty can be easily traced to people.
Q Source: Improvisation: Page 31: NCERT Class VI Social and Political Life - I
TEST - 1 (MOCK TEST - 1)
www.insightsias.com 66
© Insights Active Learning| All rights reserved. You may not reproduce, distribute or exploit the contents in any form without written
permission by copyright owner. Copyright infringers may face civil and criminal liability
59 Chandragupta II, a ruler of the Gupta Empire, assumed the famous title of ‘Sakari’. What did the title
imply?
A. Destroyer of Sakas
B. Lord of the world
C. Well Wisher of masses
D. Devotee of Shankara
Your Answer :
Correct Answer : A
Answer Justification :
Learning: The greatest of the military achievements of Chandragupta II was his war against the
Saka satraps of western India.
Rudrasimha III, the last ruler of the Saka satrap was defeated, dethroned and killed.
His territories in western Malwa and the Kathiawar Peninsula were annexed into the Gupta
Empire.
After this victory he performed the horse sacrifice and assumed the title Sakari, meaning,
‘destroyer of Sakas’. He also called himself Vikramaditya.
Q Source: Page 101: TN 11th Standard History Textbook
TEST - 1 (MOCK TEST - 1)
www.insightsias.com 67
© Insights Active Learning| All rights reserved. You may not reproduce, distribute or exploit the contents in any form without written
permission by copyright owner. Copyright infringers may face civil and criminal liability
60 Consider the following statements about the recently established Global Foreign Exchange
Committee (GFXC).
1. It was established as a research arm of International Monetary Fund (IMF).
2. GFXC will enforce a FX Global Code that will legally regulate the manipulation of floating foreign
exchange by Central Banks.
3. It will promote collaboration and communication among the local foreign exchange committees
(FXCs) of member nations.
4. It will replace the institution of Bank of International Settlements (BIS).
5. Indian government has boycotted GFXC due to its non-democratic governance structure.
Select the correct answer using the codes below.
A. 2, 3 and 4 only
B. 1, 2 and 5 only
C. 3 only
D. 1, 2, 3 and 4 only
Your Answer : C
Correct Answer : C
Answer Justification :
Justification: This is a very important development, hence the long explanation.
Statement 1: GFXC was established in 2017 as a forum bringing together central banks and private
sector participants with the aim to promote a robust, liquid, open, and appropriately transparent FX
market.
Statement 2: FX Global Code is a set of global principles of good practice in the foreign exchange
market, developed to provide a common set of guidelines to promote the integrity and effective
functioning of the wholesale foreign exchange market.
The Global Code does not impose legal or regulatory obligations on Market Participants, nor
does it substitute for regulation.
It is rather intended to serve as a supplement to any and all local laws, rules and regulations
by identifying global good practices and processes.
Statement 3: The objectives of the GFXC are:
to promote collaboration and communication among the local foreign exchange committees
(FXCs) and non-GFXC jurisdictions with significant FX markets;
to exchange views on trends and developments in global FX markets, including on the
structure and functioning of those markets, drawing on information gathered at the various
FXCs; and
TEST - 1 (MOCK TEST - 1)
www.insightsias.com 68
© Insights Active Learning| All rights reserved. You may not reproduce, distribute or exploit the contents in any form without written
permission by copyright owner. Copyright infringers may face civil and criminal liability
to promote, maintain and update on a regular basis the FX Global Code (the Code) and to
consider good practices regarding effective mechanisms to support adherence.
Statement 4: The GFXC committee has been set up under guidance of the Bank for International
Settlements (BIS), an international financial organisation.
Statement 5: India will soon join Global Foreign Exchange Committee (GFXC).
Q Source:
http://www.thehindubusinessline.com/markets/forex/india-to-join-new-global-foreign-exchange-com
mittee/article9756342.ece
TEST - 1 (MOCK TEST - 1)
www.insightsias.com 69
© Insights Active Learning| All rights reserved. You may not reproduce, distribute or exploit the contents in any form without written
permission by copyright owner. Copyright infringers may face civil and criminal liability
61 Why the sale of Oxytocin, despite being a natural hormone, is heavily regulated?
1. It was used as a depressant by a large segment of population.
2. Its indiscriminate use by dairy farmers was causing irreversible hormone damage in milch animals.
3. Consumption of food laced with Oxytocin can cause cardiac problems in humans.
4. It is abused to speed up child birth in women.
Select the correct answer using the codes below.
A. 1 and 3 only
B. 2, 3 and 4 only
C. 2 and 4 only
D. 1, 2, 3 and 4
Your Answer : B
Correct Answer : B
Answer Justification :
Justification: Statement 1: Oxytocin has also been dubbed the love or the bliss hormone due to its
effects on behaviour. It isn’t a depressant. So, 1 is wrong.
Statement 2: The drug Oxytocin is banned in India under Prevention of Cruelty to Animals Act and
Food and Drug Adulteration Prevention Act, 1960.
In mammals, Oxytocin stimulates ejection of milk from the mammary glands.
But, its use can cause reproductive damage in animals and even their death.
Statement 3: Overdose of the drug in humans can lead to haemorrhage, palpitation, low blood
pressure and in some cases even death.
Statement 4: The release of oxytocin by the pituitary gland acts to regulate two female reproductive
functions: Childbirth and Breast-feeding. So, it is used to speed up child birth which is wrong and
should be shunned.
So, after being banned in retail markets, the Indian Drug Controller will soon restrict
manufacturing of controversial hormone drug Oxytocin to public sector undertakings (PSUs).
Q Source:
http://www.thehindu.com/news/national/only-psus-may-be-let-to-make-oxytocin/article18868286.ece
TEST - 1 (MOCK TEST - 1)
www.insightsias.com 70
© Insights Active Learning| All rights reserved. You may not reproduce, distribute or exploit the contents in any form without written
permission by copyright owner. Copyright infringers may face civil and criminal liability
62 Consider the following statements.
1. Manimegalai is a Tamil epic that criticizes Buddhism while investigating the strengths of
contemporary Hindu traditions.
2. Silappatikaram, produced at Sangam, is a poetic rendition with details of Tamil culture.
Which of the above is/are correct?
A. 1 only
B. 2 only
C. Both 1 and 2
D. None
Your Answer :
Correct Answer : B
Answer Justification :
Justification: The corpus of Sangam literature includes Tolkappiyam, Ettutogai, Pattuppattu,
Pathinenkilkanakku, and the two epic – Silappathigaram and Manimegalai.
Statement 1: The aim of the author of Manimegalai, Seethalai Saathanar, was to compare Buddhism
favourably with the other prevailing religions in South India in order to propagate Buddhism.
He criticizes Hinduism, and exposes the weaknesses of the other contemporary Indian
religions, while praising the Buddha's Teaching, the Dharma, as the most perfect religion. So,
1 is wrong.
Statement 2: Regarded as one of the great works of Tamil literature, the Silappatikaram is a poetic
rendition with details of Tamil culture; its varied religions; its town plans and city types; the
mingling of different people; and the arts of dance and music.
Q Source: Improvisation: Page 85-86: TN 11th Standard History Textbook
TEST - 1 (MOCK TEST - 1)
www.insightsias.com 71
© Insights Active Learning| All rights reserved. You may not reproduce, distribute or exploit the contents in any form without written
permission by copyright owner. Copyright infringers may face civil and criminal liability
63 Consider the following statements.
1. Harsha prohibited the use of animal food in his kingdom and punished those who killed any living
being.
2. Harsha gave away his enormous wealth as gifts to the members of all religious sects during the
Allahabad Conference.
Which of the above is/are correct?
A. 1 only
B. 2 only
C. Both 1 and 2
D. None
Your Answer : C
Correct Answer : C
Answer Justification :
Justification: Statement 1: In his early life, Harsha was a devout Saiva but later he became an
ardent Hinayana Buddhist. Hiuen Tsang converted him to Mahayana Buddhism. This led him to
shed eating meat and enforce the same as well.
Statement 2: Hiuen Tsang mentions in his account about the conference held at Allahabad, known
as Prayag.
It was the one among the conferences routinely convened by Harsha once in five years. Harsha gave
away his enormous wealth as gifts to the members of all religious sects.
According to Hiuen Tsang, Harsha was so lavish that he emptied the treasury and even gave away
the clothes and jewels he was wearing. His statement might be one of admiring exaggeration.
Q Source: Page 116: TN 11th Standard History Textbook
TEST - 1 (MOCK TEST - 1)
www.insightsias.com 72
© Insights Active Learning| All rights reserved. You may not reproduce, distribute or exploit the contents in any form without written
permission by copyright owner. Copyright infringers may face civil and criminal liability
64 With reference to the rights and responsibilities of the Attorney General of India, consider the
following:
1. He has right of audience in all courts within the territory of India.
2. He has the right to speak and take part in the proceeding of a joint sitting of the Parliament.
3. He is entitled to vote on crucial national resolutions and motions in Rajya Sabha, excluding
legislations.
4. He can be made a member of any parliamentary committee with a right to discuss and vote.
5. He is not allowed to take up private practice as long as he is an incumbent.
Select the correct answer using the codes below.
A. 1 and 2 only
B. 1, 3 and 4 only
C. 1, 4 and 5 only
D. 2, 3 and 5 only
Your Answer : C
Correct Answer : A
Answer Justification :
Justification: He is the first law officer of the government of India and acts as top advocate for
Union Government. He is responsible for giving advice to President/ Union Government upon legal
matters.
Statement 2 and 3: He has the right to speak and take part in proceedings of both the houses of
parliament including joint sittings. But cannot vote in parliament.
Statement 4: He can also be made a member of any parliamentary committee but in the committee
also, he has no power to vote. He has all the powers and privileges that of a member of parliament.
Statement 5: Attorney General of India is not a full time Government servant. He is an advocate of
the government and is allowed to take up private practice, provided the other party is not the state.
Further, without permission of the government he cannot defend the accused persons in criminal
matters.
Q Source:
http://www.thehindu.com/news/national/attorney-general-rohatgi-will-defend-indias-record-on-rights
-abuses-at-unhrcs-review/article18312468.ece
TEST - 1 (MOCK TEST - 1)
www.insightsias.com 73
© Insights Active Learning| All rights reserved. You may not reproduce, distribute or exploit the contents in any form without written
permission by copyright owner. Copyright infringers may face civil and criminal liability
65 In the context of the rule of South Indian dynasties, Devadhana and Brahmadeya referred to?
A. Pristine land
B. Divine king rule
C. Treasure appropriated to the Gods
D. Land grants to the temples
Your Answer : C
Correct Answer : D
Answer Justification :
Justification: Land tax was the primary source of the government revenue. The Brahmadeya and
Devadhana lands were exempted from tax.
Learning: This is a lateral explanation and deals with aspects other than those mentioned in the
question, because detailing the topic at hand is not of much use.
The Pallavas had a well organized administrative system. The Pallava state was divided into
Kottams. The Kottam was administered by officers appointed by the king.
It was also the responsibility of the central government to provide irrigation facilities to the
lands.
Traders and artisans such as carpenters, goldsmiths, washer-men, oil-pressers and weavers
paid taxes to the government.
The Pallava inscriptions throw much light on the village assemblies called sabhas and their
committees. They maintained records of all village lands, looked after local affairs and
managed temples.
Q Source: Page 126: TN 11th Standard History Textbook
TEST - 1 (MOCK TEST - 1)
www.insightsias.com 74
© Insights Active Learning| All rights reserved. You may not reproduce, distribute or exploit the contents in any form without written
permission by copyright owner. Copyright infringers may face civil and criminal liability
66 They introduced the art of excavating temples from the rock, and the Dravidian style of temple
architecture began with their rule. They were?
A. Cheras
B. Pallavas
C. Eastern Chalukyas
D. Jaffna
Your Answer : B
Correct Answer : B
Answer Justification :
Learning: Pallava age was a great age of temple building.
It was a gradual evolution starting from the cave temples to monolithic rathas and culminated in
structural temples.
The development of temple architecture under the Pallavas can be seen in several stages, which
started with Mahendravarman I introducing rock-cut temples.
This style of Pallava temples are seen at places like Mandagappattu, Mahendravadi, Mamandur,
Dalavanur, Tiruchirappalli, Vallam, Siyamangalam and Tirukalukkunram.
Q Source: Page 127: TN 11th Standard History Textbook
TEST - 1 (MOCK TEST - 1)
www.insightsias.com 75
© Insights Active Learning| All rights reserved. You may not reproduce, distribute or exploit the contents in any form without written
permission by copyright owner. Copyright infringers may face civil and criminal liability
67 In Delhi Sultanate, the Sultan was assisted by a number of departments and officials in his
administration. Consider the following with reference to these departments:
1. Diwani Insha dealt with the correspondence between the ruler and the officials.
2. Diwani Ariz gave grants for the construction and maintenance of mosques, tombs and madrasas.
Which of the above is/are correct?
A. 1 only
B. 2 only
C. Both 1 and 2
D. None
Your Answer :
Correct Answer : A
Answer Justification :
Justification: Statement 1: It is correct.
Statement 2: The military department was called Diwani Ariz. It was headed by Ariz-i-mumalik. He
was responsible for recruiting the soldiers and administering the military department.
Learning: Under the Delhi sultanate, the post of Naib was the most powerful one. The Naib
practically enjoyed all the powers of the Sultan and exercised general control over all the
departments.
Next to him was the Wazir who was heading the finance department called Diwani Wizarat.
Diwani Rasalat was the department of religious affairs. It was headed by chief Sadr. Grants
were made by this department for religious construction.
Q Source: Page 192: TN 11th Standard History Textbook
TEST - 1 (MOCK TEST - 1)
www.insightsias.com 76
© Insights Active Learning| All rights reserved. You may not reproduce, distribute or exploit the contents in any form without written
permission by copyright owner. Copyright infringers may face civil and criminal liability
68 In the context of Ancient India, Amogavarsha’s work Kavirajamarga is significant because?
A. It was the first poetic work in Kannada language.
B. The largest repository of royal assets was commissioned in this work.
C. All artists of the empire contributed to this work.
D. It was a prose written in the middle of a battle ground.
Your Answer :
Correct Answer : A
Answer Justification :
Learning: The Kannada literature saw its beginning during the period of the Rashtrakutas.
Amogavarsha’s Kavirajamarga was the first poetic work in Kannada language.
The name literally means "Royal Path for Poets" and was written as a guide book for poets and
scholars (Kavishiksha).
Q Source: Page 137: TN 11th Standard History Textbook
TEST - 1 (MOCK TEST - 1)
www.insightsias.com 77
© Insights Active Learning| All rights reserved. You may not reproduce, distribute or exploit the contents in any form without written
permission by copyright owner. Copyright infringers may face civil and criminal liability
69 Several accords have been signed between the Government of India (GoI) and different groups to
bring peace and prosperity to the North-eastern states. Consider the following with reference to
them.
1. Mizo accord promised statehood to Mizoram.
2. Assam Accord was reached between GoI and ULFA calming the Assam agitation.
3. Nagaland peace accord between GoI and the National Socialist Council of Nagaland (NSCN) was
signed to end the Naga insurgency.
Select the correct answer using the codes below.
A. 1 and 3 only
B. 3 only
C. 2 and 3 only
D. 1 and 2 only
Your Answer : A
Correct Answer : A
Answer Justification :
Justification: Statement 1: Under the Mizo accord, the Mizo National Front (MNF) fighters
surrendered en masse and government fulfilled its promise by giving full-fledged state status to
Mizoram, along with its own High Court.
Mizo was notified as an official Indian language. Further, Mizo religious and social customs
were promised Constitutional protection.
As per the accord, the MNF agreed to forgo all contacts with other insurgent groups in the
Northeast. Mizo Accord is considered to be the most successful agreement with insurgents.
Statement 2: ULFA is a banned organization. So, 2 is wrong.
The accord brought an end to the Assam Agitation and paved the way for the leaders of the
agitation to form a political party and form a government in the state of Assam soon after. However,
not all promises have been fulfilled that remains a bone of contention.
Statement 3: This article explains the history and issues with Naga insurgency very clearly and
chronologically: A must read – This is an important issue.
http://indianexpress.com/article/explained/simply-put-towards-accord-step-by-step/
Q Source:
http://www.thehindu.com/news/national/mizoram-celebrates-31st-anniversary-of-mizo-peace-accord/
article19187194.ece
TEST - 1 (MOCK TEST - 1)
www.insightsias.com 78
© Insights Active Learning| All rights reserved. You may not reproduce, distribute or exploit the contents in any form without written
permission by copyright owner. Copyright infringers may face civil and criminal liability
70 The system of village autonomy with sabhas and their committees developed through the ages and
reached its culmination during the Chola rule. A village was divided in several wards, what was/were
the qualifications to become a ward member?
1. A woman could not become a ward member.
2. Only those who have completed seventy years of age
3. Have knowledge of Vedas
4. Owns a residence
Select the correct answer using the codes below.
A. 3 and 4 only
B. 1, 2 and 3 only
C. 1 and 4 only
D. 1, 2, 3 and 4
Your Answer : C
Correct Answer : A
Answer Justification :
Learning: That village was divided into thirty wards and each was to nominate its members to the
village council.
The qualifications to become a ward member were:
Ownership of at least one fourth veli of land.
Own residence.
Above thirty years and below seventy years of age.
Knowledge of Vedas.
However, certain norms of disqualification were also mentioned in the inscriptions. They were:
Those who had been members of the committees for the past three years.
Those who had failed to submit accounts as committee members.
Those who had committed sins.
Those who had stolen the property of others
Two inscriptions belonging to the period of Parantaka I found at Uttiramerur provide details of the
TEST - 1 (MOCK TEST - 1)
www.insightsias.com 79
© Insights Active Learning| All rights reserved. You may not reproduce, distribute or exploit the contents in any form without written
permission by copyright owner. Copyright infringers may face civil and criminal liability
formation and functions of village councils.
Q Source: Page 148: TN 11th Standard History Textbook
TEST - 1 (MOCK TEST - 1)
www.insightsias.com 80
© Insights Active Learning| All rights reserved. You may not reproduce, distribute or exploit the contents in any form without written
permission by copyright owner. Copyright infringers may face civil and criminal liability
71 In the ancient traditions, traders’ voyages often refer which of these lands to Suvarnabhumi (the land
of gold)?
A. Countries of the East Asia
B. African nations
C. Sri Lankan islands
D. European countries
Your Answer :
Correct Answer : A
Answer Justification :
Learning: The east coast of India is studded with numerous ports and Indians undertook frequent
voyages to these lands.
Also, Indians began to colonize the East Asia in the Gupta period. It was further encouraged
by the Pallavas.
The Indian colonists established great kingdoms (with Indian names) and some of them lasted
for more than a thousand years.
Till the arrival of Islam in the fifteenth century, Indian culture dominated this region.
Q Source: Page 156: TN 11th Standard History Textbook
TEST - 1 (MOCK TEST - 1)
www.insightsias.com 81
© Insights Active Learning| All rights reserved. You may not reproduce, distribute or exploit the contents in any form without written
permission by copyright owner. Copyright infringers may face civil and criminal liability
72 The system of four ashrams is advocated in
A. Buddhism
B. Jainism
C. Lokayatas
D. Brahmanism
Your Answer : C
Correct Answer : D
Answer Justification :
Learning: Around the time when Jainism and Buddhism were becoming popular, brahmins
developed the system of ashramas.
Ashramas refer to stage of life: brahmacharya, grihastha, vanaprastha and samnyasa.
Brahmin, kshatriya and vaishya men were expected to lead simple lives and study the Vedas
during the early years of their life (brahmacharya).
Then they had to marry and live as householders (grihastha).
Then they had to live in the forest and meditate (vanaprastha). Finally, they had to give up
everything and become samnyasins.
The system of ashramas allowed men to spend some part of their lives in meditation.
Generally, women were not allowed to study the Vedas, and they had to follow the ashramas
chosen by their husbands.
Q Source: Page 173: TN 11th Standard History Textbook
TEST - 1 (MOCK TEST - 1)
www.insightsias.com 82
© Insights Active Learning| All rights reserved. You may not reproduce, distribute or exploit the contents in any form without written
permission by copyright owner. Copyright infringers may face civil and criminal liability
73 ‘Petya’ recently seen in news was a/an
A. Laser beaming innovation
B. Form of cyber-attack
C. Alternate to fossil fuels
D. Emergency helpline launched by PETA
Your Answer : B
Correct Answer : B
Answer Justification :
Learning: It is a ransomware, similar to the Wannacry ransomware.
In India, this ransomware crippled the operations at one of the terminals of the Jawaharlal Nehru
Port Trust and impacted the external trade by affecting the systems dealing with the cargo and
ships.
Ransomware is a type of malicious software that threatens to publish the victim's data or
perpetually block access to it unless a ransom is paid.
Q Source:
https://www.theguardian.com/technology/2017/jun/27/petya-ransomware-cyber-attack-who-what-wh
y-how
TEST - 1 (MOCK TEST - 1)
www.insightsias.com 83
© Insights Active Learning| All rights reserved. You may not reproduce, distribute or exploit the contents in any form without written
permission by copyright owner. Copyright infringers may face civil and criminal liability
74 In 1329-30 Muhammad bin Tughlaq introduced a token copper currency in place of silver, but banned
it later and promised to exchange silver coins for these copper coins. This was done because
A. Copper was scarce in the empire, but had huge demand in the world.
B. Token currency was trampled and disrespected by people.
C. Goldsmiths began to forge the token coins on a large scale
D. Treasury became empty with issue of token coins.
Your Answer : A
Correct Answer : C
Answer Justification :
Learning: There was a shortage of silver through out the world in the fourteenth century.
Kublai Khan issued paper money in China. In the same manner, Muhammad bin Tughlaq
issued copper coins at par with the value of the silver tanka coins.
But he was not able to prevent forging the new coins. The goldsmiths began to forge the
token coins on a large scale. Soon the new coins were not accepted in the markets due to its
widescale duplication and thus credibility.
Finally, Muhammad bin Tughlaq stopped the circulation of token currency and promised to
exchange silver coins for the copper coins.
Many people exchanged the new coins but the treasury became empty.
According the Barani, the heap of copper coins remained lying on roadside in Tughlaqabad.
Q Source: Page 184: TN 11th Standard History Textbook
TEST - 1 (MOCK TEST - 1)
www.insightsias.com 84
© Insights Active Learning| All rights reserved. You may not reproduce, distribute or exploit the contents in any form without written
permission by copyright owner. Copyright infringers may face civil and criminal liability
75 Writing in the journal Young India in 1931, who said, "I cannot possibly bear the idea that a man who
has got wealth should get the vote but a man who has got character but no wealth or literacy should
have no vote…. for the crime of being a poor man…"?
A. Jawahar Lal Nehru
B. M.K. Gandhi
C. Rajendra Prasad
D. Maulana Abul Kalam Azad
Your Answer :
Correct Answer : B
Answer Justification :
Learning: Young India, Harijan and Hind Swaraj were some of the most famous works of Gandhi.
Young India was a weekshed - a weekly paper or journal.
He used Young India to spread his unique ideology and thoughts regarding the use of nonviolence
in organising movements and to urge readers to consider, organise, and plan for India's eventual
independence from Britain.
The journal was reprinted in USA by the India Home Rule League of America.
Q Source: Improvisation: Page 33: NCERT Class VI Social and Political Life - I
TEST - 1 (MOCK TEST - 1)
www.insightsias.com 85
© Insights Active Learning| All rights reserved. You may not reproduce, distribute or exploit the contents in any form without written
permission by copyright owner. Copyright infringers may face civil and criminal liability
76 In the context of Pallava’s history, what was Dakshinachitra?
A. A compendium of temples
B. A spiritual treatise on Pallava practices
C. An autobiography of a Pallava ruler
D. An instruction manual for painters
Your Answer :
Correct Answer : D
Answer Justification :
Learning: Mahendravarman, one of the greatest of the rulers of the Pallava dynasty, was found of
painting and is said to have commissioned a scholar at his court to pen down an instruction manual
for painters, which is a treatise on south Indian paintings called Dakshinachitra.
Unfortunately, it has been lost in the course of time.
Mahendravarman I gave himself the title of “Vchitrachitta” (the man with new fangled ideas).
Q Source: Page 129: TN 11th Standard History Textbook
TEST - 1 (MOCK TEST - 1)
www.insightsias.com 86
© Insights Active Learning| All rights reserved. You may not reproduce, distribute or exploit the contents in any form without written
permission by copyright owner. Copyright infringers may face civil and criminal liability
77 In medieval India, takkavi loans were given in order to
A. Improve agriculture
B. Build religious shrines
C. Construct flood saving embankments in cities
D. Serve sovereign debt
Your Answer :
Correct Answer : A
Answer Justification :
Learning: Takkavi loans were a measure of reprieve taken after few suppressing moves of Tughlaq.
In order to overcome financial difficulties in his empire, Muhammad bin Tughlaq increased
the land revenue on the farmers of Doab (land between Ganges and Yamuna rivers).
It was an excessive and arbitrary step on the farmers. A severe famine was also ravaging that
region at that time. It had resulted in a serious peasant revolts. The revolts were crushed.
However, the Sultan realized later that adequate relief measures and the promotion of
agriculture were the real solution to the problem.
He launched a scheme by which takkavi loans (loans for cultivation) were given to the farmers
to buy seed and to extend agriculture.
Q Source: Page 193: TN 11th Standard History Textbook
TEST - 1 (MOCK TEST - 1)
www.insightsias.com 87
© Insights Active Learning| All rights reserved. You may not reproduce, distribute or exploit the contents in any form without written
permission by copyright owner. Copyright infringers may face civil and criminal liability
78 Consider the following statements.
1. Sankara denounced Hindu revivalist movement citing them as going against rationality and
modernity.
2. Ramanuja advocated prabattimarga or the path of self-surrender to God for salvation.
Which of the above is/are correct?
A. 1 only
B. 2 only
C. Both 1 and 2
D. None
Your Answer :
Correct Answer : B
Answer Justification :
Learning: Statement 1: In the ninth century Sankara started a Hindu revivalist movement giving a
new orientation to Hinduism. So, 1 is wrong.
However, his doctrine of Advaita or Monism was too abstract to appeal to the common man.
Moreover, there was a reaction against the Advaita concept of Nirgunabrahman (God without
attributes) with the emergence of the idea of Sagunabrahman (God with attributes).
Statement 2: In the twelfth century, Ramanuja, preached Visishtadvaita.
According to him God is Sagunabrahman. The creative process and all the objects in creation
are real but not illusory as was held by Sankaracharya.
Therefore, God, soul, matter are real. But God is inner substance and the rest are his
attributes.
He invited the downtrodden to Vaishnavism
Q Source: Page 202: TN 11th Standard History Textbook
TEST - 1 (MOCK TEST - 1)
www.insightsias.com 88
© Insights Active Learning| All rights reserved. You may not reproduce, distribute or exploit the contents in any form without written
permission by copyright owner. Copyright infringers may face civil and criminal liability
79 Consider the following statements.
1. A Supernova helps us know about the Universe since they are the longest existing phenomenon in
space.
2. Pulsars are de-magnetized neutron stars that neither rotate nor emit electromagnetic radiation.
3. Black hole refers to the remains of a supergiant star that has collapsed into itself.
4. The effects of microgravity can be seen when astronauts and objects float in space.
Select the correct answer using the codes below.
A. 1 and 3 only
B. 3 and 4 only
C. 2, 3 and 4 only
D. 1, 2 and 4 only
Your Answer : D
Correct Answer : B
Answer Justification :
Background: China has launched its first X-ray space telescope to study black holes, pulsars and
gamma-ray bursts.
Justification: Statement 1: A supernova happens where there is a change in the core, or centre, of
a star.
A supernova burns for only a short period of time, but it can tell scientists a lot about the universe.
One kind of supernova has shown scientists that we live in an expanding universe, one that is
growing at an ever increasing rate.
Statement 2: Pulsars are highly magnetized, rotating neutron stars that emit a beam of
electromagnetic radiation.
They are spherical, compact objects that are about the size of a large city but contain more
mass than that of the sun.
Scientists are using pulsars to study extreme states of matter, search for planets beyond
Earth’s solar system and measure cosmic distance.
Statement 3: A black hole cannot be seen because of the strong gravity that is pulling all of the light
into the black hole's centre.
However, scientists can see the effects of its strong gravity on the stars and gases around it.
If a star is orbiting a certain point in space, scientists can study the star's motion to find out if
it is orbiting a black hole.
TEST - 1 (MOCK TEST - 1)
www.insightsias.com 89
© Insights Active Learning| All rights reserved. You may not reproduce, distribute or exploit the contents in any form without written
permission by copyright owner. Copyright infringers may face civil and criminal liability
Statement 4: Microgravity is the condition in which people or objects appear to be weightless. This
occurs when gravitation force around is weak (but not zero) or when objects are in a state of free
fall (as in a satellite).
For example, astronauts can move equipment weighing hundreds of pounds with their fingertips
due to microgravity.
Q Source: NASA Website and China’s telescope launch
TEST - 1 (MOCK TEST - 1)
www.insightsias.com 90
© Insights Active Learning| All rights reserved. You may not reproduce, distribute or exploit the contents in any form without written
permission by copyright owner. Copyright infringers may face civil and criminal liability
80 The highest court of appeal in the Vijayanagar Empire was
A. Gram Sabha
B. Mantri Parishad
C. Qazi’s Court
D. The King
Your Answer :
Correct Answer : D
Answer Justification :
Learning: The administration under the Vijayanagar Empire was organized in the way that the king
enjoyed absolute authority in executive, judicial and legislative matters. He was the highest court of
appeal.
The king was assisted by a council of ministers in his day to day administration. But, word of the
king was final.
Moreover, the succession to the throne was on the principle of hereditary.
Q Source: Page 212: TN 11th Standard History Textbook
TEST - 1 (MOCK TEST - 1)
www.insightsias.com 91
© Insights Active Learning| All rights reserved. You may not reproduce, distribute or exploit the contents in any form without written
permission by copyright owner. Copyright infringers may face civil and criminal liability
81 The beginning of agriculture can be ascribed most suitably to which of these times?
A. 2,500 years ago with the onset of Magadha empire
B. 4,700 years ago with the appearance of the first cities on the Indus
C. Nearly 8,000-10,000 years ago
D. About 25,000 years ago in the first Penistone glacial period
Your Answer : B
Correct Answer : C
Answer Justification :
Justification & Learning: Option D: People evolved into their current form some 200,000 years
ago (see human evolution), yet they did not begin to engage in agriculture until about
15,000–10,000 years before the present.
Wild grains were collected and eaten from at least 20,000 BC. So, D is wrong.
Option C: The first agriculture appears to have developed at the closing of the last Pleistocene
glacial period, or Ice Age (about 11,700 years ago).
From around 9,500 BC, the Neolithic founder crops such as emmer wheat, einkorn wheat, hulled
barley, peas etc were cultivated in the Levant. Rice was domesticated in China between 11,500 and
6,200 BC.
Q Source: Page 10: NCERT Class VI Our Pasts - I
TEST - 1 (MOCK TEST - 1)
www.insightsias.com 92
© Insights Active Learning| All rights reserved. You may not reproduce, distribute or exploit the contents in any form without written
permission by copyright owner. Copyright infringers may face civil and criminal liability
82 With reference to the International Tribunal for the Law of the Sea (ITLOS), consider the following:
1. It is an organ of the United Nations adjudicating disputes arising out of the interpretation and
application of the UN Convention on the Law of the Seas (UNCLOS).
2. Each state party of the ITLOS nominates a representative to become a member of ITLOS.
Which of the above is/are correct?
A. 1 only
B. 2 only
C. Both 1 and 2
D. None
Your Answer :
Correct Answer : D
Answer Justification :
Justification: Statement 1: The Tribunal is an independent judicial body but maintains close links
with the United Nations.
The Tribunal and the UN have entered into agreement concerning cooperation and relationship.
UN has granted the Tribunal observer status in the General Assembly.
Statement 2: The Tribunal is composed of 21 independent members, elected from among persons
enjoying the highest reputation for fairness and integrity and of recognized competence in the field
of the law of the sea.
Recently, law expert Neeru Chadha has become the first Indian women to elected to the
International Tribunal for the Law of the Seas (ITLOS).
Neeru has also earned a distinction of becoming the second Indian woman to occupy a top
position at the United Nations after Lakshmi Pandit who served as the President of the United
Nations General Assembly.
Q Source:
http://www.thehindu.com/news/international/neeru-chadha-becomes-1st-indian-woman-as-member-o
f-itlos/article19053500.ece
TEST - 1 (MOCK TEST - 1)
www.insightsias.com 93
© Insights Active Learning| All rights reserved. You may not reproduce, distribute or exploit the contents in any form without written
permission by copyright owner. Copyright infringers may face civil and criminal liability
83 This archaeological site consist of a number of low archaeological mounds created by generations of
superimposed mudbrick structures. Located close to the mouth of the Bolan Pass, it was abandoned
by the time of the emergence of the literate urbanised phase of the Indus Civilisation. An entry in the
UNESCO tentative list it is?
A. Lothal
B. Mehrgarh
C. Inamgaon
D. Surkotada
Your Answer :
Correct Answer : B
Answer Justification :
Learning: It was a small farming village. It is one of the earliest sites with evidence of farming and
herding in South Asia.
Mehrgarh is now seen as a precursor to the Indus Valley Civilization, displaying the whole sequence
from earliest settlement and the start of agriculture, to the mature Harappan Civilisation (based on
the above Q description of site abandonment).
Most of its archaeological deposits are buried deep beneath accumulations of alluvium.
Q Source: Improvisation: Page 29 (image): NCERT Class VI Our Pasts - I
TEST - 1 (MOCK TEST - 1)
www.insightsias.com 94
© Insights Active Learning| All rights reserved. You may not reproduce, distribute or exploit the contents in any form without written
permission by copyright owner. Copyright infringers may face civil and criminal liability
84 Integrated Child Development Services (ICDS) Scheme has a bearing on
1. Infant Mortality rate (IMR)
2. Maternal Mortality rate (MMR)
3. Reduction of malnutrition in children
4. Pre-school non-formal education
Select the correct answer using the codes below.
A. 1 and 3 only
B. 2 and 4 only
C. 1, 2 and 3 only
D. 1, 2, 3 and 4
Your Answer : C
Correct Answer : D
Answer Justification :
Justification: It is a centrally sponsored scheme implemented by the states/UTs. The Scheme was
launched in 1975.
Statement 1 and 3: The scheme has the following objectives:
to improve the nutritional and health status of children in the age-group 0-6 years; this leads
to an improvement in IMR.
to reduce mortality, morbidity, malnutrition and school dropout;
to promote policy coordination and implementation for promoting child development;
to lay an adequate foundation for the psychological, physical and social well-being of the
children etc.
Statement 2 and 4: Under the scheme, the services such as Supplementary nutrition; Pre-school
non-formal education; nutrition and health education, immunization, health check-ups; and Referral
services through Anganwadi Centres for children below 6 years of age as well as to pregnant
women& lactating mothers are offered. These improve MMR.
Learning: Infant mortality rate (IMR) is the number of deaths per 1,000 live births of children
under one year of age. It is presently around 40 in India.
The Maternal mortality rate (MMR) is the annual number of female deaths per 100,000 live births
from any cause related to or aggravated by pregnancy or its management (excluding accidental or
incidental causes). The recent World Bank data puts the MMR for India reported in 2015 at 174.
Q Source: CRY report on children in India
TEST - 1 (MOCK TEST - 1)
www.insightsias.com 95
© Insights Active Learning| All rights reserved. You may not reproduce, distribute or exploit the contents in any form without written
permission by copyright owner. Copyright infringers may face civil and criminal liability
85 The river(s) that do NOT find mention in Rigveda is?
A. Ganga and Yamuna
B. Indus and its other tributaries
C. Saraswati
D. All of the above find mention.
Your Answer : D
Correct Answer : D
Answer Justification :
Learning: Option B: There is a hymn in the form of a dialogue between sage Vishvamitra, and two
rivers, (Beas and Sutlej) that were worshipped as goddesses.
Option C: The most prominent river of the Rigveda is the Sarasvati, next to the Indus.
Option A: The Ganga and Yamuna are named only once.
Q Source: Page 45: NCERT Class VI Our Pasts - I
TEST - 1 (MOCK TEST - 1)
www.insightsias.com 96
© Insights Active Learning| All rights reserved. You may not reproduce, distribute or exploit the contents in any form without written
permission by copyright owner. Copyright infringers may face civil and criminal liability
86 There are plenty of source materials to reconstruct the history of the Gupta period. Some of the
renowned material, in this context, includes
1. Puranas throw light on the royal genealogy of the Gupta kings.
2. Devichandraguptam and Mudhrakshasam provide information regarding the rise of the Guptas
3. Meherauli Iron Pillar Inscription mention their conquests.
4. Fahien’s writings gives an account of the social, economic and religious condition of the empire.
Select the correct answer using the codes below.
A. 1 and 4 only
B. 2, 3 and 4 only
C. 1 and 3 only
D. 1, 2, 3 and 4
Your Answer : D
Correct Answer : D
Answer Justification :
Learning: S1 and S2 mention commonly known sources.
Statement 3: The Meherauli Iron Pillar inscription mentions the extensive conquests of
Chandragupta I. He is considered to be the founder of the Gupta era which starts with his accession
in A.D. 320.
The Allahabad Pillar inscription mentions that Samudragupta defeated twelve rulers in his South
Indian Expedition.
Statement 4: We will cover a separate question on Fahien’s accounts. It is important to note that his
main interest of visiting India was religious.
Q Source: Page 96-97: TN 11th Standard History Textbook
TEST - 1 (MOCK TEST - 1)
www.insightsias.com 97
© Insights Active Learning| All rights reserved. You may not reproduce, distribute or exploit the contents in any form without written
permission by copyright owner. Copyright infringers may face civil and criminal liability
87 Buddhist Viharas are
A. A monument to meditate and chant for the entire Sangha
B. Permanent shelters or monasteries for dwelling and rest
C. Divine extensions of stupas
D. Sites containing relics of Buddha
Your Answer : B
Correct Answer : B
Answer Justification :
Learning: To begin with, both Jaina and Buddhist monks went from place to place throughout the
year, teaching people.
The only time they stayed in one place was during the rainy season, when it was very difficult
to travel.
Then, their supporters built temporary shelters for them in gardens, or they lived in natural
caves in hilly areas.
As time went on, many supporters of the monks and nuns, and they themselves, felt the need
for more permanent shelters and so monasteries were built.
These were known as viharas.
Q Source: Page 71: NCERT Class VI Our Pasts - I
TEST - 1 (MOCK TEST - 1)
www.insightsias.com 98
© Insights Active Learning| All rights reserved. You may not reproduce, distribute or exploit the contents in any form without written
permission by copyright owner. Copyright infringers may face civil and criminal liability
88 PRAGATI is a unique integrating and interactive platform to
1. Address the grievances of citizens
2. Monitoring and reviewing important government projects
Which of the above is/are correct?
A. 1 only
B. 2 only
C. Both 1 and 2
D. None
Your Answer : B
Correct Answer : C
Answer Justification :
Justification: PRAGATI stands for Pro-Active Governance And Timely Implementation.
Key features of the PRAGATI application are as follows:
It is a three-tier system (PMO, Union Government Secretaries, and Chief Secretaries of the
States);
Prime Minister will hold a monthly programme where he will interact with the Government of
India Secretaries, and Chief Secretaries through Video-conferencing enabled by data and geoinformatics
visuals;
Issues to be flagged before the PM are picked up from the available database regarding
Public Grievances, on-going Programmes and pending Projects;
It will also take into consideration various correspondences to PM’s office by the common
people or from high dignitaries of States and/or developers of public projects;
Q Source: PIB AKT/NT: Release ID :167366
TEST - 1 (MOCK TEST - 1)
www.insightsias.com 99
© Insights Active Learning| All rights reserved. You may not reproduce, distribute or exploit the contents in any form without written
permission by copyright owner. Copyright infringers may face civil and criminal liability
89 In ancient Southern India, kadaisiyar and adimai were used to refer to
A. Ordinary ploughmen
B. Landless labourers, including slaves
C. Tax officials
D. Large landowners
Your Answer :
Correct Answer : B
Answer Justification :
Learning: Society was segregated even in ancient India.
There were at least three different kinds of people living in most villages in the southern and
northern parts of the ancient Indian subcontinent.
In the Tamil region, large landowners were known as vellalar, ordinary ploughmen were known as
uzhavar, and landless labourers, including slaves, were known as kadaisiyar and adimai.
In the northern region, this distinction was made in terms of land, political position and assets held.
Q Source: Page 88: NCERT Class VI Our Pasts - I
TEST - 1 (MOCK TEST - 1)
www.insightsias.com 100
© Insights Active Learning| All rights reserved. You may not reproduce, distribute or exploit the contents in any form without written
permission by copyright owner. Copyright infringers may face civil and criminal liability
90 Popularly known Jatakas were
A. Stories of previous lives of the Buddha
B. Objects of concentration
C. Donation passed from student to teacher
D. Arahants who sacrificed their lives for the service of Sangha
Your Answer : A
Correct Answer : A
Answer Justification :
Learning: They are a voluminous body of literature native to India concerning the previous births
of Gautama Buddha in both human and animal form.
The future Buddha may appear as a king, an outcast, a god, an elephant—but, in whatever
form, he exhibits some virtue that the tale thereby inculcates.
In Theravada Buddhism, the Jatakas are a textual division of the Pali Canon, included in the
Khuddaka Nikaya of the Sutta Pitaka.
The term Jataka may also refer to a traditional commentary on this book. Q Source: Page 89:
NCERT Class VI Our Pasts – I
TEST - 1 (MOCK TEST - 1)
www.insightsias.com 101
© Insights Active Learning| All rights reserved. You may not reproduce, distribute or exploit the contents in any form without written
permission by copyright owner. Copyright infringers may face civil and criminal liability
91 Consider the following statements.
1. The Anguttara Nikaya, a part of Abhidhamma Pitaka, contains thematically linked discourses of
Buddha.
2. The Majjhima Nikaya, a segment of Vinaya Pitaka, contains the smallest discourses given by the
Buddha to his close disciples.
Which of the above is/are correct?
A. 1 only
B. 2 only
C. Both 1 and 2
D. None
Your Answer :
Correct Answer : D
Answer Justification :
Justification: Both are a part of Sutta Pitaka. The word Nikaya is most commonly used in reference
to the Buddhist texts of the Sutta Piáš­aka.
Statement 1: This is incorrect because Abhidhamma Pitaka is a detailed scholastic reworking of
material appearing in the Suttas, not a collection of original discourses by the Buddha himself.
Statement 2: This is incorrect because Vinaya pitaka contains rules for the Sangha, it isn’t known
for hosting discourses.
Q Source: Improvisation: Page 47: TN 11th Standard History Textbook
TEST - 1 (MOCK TEST - 1)
www.insightsias.com 102
© Insights Active Learning| All rights reserved. You may not reproduce, distribute or exploit the contents in any form without written
permission by copyright owner. Copyright infringers may face civil and criminal liability
92 Changthangi goat of Northern Himalayan region is known for yielding
A. Pashmina wool
B. Shahtoosh
C. Himayalan Yew
D. Pitas and Humas
Your Answer : A
Correct Answer : A
Answer Justification :
Learning: They are a breed of goat inhabiting the plateaus in Tibet and neighbouring areas of
Ladakh in Jammu & Kashmir.
They are raised for ultra-fine cashmere wool Pashmina, which literally translates to "Soft Gold" in
Kashmiri.
The Changthangi goats have revitalized the poor economy of Changthang, Leh and Ladakh region.
Q Source: Page 9: NCERT Class VI Social and Political Life - I
TEST - 1 (MOCK TEST - 1)
www.insightsias.com 103
© Insights Active Learning| All rights reserved. You may not reproduce, distribute or exploit the contents in any form without written
permission by copyright owner. Copyright infringers may face civil and criminal liability
93 Consider the following about the role and contributions of B.R. Ambedkar in modern India:
1. He inspired the Dalit Buddhist Movement.
2. He was Independent India's first law minister.
3. He served as a Governor of Reserve Bank of India.
Select the correct answer using the codes below.
A. 1 and 2 only
B. 2 and 3 only
C. 3 only
D. 1 and 3 only
Your Answer :
Correct Answer : A
Answer Justification :
Justification: Statement 1: He spent his life advocating political rights and social freedom for
Dalits. In 1956 he converted to Buddhism, initiating mass conversions of Dalits.
Statement 3: The Reserve Bank of India (RBI), was based on the ideas that Ambedkar presented to
the Hilton Young Commission. He, however, did not serve as its Governor.
Ambedkar was the first Indian to pursue a doctorate in economics abroad. He argued that
industrialisation and agricultural growth could enhance the Indian economy.
Q Source: Improvisation: Page 20: NCERT Class VI Social and Political Life - I
TEST - 1 (MOCK TEST - 1)
www.insightsias.com 104
© Insights Active Learning| All rights reserved. You may not reproduce, distribute or exploit the contents in any form without written
permission by copyright owner. Copyright infringers may face civil and criminal liability
94 SATH - ‘Sustainable Action for Transforming Human capital’ initiative of NITI Aayog focuses on
A. Creating ‘role model’ states for health systems
B. Revamping higher education
C. Building workforce for the future in areas of cutting edge technology
D. Eradicating manual scavenging
Your Answer : C
Correct Answer : A
Answer Justification :
Learning: This was launched with the State Governments for furthering the agenda of cooperative
federalism.
NITI Aayog under the SATH initiative plans to identify and build three future ‘role model’
states for health systems.
After identification of three states, NITI Aayog will work in close collaboration with the state
machinery of these three states.
It will help in designing a robust roadmap, develop a program governance structure, establish
monitoring and tracking mechanisms, and provide support to the state institutions to achieve
the end objectives.
SATH program will be implemented by NITI Aayog along with McKinsey & Company and IPE
Global consortium.
Parameters such as MMR, IMR, incidence of malaria will be considered for determining
potential impact.
Q Source:
http://www.thehindu.com/business/niti-aayog-unveils-sath-programme/article18956584.ece
TEST - 1 (MOCK TEST - 1)
www.insightsias.com 105
© Insights Active Learning| All rights reserved. You may not reproduce, distribute or exploit the contents in any form without written
permission by copyright owner. Copyright infringers may face civil and criminal liability
95 The decision to locate a common well in a particular area of the village can be taken by
A. Central Government only
B. State Government only
C. District bureaucracy only
D. Panchayati Raj Institutions (PRIs)
Your Answer : D
Correct Answer : D
Answer Justification :
Learning: PRIs were established with an objective of democratic decentralization.
State legislatures can devolve powers to PRIs under 73rd constitutional amendment. Such powers
can involve management of local markets, public works, sanitation, public health etc.
Under these provisions, a public well can be established by PRIs.
Q Source: Improvisation: Page 30: NCERT Class VI Social and Political Life - I
TEST - 1 (MOCK TEST - 1)
www.insightsias.com 106
© Insights Active Learning| All rights reserved. You may not reproduce, distribute or exploit the contents in any form without written
permission by copyright owner. Copyright infringers may face civil and criminal liability
96 With reference to Buddhist disciples, consider the following.
1. Sariputta was a chief female disciple of Gautama Buddha.
2. Khema, who joined the Buddhist Sangha, was one of the queens of King Bimbisara.
3. Moggallana, a disciple of Buddha was known for his psychic powers.
4. King Ajatasatru of Magadha and King Prasenajit of Kosala became Buddha’s disciples.
Select the correct answer using the codes below.
A. 1 and 2 only
B. 3 and 4 only
C. 2, 3 and 4 only
D. 1 and 3 only
Your Answer :
Correct Answer : C
Answer Justification :
Justification: Statement 1: Sariputta was one of two chief male disciples of Gautama Buddha along
with Moggallana, counterparts to the bhikkhunis Khema and Uppalavanna, his two chief female
disciples. They were to maintain the order of monks and nuns.
Statement 2: The conversion of Khema was one of the rare cases where the Buddha used his
psychic powers to make a change in the heart of another.
Statement 3: Moggallana attained enlightenment shortly after joining the Sangha. As a teacher, he
became known for his psychic powers, which he used extensively in his teaching methods.
Statement 4: Kings like Prasenajit of Kosala and Bimbisara and Ajatasatru of Magadha too accepted
the doctrines of the Buddha and became his disciples.
Q Source: Improvisation: Page 41: TN 11th Standard History Textbook
TEST - 1 (MOCK TEST - 1)
www.insightsias.com 107
© Insights Active Learning| All rights reserved. You may not reproduce, distribute or exploit the contents in any form without written
permission by copyright owner. Copyright infringers may face civil and criminal liability
97 With reference to the International Telecommunication Union (ITU), consider the following
statements.
1. It is a non-profit non-governmental organization comprising private sector ISP representatives.
2. It releases the Global Cybersecurity Index (GCI).
3. It is responsible for allocating global radio spectrum and satellite orbits.
4. It develops the technical standards that ensure networks and technologies to seamlessly
interconnect.
5. NASSCOM is the nodal agency for coordinating with ITU from India, as a member country.
6. It strives to improve access to Information and Communication Technology among the underserved
communities worldwide.
Select the correct answer using the codes below.
A. 1, 4 and 5 only
B. 2, 3, 4 and 6 only
C. 3, 5 and 6 only
D. 1, 2, 3, 4, 5 and 6
Your Answer : D
Correct Answer : B
Answer Justification :
Justification: Statement 1: ITU is the United Nations specialised agency for information and
communication technologies (ICTs). ITU has 193 countries as its members and also 800 privatesector
entities and academic institutions.
Statement 2: India has been ranked 23rd (“maturing category” of the index ) out of 165 nations in
the second Global Cybersecurity Index (GCI) that measures the commitment of nations across the
world to cyber security.
ITU has ranked the countries based on the countries’ legal, technical and organisational
institutions, educational and research capabilities, as well as their cooperation in informationsharing
networks.
Statement 3 and 4: There must be a global agency that coordinates these standards (lie ICANN),
else telecommunication between different nations would have been difficult.
Statement 6: India is a member country since 1869 (british times), but it is the Department of
Telecommunications that is the nodal agency for such coordination.
Q Source: Release of Global Cybersecurity Index (GCI) 2017
TEST - 1 (MOCK TEST - 1)
www.insightsias.com 108
© Insights Active Learning| All rights reserved. You may not reproduce, distribute or exploit the contents in any form without written
permission by copyright owner. Copyright infringers may face civil and criminal liability
98 Firoz Tughlaq’s welfare state included which of these administrative measures?
1. Treating Hindus and Muslims alike in terms of political and economic rights
2. Abolition of the practice of slavery
3. Establishment of government department to take care of orphans and widows
Select the correct answer using the codes below.
A. 1 and 3 only
B. 2 and 3 only
C. 3 only
D. 1 and 2 only
Your Answer :
Correct Answer : C
Answer Justification :
Justification: Statement 1: As Firoz was guided by the ulemas, he was intolerant towards Shia
Muslims and Sufis. He treated Hindus as second grade citizens and imposed Jiziya. In this respect
he was the precursor of Sikandar Lodi and Aurangazeb.
Statement 2: Also he increased the number of slaves by capturing the defeated soldiers and young
persons. So, 2 is wrong.
Statement 3: A new department called Diwan-i-Khairat was created to take care of orphans and
widows. Free hospitals and marriage bureaus for poor Muslims were also established.
Q Source: Page 185-186: TN 11th Standard History Textbook
TEST - 1 (MOCK TEST - 1)
www.insightsias.com 109
© Insights Active Learning| All rights reserved. You may not reproduce, distribute or exploit the contents in any form without written
permission by copyright owner. Copyright infringers may face civil and criminal liability
99 With reference to the National Mission on Education through ICT (NMEICT), consider the following:
1. Certification of attainments of any kind at any level acquired through formal or non-formal means in
conventional or non-conventional fields
2. Platform for sharing of ideas and techniques and pooling of knowledge resources
3. Scholarship and Talent management including identification, nurturing and disbursement
electronically
Which of the above is/are the objectives of NMEICT?
A. 1 and 2 only
B. 2 only
C. 1 and 3 only
D. 1, 2 and 3
Your Answer :
Correct Answer : D
Answer Justification :
Learning: These are the guiding philosophies for NMECIT –
(a) no talent of the country should be allowed to go waste,
(b) all the services available through the content delivery portal Sakshat (NMEICT) should be free
and freely available material on the web should be used so as to avoid reinventing the wheel.
Scheme: It has been envisaged as a Centrally Sponsored Scheme to leverage the potential of ICT,
in teaching and learning process for the benefit of all the learners in Higher Education Institutions
in any time anywhere mode.
The Mission has two major components:
providing connectivity, along with provision for access devices, to institutions and learners;
Content generation.
The content portion of this Mission would have an ambitious vision of catering to the learning needs
of more than 50 crore Indians (working population) and of providing a one stop solution to all the
requirements of the learning community.
The objectives and details can be read at
http://mhrd.gov.in/sites/upload_files/mhrd/files/upload_document/MissionDocument.pdf
TEST - 1 (MOCK TEST - 1)
www.insightsias.com 110
© Insights Active Learning| All rights reserved. You may not reproduce, distribute or exploit the contents in any form without written
permission by copyright owner. Copyright infringers may face civil and criminal liability
100 India Infrastructure Fund (IIF)
1. is a SEBI-registered domestic venture capital fund
2. sponsored entirely by foreign governments
3. invests only in greenfield projects
Select the correct answer using the codes below.
A. 1 only
B. 2 and 3 only
C. 1 and 3 only
D. None of the above
Your Answer :
Correct Answer : A
Answer Justification :
Justification: Statement 1: It is focused on long-term equity investments in a diversified portfolio of
infrastructure projects.
Statement 2: IIF has been sponsored by IDFC Limited (IDFC), along with Citigroup Inc.(Citi) and
India Infrastructure Finance Company Limited (IIFCL) as founder investors.
The establishment of IIF has received the strong support of the Government of India.
Statement 3: IIF’s portfolio is expected to comprise greenfield, brownfield and operational
assets/projects in core infrastructure sub-sectors including transport etc.
The current size of IIF is INR 38 billion with investor commitments from institutional investors in
India, USA, Canada, Europe, Japan and the Middle-East.
China-led Asian InfrastruTEST - 1 (MOCK TEST - 1)
www.insightsias.com 1
© Insights Active Learning| All rights reserved. You may not reproduce, distribute or exploit the contents in any form without written
permission by copyright owner. Copyright infringers may face civil and criminal liability
1 Social Entrepreneurship is running
A. Any non-profit business
B. An ecologically sustainable business
C. An inclusive business that has participation from major social groups and local community
D. A business that makes its money in a socially responsible way
Your Answer :
Correct Answer : D
Answer Justification :
Learning: It is the use of the techniques by start-up companies and other entrepreneurs to develop,
fund and implement solutions to social, cultural, or environmental issues.
Muhammad Yunus (Grameen Bank) was a famous social entrepreneur. Another example can be a
crowdfunding internet platform like https://milaap.org
These ventures are not necessarily formed to reinvest all profits into the communities.
So, these firms apply commercial strategies to maximize improvements in human and
environmental well-being.
Q Source: Often seen in news: A recent speech by the PM
TEST - 1 (MOCK TEST - 1)
www.insightsias.com 2
© Insights Active Learning| All rights reserved. You may not reproduce, distribute or exploit the contents in any form without written
permission by copyright owner. Copyright infringers may face civil and criminal liability
2 Which of these are the chief difference(s) between Mesolithic and Neolithic cultures?
1. Hunting-gathering pattern of life began not before the Neolithic period.
2. Mesolithic cultures did not practice domestication of animals unlike Neolithic people.
3. The use of bow and arrow cannot be found in Mesolithic culture, unlike Neolithic culture.
4. Evidence of pottery is absent from the Neolithic culture but is markedly found in Mesolithic culture.
Select the correct answer using the codes below.
A. 3 and 4 only
B. 2 only
C. 1 and 3 only
D. None of the above
Your Answer : B
Correct Answer : D
Answer Justification :
Justification: Statement 1 and 3: Hunting-gathering pattern of life was prevalent since Old stone
age to Mesolithic and Neolithic.
However, there seems to have been a shift from big animal hunting to small animal hunting and
fishing. This is how bows and arrows were used in this period. Both 1 and 3 are thus wrong.
Statement 2: Domestication of animals, horticulture and primitive cultivation started during
Mesolithic period itself.
However, during Neolithic period, domestication of sheep, goats and cattle was widely prevalent.
Cattle were used for cultivation and for transport.
Statement 4: During Neolithic age, wheels were used to make pottery. Pottery was used for cooking
as well as storage of food grains. So, 4 is wrong.
Learning: During the Neolithic phase, the cultivation of plants and domestication of animals led to
the emergence of village communities based on sedentary life.
There was a great improvement in technology of making tools and other equipments used by
man. Stone tools were now polished.
Mud brick houses were built instead of grass huts.
Large urns were used as coffins for the burial of the dead.
Wheat, barely, rice, millets were cultivated in different areas at different points of time. Rice
cultivation was extensive in eastern India.
The people of Neolithic Age used clothes made of cotton and wool
TEST - 1 (MOCK TEST - 1)
www.insightsias.com 3
© Insights Active Learning| All rights reserved. You may not reproduce, distribute or exploit the contents in any form without written
permission by copyright owner. Copyright infringers may face civil and criminal liability
Q Source: Page 13-14: TN 11th Standard History Textbook
56154 - ROHIT YADAV - rohit.kshaurya@gmail.com - 9410878797
TEST - 1 (MOCK TEST - 1)
Total Marks : 200
Mark Scored :
www.insightsias.com 4
© Insights Active Learning| All rights reserved. You may not reproduce, distribute or exploit the contents in any form without written
permission by copyright owner. Copyright infringers may face civil and criminal liability
3 The Government of India has recently announced the sale of “Government of India Floating Rate Bonds
2024”. With reference to government securities, consider the following:
1. Floating Rate Bonds are sold by the Reserve Bank of India.
2. RBI issues Cash Management Bills (CMBs) to meet long-term borrowing needs of the Government
of India.
3. Treasury bills are short term debt instruments issued by the Government of India.
Select the correct answer using the codes below.
A. 1 and 2 only
B. 1 and 3 only
C. 2 only
D. 2 and 3 only
Your Answer : B
Correct Answer : B
Answer Justification :
Concept: A Government security is a tradable instrument issued by the Central Government or the
State Governments.
It acknowledges the Government’s debt obligation (to raise money from the market for
various purposes).
Such securities are short term (usually called treasury bills, with original maturities of less
than one year) or long term (usually called Government bonds or dated securities with
original maturity of one year or more).
Besides providing a return in the form of coupons (interest), Government securities offer the
maximum safety as they carry the Sovereign’s commitment for payment of interest and
repayment of principal.
Justification: Statement 1: There are two types of bonds:
Fixed Rate Bonds – These are bonds on which the coupon rate is fixed for the entire life of the
bond. Most Government bonds are issued as fixed rate bonds.
Floating Rate Bonds – Floating Rate Bonds are securities which do not have a fixed coupon rate.
The coupon is re-set at pre-announced intervals (say, every six months or one year) by adding a
spread over a base rate. These are issued by RBI.
Statement 2: Cash Management Bills (CMBs), are a relatively new short-term instrument to meet
the temporary mismatches in the cash flow of the Government. The CMBs have the generic
character of T-bills but are issued for maturities less than 91 days.
Statement 3: Treasury bills or T-bills, which are money market instruments, are short term debt
instruments issued by the Government of India and are presently issued in three tenors, namely, 91
day, 182 day and 364 day.
TEST - 1 (MOCK TEST - 1)
www.insightsias.com 5
© Insights Active Learning| All rights reserved. You may not reproduce, distribute or exploit the contents in any form without written
permission by copyright owner. Copyright infringers may face civil and criminal liability
Q Source: RBI Bond sales
TEST - 1 (MOCK TEST - 1)
www.insightsias.com 6
© Insights Active Learning| All rights reserved. You may not reproduce, distribute or exploit the contents in any form without written
permission by copyright owner. Copyright infringers may face civil and criminal liability
4 Ashokan inscriptions remain valuable sources for the study of Asoka and the Mauryan Empire. This is
because
1. His wooden pillars heralded a new architectural era because most of the monuments before his
period were made of stone.
2. Pillar edicts give a summary of his efforts to promote the Dhamma within his kingdom and outside.
3. They often deal with instructions given to his officials which is informative of Mauryan polity.
Select the correct answer using the codes below.
A. 2 and 3 only
B. 1 and 3 only
C. 1 and 2 only
D. 2 only
Your Answer : A
Correct Answer : A
Answer Justification :
Justification: Statement 1: The monuments before the period of Asoka were mostly made of wood
and therefore perished.
The use of stone started from the time of Asoka. Even of the numerous monuments of Asoka, only a
few have remained.
His palace and monasteries and most of his stupas have disappeared. The only remaining stupa is at
Sanchi.
Statement 2: The state and spread of Dhamma gives crucial information about the social life at the
time of Ashoka.
Statement 3: Ashoka often gave instructions to his Dhamma mahamahattas to spread Dhamma.
These instructions, their style etc were recorded on the edicts.
Learning: Ashokan inscriptions (first deciphered by James Princep) are written in Pali language
and in some places Prakrit was used.
The Brahmi script was employed for writing. In the northwestern India Asokan inscriptions were
found in Karoshti script.
There are fourteen Major Rock Edicts. The two Kaling Edicts are found in the newly conquered
territory. The major pillar Edicts were erected in important cities. There are minor Rock Edicts and
minor pillar Edicts.
Q Source: Page 60-61: TN 11th Standard History Textbook
TEST - 1 (MOCK TEST - 1)
www.insightsias.com 7
© Insights Active Learning| All rights reserved. You may not reproduce, distribute or exploit the contents in any form without written
permission by copyright owner. Copyright infringers may face civil and criminal liability
5 Which of the following is the closest analogy of the Indian Constitution?
A. Rulebook
B. Anthology
C. Dictionary
D. Social Register
Your Answer : A
Correct Answer : A
Answer Justification :
Justification: Option A: The Indian Constitution lays down the basic rules or laws that have to be
followed by everyone. These laws are for both the government and the people. Hence, A is
appropriate.
Option B: An anthology is a collection of literary works, originally of poems, which constitution is
not.
Option C: Constitution is not a mere dictionary of political and legal terms, it is much more than
that.
Option D: Social Register (SR) is a directory of people from certain social classes. Constitution is for
everyone. It isn’t a directory.
Q Source: Improvisation: Page 39: NCERT Class VI Social and Political Life - I
TEST - 1 (MOCK TEST - 1)
www.insightsias.com 8
© Insights Active Learning| All rights reserved. You may not reproduce, distribute or exploit the contents in any form without written
permission by copyright owner. Copyright infringers may face civil and criminal liability
6 ‘JIGYASA’, recently seen in news is a
A. Interstellar exploration programme of ISRO
B. Student- scientist connect programme
C. Earth core drilling programme of Ministry of Earth Sciences
D. Biodiversity documentation programme in the Western Ghats
Your Answer : B
Correct Answer : B
Answer Justification :
Learning: Council of Scientific and Industrial Research (CSIR), has joined hands with Kendriya
Vidyalaya Sangathan (KVS) to implement this programme.
The program will also enable the students and teachers to practically live the theoretical concepts
taught in science by visiting CSIR laboratories and by participating in mini-science projects. The
model of engagement includes:
Student Residential Programmes;
Scientists as Teachers and Teachers as Scientists;
Lab specific activities / Onsite Experiments;
The focus is on connecting school students and scientists so as to extend student’s classroom
learning with that of a very well-planned research laboratory based learning.
Q Source: http://pib.nic.in/newsite/PrintRelease.aspx?relid=167194
TEST - 1 (MOCK TEST - 1)
www.insightsias.com 9
© Insights Active Learning| All rights reserved. You may not reproduce, distribute or exploit the contents in any form without written
permission by copyright owner. Copyright infringers may face civil and criminal liability
7 Consider the following statements. Assertion (A): Every elector at the Presidential election is bound by
the voting instructions issued by the whip of the respective political party. Reason (R): Anti-defection
provisions are enforced by the constitution under the Tenth Schedule.
In the context of the above, which of these is correct?
A. A is correct, and R is an appropriate explanation of A.
B. A is correct, but R is not an appropriate explanation of A.
C. A is correct, but R is incorrect.
D. A is incorrect, but R is correct.
Your Answer : C
Correct Answer : D
Answer Justification :
Concept: Defection is defined as “to abandon a position or association, often to join an opposing
group” which essentially describes a situation when a member of a particular party abandons his
loyalty towards that party and provide his support (in the form of his vote or otherwise) to another
party.
To avoid such a situation (about which you’ll read later in Laxmikanth and 11th NCERT), an antidefection
provision was added in the constitution. If a member of a political party disobeys party
directions concerning matters such as voting on a bill or for a candidate, this amounts to defection.
The member is then liable to be punished by the party.
Justification: However, the ECI has clarified recently that voting or not voting as per his/her own
free will at the Presidential election and the electors are at liberty to vote or not to vote at the
Presidential election as per their own free will and choice.
This will equally apply to the political parties and they are free to canvas or seek votes of
electors for any candidate or requesting or appealing to them to refrain from voting.
However, the political parties cannot issue any direction or whip to their members to vote in a
particular manner or not to vote at the election leaving them with no choice, as that would
tantamount to the offence of undue influence within the meaning of section 171C of the IPC.
This is because, as per the ECI, voting at election to the office of President is different from
voting by a member of Parliament or State Legislature inside the House.
Q Source: RM/ RS: Release ID :167193
TEST - 1 (MOCK TEST - 1)
www.insightsias.com 10
© Insights Active Learning| All rights reserved. You may not reproduce, distribute or exploit the contents in any form without written
permission by copyright owner. Copyright infringers may face civil and criminal liability
8 With reference to unique BRICS countries initiatives, consider the following statements:
1. BRICS Network University (NU) is an open educational project and does not involve establishment
of rigid organizational forms such as a secretariat.
2. BRICS Think Tanks Council (BTTC) comprises research organizations from BRICS countries
including Observer Research Foundation (ORF) from India.
Which of the above is/are correct?
A. 1 only
B. 2 only
C. Both 1 and 2
D. None
Your Answer :
Correct Answer : C
Answer Justification :
Justification: Recently at the 5th BRICS Education ministers meeting, Beijing declaration on
education was adopted which advocated support for both these organizations.
Statement 1: NU is a consortium of the universities, elaborating Master and PhD programmes in
different disciplines. It was established in 2015 in BRICS meeting at Moscow.
NU follows principles of:
Openness, which enables various universities to join the network respecting national criteria.
Focus on educational programmes that can be supplemented with network research and
innovation projects.
There is also a BRICS Universities League which is a consortium of leading research universities
from BRICS countries.
Statement 2: The BTTC (established in 2013) comprises the Institute for Applied Economic
Research (IPEA) (Brazil), the National Committee for BRICS Research (NRC/BRICS) (Russia), the
Observer Research Foundation (ORF) etc.
The main purposes of its establishment was to form a platform for the exchange of ideas and
present policy recommendations for BRICS leaders.
Q Source: Beijing declaration on Education: 5th July 2017:
http://pib.nic.in/newsite/PrintRelease.aspx?relid=167154
TEST - 1 (MOCK TEST - 1)
www.insightsias.com 11
© Insights Active Learning| All rights reserved. You may not reproduce, distribute or exploit the contents in any form without written
permission by copyright owner. Copyright infringers may face civil and criminal liability
9 With reference to the Sangam age, Panar and Viraliyar were
A. Temple guards
B. Bull fighters
C. Singing bards
D. Charity donors
Your Answer :
Correct Answer : C
Answer Justification :
Learning: Poetry, music and dancing were popular among the people of the Sangam age. Liberal
donations were given to poets by the kings, chieftains and nobles.
The royal courts were crowded with singing bards called Panar and Viraliyar. They were
experts in folk songs and folk dances.
The arts of music and dancing were highly developed. A variety of Yazhs and drums are
referred to in the Sangam literature.
Dancing was performed by Kanigaiyar. Koothu was the most popular entertainment of the
people.
Q Source: Page 91: TN 11th Standard History Textbook
TEST - 1 (MOCK TEST - 1)
www.insightsias.com 12
© Insights Active Learning| All rights reserved. You may not reproduce, distribute or exploit the contents in any form without written
permission by copyright owner. Copyright infringers may face civil and criminal liability
10 The 600-year-old Walled City of Ahmedabad was recently declared a World Heritage City. With
reference to the World Heritage Cities Programme and associated initiatives, consider the following:
1. It is a thematic programmes formally approved and monitored by the World Heritage Committee.
2. The programme has provisions for technical assistance to States Parties for the implementation of
new approaches and schemes for urban heritage conservation.
3. IUCN has set up the ‘Historic Urban Landscape initiative’ which is an international working group
comprising, inter alia, UNESCO as an advisory body to the World Heritage Convention.
4. The Organization of World Heritage Cities (OWHC) is an inter-governmental organization of the
municipal corporations of cities in which sites of the UNESCO World Heritage list are located.
Select the correct answer using the codes below.
A. 1 and 2 only
B. 3 and 4 only
C. 1, 2 and 3 only
D. 1, 2, 3 and 4
Your Answer :
Correct Answer : A
Answer Justification :
Justification: Statement 1 and 2: It aims to assist States Parties in the challenges of protecting and
managing their urban heritage. The programme is structured along a two-way process, with 1) the
development of a theoretical framework for urban heritage conservation, and 2) the provision of
technical assistance to States Parties for the implementation of new approaches and schemes.
Statement 3: As part of this policy process of conservation, UNESCO (not IUCN) set up the Historic
Urban Landscape initiative, an international working group comprising ICOMOS, IUCN and
ICCROM (as Advisory Bodies to the 1972 World Heritage Convention) and other partner
organizations, including UN-Habitat etc.
Statement 4: OWHC is an international non-profit, non-governmental organization of 250 cities in
which sites of the UNESCO World Heritage list are located.
Learning: It is first Indian city get the honour. The Walled City of Ahmedabad is located on the
eastern banks of Sabarmati river.
It was founded by Sultan Ahmed Shah I of Gujarat Sultanate in in 1411.
It remained the capital of the Gujarat Sultanate and later important political and commercial
centre of Gujarat.
It presents a rich architectural heritage from the sultanate period, notably the Bhadra citadel,
walls and gates of the Fort city and numerous mosques and tombs, as well as important Hindu
and Jain temples of later periods.
Q Source: Improvisation:
http://www.thehindu.com/news/national/other-states/ahmedabad-declared-indias-first-heritage-cityTEST
- 1 (MOCK TEST - 1)
www.insightsias.com 13
© Insights Active Learning| All rights reserved. You may not reproduce, distribute or exploit the contents in any form without written
permission by copyright owner. Copyright infringers may face civil and criminal liability
by-unesco/article19244478.ece
TEST - 1 (MOCK TEST - 1)
www.insightsias.com 14
© Insights Active Learning| All rights reserved. You may not reproduce, distribute or exploit the contents in any form without written
permission by copyright owner. Copyright infringers may face civil and criminal liability
11 What argument(s) would you advance to justify that the position of women had become miserable
during the Gupta period?
1. Women were prohibited from studying religious texts.
2. The practice of Swyamvara was given up and Manusmriti suggested early marriage for girls.
Which of the above is/are correct?
A. 1 only
B. 2 only
C. Both 1 and 2
D. None
Your Answer : A
Correct Answer : C
Answer Justification :
Justification: Statement 1: This ensured the domination of men over women.
Statement 2: Swaymvara was the practice of women choosing their own partners by way of a
competition/selection. When this was given up, women lost an important right. Manusmriti (law
book) further entrenched patriarchal rules to worsen the position of women.
Learning: During the Gupta dynasty period the male members of a family were entitled to inherit
property left behind by their fathers. However, such right of inheritance was not given to women or
girls.
The power of inheritance enabled the male members of a family to exercise dominance over the
women.
Q Source: Page 104: TN 11th Standard History Textbook
TEST - 1 (MOCK TEST - 1)
www.insightsias.com 15
© Insights Active Learning| All rights reserved. You may not reproduce, distribute or exploit the contents in any form without written
permission by copyright owner. Copyright infringers may face civil and criminal liability
12 The year of 2017 marks 25 years of dialogue partnership between India and ASEAN and 15 years of
summit-level interaction. With reference to India-ASEAN relations, consider the following:
1. India's relationship with ASEAN is a key pillar of our Act East Policy.
2. AIFTA is a free trade area among the member states of the ASEAN and India.
3. India and ASEAN are yet to upgrade their relationship to a ‘strategic partnership’.
Select the correct answer using the codes below.
A. 2 only
B. 1 and 3 only
C. 1 and 2 only
D. 3 only
Your Answer : C
Correct Answer : C
Answer Justification :
Justification: Statement 1: India’s commitment to strengthen its ties with ASEAN countries is
reflected by its ‘Look East Policy’ becoming ‘Act East Policy.’ Three Cs are the focus -Commerce,
Connectivity and Culture.
Statement 2: ASEAN became ASEAN Plus Six with additional countries: Australia, New Zealand and
India. Codification of the relations between these nations has seen progress through the
development of the Regional Comprehensive Economic Partnership (RCEP), a proposed free-trade
agreement involving the 16 countries of ASEAN plus six.
Statement 3: India and ASEAN had upgraded their relationship to a strategic partnership in 2012.
ASEAN consists of 10-member countries- Brunei, Cambodia, Indonesia, Laos, Malaysia, Myanmar,
Philippines, Singapore, Thailand and Vietnam.
It is a regional organisation that aims to promote intergovernmental cooperation and economic
integration amongst its members.
Q Source: 25 years of Dialogue partnership between India-ASEAN
TEST - 1 (MOCK TEST - 1)
www.insightsias.com 16
© Insights Active Learning| All rights reserved. You may not reproduce, distribute or exploit the contents in any form without written
permission by copyright owner. Copyright infringers may face civil and criminal liability
13 Universal suffrage (voting rights to all eligible) was granted to women all over India in
A. 1950 once Indian republic was formed
B. Morley-Minto Act
C. Government of India Act, 1935
D. 1939 General elections
Your Answer : B
Correct Answer : A
Answer Justification :
Justification & Learning: First local effort was made in Madras, which granted women’s suffrage
in 1921, but only to those men and women who owned land property according to British
administration's records. This was neither Universal nor equitable.
In 1950, universal suffrage granted voting rights to all women. This is enshrined in Article 326 in
our constitution.
Q Source: Improvisation: Page 32: NCERT Class VI Social and Political Life - I
TEST - 1 (MOCK TEST - 1)
www.insightsias.com 17
© Insights Active Learning| All rights reserved. You may not reproduce, distribute or exploit the contents in any form without written
permission by copyright owner. Copyright infringers may face civil and criminal liability
14 Karla is one of the most famous centres of early rock-cut architecture and has received much
attention of scholars and commoners alike. What is important about Karla caves?
1. The grand Chaitya griha of Karla is the largest of all the chaita-grihas of India.
2. The rock-cut cave was completed without accepting any financial donation from individuals apart
from their voluntary labour.
Which of the above is/are correct?
A. 1 only
B. 2 only
C. Both 1 and 2
D. None
Your Answer :
Correct Answer : A
Answer Justification :
Justification: Statement 1: The chaityagriha at Karla is the biggest of its type in the whole of India.
The hall measures around thirty eight m deep from door to back. The object of worship is the stupa
at the rear end of the chaityagriha.
Statement 2: The caves were caused from the donations and support of a group of assorted
individuals. This includes a prince of Maharathi family; merchants, monks etc.
The inscriptions of Usavadata and Vasisthiputra Pulumavi mention donation of land to the Veluraka
Samgha for this.
Q Source: Page 105: NCERT Class VI Our Pasts - I
TEST - 1 (MOCK TEST - 1)
www.insightsias.com 18
© Insights Active Learning| All rights reserved. You may not reproduce, distribute or exploit the contents in any form without written
permission by copyright owner. Copyright infringers may face civil and criminal liability
15 With reference to the National Disaster Response Force (NDRF), consider the following statements:
1. President of India serves as the Director General (DG) of NDRF.
2. NDRF was established under the Disaster Management Act, 2005.
3. It is housed directly under the Prime Minister’s Office and functions from there.
Select the correct answer using the codes below.
A. 2 only
B. 1 and 3 only
C. 1 only
D. 2 and 3 only
Your Answer : D
Correct Answer : A
Answer Justification :
Justification: NDRF is India’s elite disaster mitigation combat force established under Disaster
Management Act, 2005
Statement 1: Senior IPS officer Sanjay Kumar was recently appointed as Director General (DG) of
National Disaster Response Force (NDRF).
Statement 3: It functions under Union Ministry of Home Affairs. It is mandated to undertake special
disaster response, relief, rescue operations and combat roles independently in the case of an event
of any disaster (natural or man-made), accident or emergency.
It also assists local authorities in launching a quick rescue and response operation to save life and
property.
Q Source:
http://www.thehindu.com/news/national/andhra-pradesh/ndrf-ready-to-tackle-emergencies-in-monso
on-says-its-commandant/article18958062.ece
TEST - 1 (MOCK TEST - 1)
www.insightsias.com 19
© Insights Active Learning| All rights reserved. You may not reproduce, distribute or exploit the contents in any form without written
permission by copyright owner. Copyright infringers may face civil and criminal liability
16 With reference to Ancient India, Ashtangasamgraha written by Vagbhata deals with
A. Astrology
B. Medicine
C. Law
D. Economics
Your Answer : B
Correct Answer : B
Answer Justification :
Learning: In the field of medicine, Vagbhata lived during this period. He was the last of the great
medical trio of ancient India.
The other two scholars Charaka and Susruta lived before the Gupta age. Vagbhata was the
author Ashtangasamgraha (Summary of the eight branches of medicine).
The Sushruta and Charaka texts differ in one major aspect, with Sushruta Samhita providing
the foundation of surgery, while Charaka Samhita being primarily a foundation of medicine.
Q Source: Page 109: TN 11th Standard History Textbook
TEST - 1 (MOCK TEST - 1)
www.insightsias.com 20
© Insights Active Learning| All rights reserved. You may not reproduce, distribute or exploit the contents in any form without written
permission by copyright owner. Copyright infringers may face civil and criminal liability
17 Which of these is/are mandatory qualification(s) to contest for the office of Vice-President of India?
1. She must be a citizen of India or a person of Indian Origin (PIO) residing in India since last ten
years.
2. She must have completed 35 years of age.
3. She must not hold an office for profit.
4. She must have served as a Member of Parliament (MP) or a member of Legislative Assembly in
India.
Select the correct answer using the codes below.
A. 2 and 3 only
B. 2, 3 and 4 only
C. 1 and 4 only
D. 3 only
Your Answer : A
Correct Answer : A
Answer Justification :
Justification: As per article 66, the candidate contesting for election of Vice-President of India
should be citizen of India, completed age of 35 years and must be qualified to become a member of
Rajya Sabha. He must not hold an office for profit.
Her main function is in the form of ex-officio Chairman of Rajya Sabha. She works as acting
president when president is not available.
Learning: Like Presidential election, Vice Presidential election is also indirect and is held via
system of proportional representation by means of the single transferable vote.
The electoral college of Vice President includes elected by elected members of Lok Sabha and
Rajya Sabha.
Unlike in Presidential election, MLAs or MLCs from state assemblies have no role to play in
Vice Presidential election.
We’ll explore this better when covering Laxmikanth.
Q Source: ECI has issued a notification for the process of filing nominations for the vicepresidential
election - Incumbent Vice President Hamid Ansari’s term comes to an end on August
10, 2017.
TEST - 1 (MOCK TEST - 1)
www.insightsias.com 21
© Insights Active Learning| All rights reserved. You may not reproduce, distribute or exploit the contents in any form without written
permission by copyright owner. Copyright infringers may face civil and criminal liability
18 Under the 2007 India-Bhutan Friendship Treaty
1. Neither Government will allow the use of its territory for activities harmful to the national security
and interest of the other
2. Both governments allow free trade and commerce between the territories of Bhutan and India.
3. Bhutanese subjects residing in Indian territories shall have equal justice with Indian subjects.
4. Government of Bhutan is free to export those arms, for commercial purposes, that it received via
imports through the territory of India.
Select the correct answer using the codes below.
A. 2 and 4 only
B. 1, 2 and 3 only
C. 1 and 3 only
D. 1, 2, 3 and 4
Your Answer : C
Correct Answer : B
Answer Justification :
Background: In June 2017, Chinese Army (People’s Liberation Army) construction party had
entered the Doklam area and attempted to construct a road. Royal Bhutan Army (RBA) patrol had
attempted to dissuade them from this unilateral activity.
Since these developments, India and Bhutan have been in continuous contact, giving a view of
India’s role in Bhutan’s security and matters of mutual interest.
The present standoff between Indian and Chinese troops in Doklam (located at the tri-junction
of India, China and Bhutan) has highlighted India’s very special relationship with Bhutan,
including military responsibilities.
Justification: The 2007 treaty is meant to respect the sensitivities of Bhutan regarding its
sovereignty.
Statement 1: This is to avoid Chinese intrusion from the soil of Bhutan.
Statement 2 and 3: You can read the treaty (hardly 2 pages) here
https://mea.gov.in/Images/pdf/india-bhutan-treaty-07.pdf
Statement 4: Government of Bhutan shall be free to import, from or through India into Bhutan,
whatever arms, ammunition etc as may be required for Bhutan, and that this arrangement shall
hold good for all time as long as the Government of India is satisfied that the intentions of the
Government of Bhutan are friendly and that there is no danger to India from such importations.
However, there shall be no export of such arms, ammunition and materials outside Bhutan either by
the Government of Bhutan or by private individuals. So, 4 is wrong.
Q Source: As mentioned in the background
TEST - 1 (MOCK TEST - 1)
www.insightsias.com 22
© Insights Active Learning| All rights reserved. You may not reproduce, distribute or exploit the contents in any form without written
permission by copyright owner. Copyright infringers may face civil and criminal liability
19 Sunderbans delta is facing potential threats due to
1. Global warming induced sea level rise leading to coastal erosion
2. Freshwater flows from Brahmaputra and Meghna
Which of the above is/are correct?
A. 1 only
B. 2 only
C. Both 1 and 2
D. None
Your Answer : A
Correct Answer : A
Answer Justification :
Justification: Sundarben delta hosts a large population of mangorves (that grow in salt water) - the
largest tidal halophytic mangrove forest in the world.
Statement 1: A study highlights a time series of the erosion of many mangrove forested islands of
the Indian Sunderbans from 1986 to 2012.
These islands include Sagar, Gosaba, Dulibhasani, Dalhousie, Bhangaduni and Jambudwip.
The mean sea level rise is considered a driving factor for coastal erosion, coastal flooding,
increase in the number of tidal creeks and losing land.
Statement 2: A critical minimal inflow of freshwater is necessary for the luxuriant growth of
mangroves.
Decrease in freshwater inflow results change in mangrove succession where freshwater loving
species of mangroves are replaced by salt-water loving ones. So, 2 is wrong.
The loss land, including mangrove forest is mainly due to decrease in fresh water flow and
sediment supply in the western (Indian) part of the delta, and the rate of sea level rise is
higher than sediment supply.
However, the eastern (Bangladesh) side of the Sunderbans delta is gaining land because of
the huge amount of sediment and water flow from the Brahmaputra and Meghna rivers
(which is good for the Mangroves).
This immediate impact of salinity will also have negative impact on the fishing community, as
commercially sought fish species will be replaced by fish that does not have as much market
value.
Q Source:
TEST - 1 (MOCK TEST - 1)
www.insightsias.com 23
© Insights Active Learning| All rights reserved. You may not reproduce, distribute or exploit the contents in any form without written
permission by copyright owner. Copyright infringers may face civil and criminal liability
http://www.thehindu.com/sci-tech/energy-and-environment/climate-change-impact-sunderbans-stea
dily-losing-its-famed-mangroves/article19195229.ece
TEST - 1 (MOCK TEST - 1)
www.insightsias.com 24
© Insights Active Learning| All rights reserved. You may not reproduce, distribute or exploit the contents in any form without written
permission by copyright owner. Copyright infringers may face civil and criminal liability
20 What is the usefulness of Double Asteroid Redirection Test (DART) being developed by NASA?
1. It will help protect earth from potential cosmic body impacts in the future.
2. It will lead to the establishment of the first human space station on an asteroid.
Which of the above is/are correct?
A. 1 only
B. 2 only
C. Both 1 and 2
D. None
Your Answer : D
Correct Answer : A
Answer Justification :
Justification: This is the first-ever mission that will deflect a near-Earth asteroid.
NASA will use kinetic impactor technique to deflect them.
This technique involves sending one or more large, high-speed spacecraft into the path of an
approaching near-earth object to shift its orbit to defend against future impact.
It works by changing the speed of a threatening asteroid by a small fraction of its total
velocity.
This is done well before the predicted impact so that this small velocity adds up over time to a
big shift of the asteroid’s path away from Earth
Q Source: https://www.nasa.gov/planetarydefense/aida/
TEST - 1 (MOCK TEST - 1)
www.insightsias.com 25
© Insights Active Learning| All rights reserved. You may not reproduce, distribute or exploit the contents in any form without written
permission by copyright owner. Copyright infringers may face civil and criminal liability
21 He was a follower of Jainism in the early part of his career and converted to Saivism later. He is
hailed as ‘Vichitrachitta’ who constructed a temple for Brahma, Vishnu and Siva without the use of
bricks, timber, metal and mortar. Who is the ruler?
A. Kadambas of Banavasi
B. Adikavi Pampa
C. Rajaraja I
D. Mahendravarman I
Your Answer :
Correct Answer : D
Answer Justification :
Learning: Mahendravarman I was converted to Saivism which led him to built a Siva temple at
Tiruvadi.
He was a great builder of cave temples. The Mandagappattu inscription hails him as
Vichitrachitta who constructed a temple for Brahma, Vishnu and Siva without the use of
bricks, timber, metal and mortar.
His rock-cut temples are found in a number of places like Tiruchirappalli.
His title Chitrakarapuli reveals his talents in painting. He is also regarded as an expert in
music.
The music inscription at Kudumianmalai is ascribed to him.
Q Source: Page 124: TN 11th Standard History Textbook
TEST - 1 (MOCK TEST - 1)
www.insightsias.com 26
© Insights Active Learning| All rights reserved. You may not reproduce, distribute or exploit the contents in any form without written
permission by copyright owner. Copyright infringers may face civil and criminal liability
22 Which of these justify the robust and benevolent system of administration that Cholas had?
1. Chola kings undertook royal tours to increase the efficiency of the administration.
2. All legal professions were exempt from tax liabilities.
3. All lands were carefully surveyed and classified for assessment of revenue.
4. During the hard times, there were remission of taxes.
Select the correct answer using the codes below.
A. 1 and 4 only
B. 1, 3 and 4 only
C. 2 and 3 only
D. 2 and 4 only
Your Answer : B
Correct Answer : B
Answer Justification :
Justification: Statement 1: There were regular drills with officials.
Statement 2 and 3: There were tolls and customs on goods taken from one place to another, various
kinds of professional taxes, dues levied on ceremonial occasions like marriages and judicial fines.
Moreover, the land revenue department was well organized. It was called as
puravuvarithinaikkalam.
The residential portion of the village was called ur nattam. These and other lands such as the lands
belonging to temples were exempted from tax.
Statement 4: During the hard times, there were remission of taxes and Kulottunga I became famous
by abolishing tolls and earned the title – Sungam Tavirtta Cholan.
The main items of government expenditure were the king and his court, army and navy, roads,
irrigation tanks and canals.
Q Source: Page 146: TN 11th Standard History Textbook
TEST - 1 (MOCK TEST - 1)
www.insightsias.com 27
© Insights Active Learning| All rights reserved. You may not reproduce, distribute or exploit the contents in any form without written
permission by copyright owner. Copyright infringers may face civil and criminal liability
23 What is the significance of Sabarmati Ashram in the modern history of India?
1. It served as the last residence of Mahatma Gandhi.
2. It has been declared as a national monument by the Indian government.
3. Salt Satyagraha was started from this Ashram in 1930.
Select the correct answer using the codes below.
A. 1 only
B. 2 and 3 only
C. 3 only
D. 1 and 3 only
Your Answer : B
Correct Answer : B
Answer Justification :
Justification: The Sabarmati Ashram (also known as Harijan or Satyagraha Ashram) was home to
Mohandas Gandhi from 1917 until 1930 and served as one of the main centres of the Indian
freedom struggle.
Statement 1: This was Sevagram cottage (not Sabarmati Ashram). Gandhiji left to Delhi from
Sevargam Cottage in 1946 and then went to Noakhali. From there he did not return to Sewagram
while engaged in his task of communal unity and peace making. He was martyred in Delhi in 1948.
Statement 2 and 3: It is common knowledge.
Learning: On his return from South Africa, Gandhi’s first Ashram in India was established in the
Kochrab area of Ahmedabad in 1915. The Ashram was then shifted in 1917 to a piece of open
barren land on the banks of the river Sabarmati.
Reasons for this shift included: he wanted to do some experiments in living eg farming, animal
husbandry, cow breeding, Khadi and related constructive activities, for which he was in
search of this kind of barren land.
The Prime Minister has recently released the coin and postal stamp in the honour of Shrimad
Rajchandraji on his 150th birth anniversary at Abhay Ghat. Shrimad Rajchandraji was the
guru to Mahatma Gandhi.
Q Source: PM recently inaugurated the centenary celebrations of Sabarmati Ashram in Ahmedabad
TEST - 1 (MOCK TEST - 1)
www.insightsias.com 28
© Insights Active Learning| All rights reserved. You may not reproduce, distribute or exploit the contents in any form without written
permission by copyright owner. Copyright infringers may face civil and criminal liability
24 Which of these systems were popularly introduced by Alauddin Khalji?
1. Branding of horses
2. Maintaining a descriptive list of soldiers
3. Paying army in kind rather than cash
4. Market based pricing for all products
Select the correct answer using the codes below.
A. 1 and 2 only
B. 3 and 4 only
C. 2 and 4 only
D. 1 and 3 only
Your Answer :
Correct Answer : A
Answer Justification :
Justification: Statement 1, 2 and 3: Alauddin Khalji maintained a large permanent standing army
and paid them in cash from the royal treasury.
He introduced the system of dagh (branding of horses) and prepared huliya (descriptive list of
soldiers).
In order to ensure maximum efficiency, a strict review of army from time to time was carried
out.
Statement 4: The introduction of paying salaries in cash to the soldiers led to price regulations
popularly called as Market Reforms.
Markets were under the control of a high officer called Shahna-i- Mandi. The supply of grain was
ensured by holding stocks in government store-houses.
Regulations were issued to fix the price of all commodities.
Learning: Every merchant was registered under the Market department. There were secret agents
called munhiyans who sent reports to the Sultan regarding the functioning of these markets.
Violation of regulations was severely punished. Harsh punishment was given if any shopkeeper
charged a higher price, or tried to cheat by using false weights and measures. Even during the
famine the same price was maintained
Q Source: Page 180: TN 11th Standard History Textbook
TEST - 1 (MOCK TEST - 1)
www.insightsias.com 29
© Insights Active Learning| All rights reserved. You may not reproduce, distribute or exploit the contents in any form without written
permission by copyright owner. Copyright infringers may face civil and criminal liability
25 Consider the following statements:
1. Under the Participatory Guarantee System for India (PGS-India), FAO bureaucracy certifies organic
products after due inspection.
2. National Programme for Organic Production (NPOP) intends to facilitate certification of organic
products in conformity with the importing countries organic standards.
Which of the above is/are correct?
A. 1 only
B. 2 only
C. Both 1 and 2
D. None
Your Answer :
Correct Answer : B
Answer Justification :
Background: At present, the consumers do not have any method to check the authenticity of the
organic food products due to the absence of a regulatory framework.
FSSAI has come out with draft regulations for organic food products to ensure the
authenticity of organic foods.
FSSAI’s regulation mandates that labelling of organic foods should provide full and accurate
information regarding the organic status of the organic foods.
Organic food products must carry a certification mark or a quality assurance mark provided
by the notified certification bodies.
Justification: Statement 1: Organic foods will have to comply with the government’s provisions
under the National Programme for Organic Production (NPOP) or the Participatory Guarantee
System for India (PGS-India) run by the Agriculture Ministry or any other standards notified by
FSSAI.
The standards of Organic Crop Production have been described in NPOP document.
The production standards include crop production plan, conversion requirement, nutrient
management, disease management etc for agricultural crops.
Details will be covered in I-Learning (new initiative on website).
Statement 2: The PGS is an internationally applicable organic quality assurance system [like ISO
9000] implemented and controlled by the committed organic farmer-producers through active
participation, along with the consumers, in the process based on verifiable trust.
TEST - 1 (MOCK TEST - 1)
www.insightsias.com 30
© Insights Active Learning| All rights reserved. You may not reproduce, distribute or exploit the contents in any form without written
permission by copyright owner. Copyright infringers may face civil and criminal liability
It is not an “inspection raj” certification system but, rather, one that is based on personal integrity
and peer pressure. Integrity is honesty when no one is looking over your shoulder to see what you
are doing. So, 1 is wrong.
http://pgsindia-ncof.gov.in/pgs_india.aspx
Q Source: http://pib.nic.in/newsite/PrintRelease.aspx?relid=155000
http://www.thehindubusinessline.com/news/fssai-issues-draft-regulations-for-organic-food-products/
article9737535.ece
TEST - 1 (MOCK TEST - 1)
www.insightsias.com 31
© Insights Active Learning| All rights reserved. You may not reproduce, distribute or exploit the contents in any form without written
permission by copyright owner. Copyright infringers may face civil and criminal liability
26 Consider the following statements. Assertion (A): Kushanas banned the use of gold coins taken from
their empire on the silk route. Reason (R): Kushanas could not control the Silk route and were at a
loss from its trade.
In the context of the above, which of these is correct?
A. A is correct, and R is an appropriate explanation of A.
B. A is correct, but R is not an appropriate explanation of A.
C. A is correct, but R is incorrect.
D. Both A and R are incorrect.
Your Answer : D
Correct Answer : D
Answer Justification :
Justification: The best-known of the rulers who controlled the Silk Route were the Kushanas, who
ruled over central Asia and north-west India around 2000 years ago.
Their two major centres of power were Peshawar and Mathura.
During their rule, a branch of the Silk Route extended from Central Asia down to the seaports at the
mouth of the river Indus, from where silk was shipped westwards to the Roman Empire.
The Kushanas were amongst the earliest rulers of the subcontinent to issue gold. So, both A and R
are wrong.
Q Source: Page 102: NCERT Class VI Our Pasts - I
TEST - 1 (MOCK TEST - 1)
www.insightsias.com 32
© Insights Active Learning| All rights reserved. You may not reproduce, distribute or exploit the contents in any form without written
permission by copyright owner. Copyright infringers may face civil and criminal liability
27 Consider the following about ‘Know India Programme (KIP)’:
1. KIP is a programme run jointly by IISC, IITs and NITs to inculcate scientific temper and general
knowledge in youth.
2. KIP involves compulsory government financed field visits in public schools to places of national
importance.
Which of the above is/are correct?
A. 1 only
B. 2 only
C. Both 1 and 2
D. None
Your Answer : B
Correct Answer : D
Answer Justification :
Justification: Know India Programme is an initiative of the Ministry of External Affairs for the
Persons of Indian Origin (PIO) (including NRIs) between the age group of 18 to 30 years to provide
them an exposure to the country of their origin (India).
The KIP offers a platform for the young PIOs to visit India share their views, expectations and
experiences and forge closer bonds with the India of present times in a three-week orientation
programme.
External Affairs Minister has recently launched a portal for the Know India Programme (KIP).
Q Source: http://www.mea.gov.in/know-india-programme.htm
TEST - 1 (MOCK TEST - 1)
www.insightsias.com 33
© Insights Active Learning| All rights reserved. You may not reproduce, distribute or exploit the contents in any form without written
permission by copyright owner. Copyright infringers may face civil and criminal liability
28 Which of the two key principles/themes of democracy are inseparable or can’t exist without each
other?
A. Equity and Justice
B. Fraternity and Sovereignty
C. Accountability and transparency
D. Both (a) and (c)
Your Answer : A
Correct Answer : D
Answer Justification :
Justification: Option A: Justice implies fairness, it is a cardinal value of democracy. Equity implies
impartiality. Take Article 14 of constitution that ensures rule of law: it ensures equality between
citizens against law, that leads to justice for all.
Option B: There is no direct correlation between both.
Option C: You can take RTI as an example. If the government isn’t transparent on information, it
can’t be held accountable. On the other hand, if there is no accountability, it will be most likely due
to opacity in functioning of government. Both go hand in hand.
Q Source: Improvisation: Page 40: NCERT Class VI Social and Political Life - I
TEST - 1 (MOCK TEST - 1)
www.insightsias.com 34
© Insights Active Learning| All rights reserved. You may not reproduce, distribute or exploit the contents in any form without written
permission by copyright owner. Copyright infringers may face civil and criminal liability
29 Ramananda, Kabir and Nanak
1. Were not linked with any particular religious creed
2. Did not believe in rituals and ceremonies
3. Condemned monotheism and believed in multiple Gods
4. Denounced all forms of idolatry
Select the correct answer using the codes below.
A. 1, 2 and 3 only
B. 1, 2 and 4 only
C. 3 and 4 only
D. 1 and 3 only
Your Answer : B
Correct Answer : B
Answer Justification :
Justification: Statement 1: In the fourteenth and fifteenth centuries, Ramananda, Kabir and Nanak
remained great apostles of the Bhakti cult.
They drew inspiration from old masters but showed a new path.
They helped the common people to shed age-old superstitions and attain salvation through
Bhakti or pure devotion.
Statement 2, 3 and 4: They condemned polytheism and believed in one god. They also denounced all
forms of idolatry. They strongly believed in Bhakti as the only means of salvation. They also
emphasised the fundamental unity of all religions.
Unlike the early reformers, they were not linked with any particular religious creed and did not
believe in rituals and ceremonies.
Q Source: Page 203: TN 11th Standard History Textbook
TEST - 1 (MOCK TEST - 1)
www.insightsias.com 35
© Insights Active Learning| All rights reserved. You may not reproduce, distribute or exploit the contents in any form without written
permission by copyright owner. Copyright infringers may face civil and criminal liability
30 Over the last decade, the country that is the World’s largest receiver of remittances is
A. USA
B. India
C. Russia
D. Canada
Your Answer : B
Correct Answer : B
Answer Justification :
Learning: According to UN International Fund for Agricultural Development (IFAD), Indians
working across the world had sent home USD 62.7 billion in 2016 making India the top remittancereceiving
country surpassing China.
The study is the first ever study conducted that analysed a 10-year trend in migration and
remittance flows over the period 2007-2016.
In the period between 2007 and 2016, India has surpassed China to become the world’s
largest receiver of remittances.
Asia remains the main remittance-receiving region with 55% of the global flows.
This increase in remittances is expected to lift millions out of poverty and in attaining the
Sustainable Development Goals (SDGs).
Top Remittance Receiving Countries: India (USD 62.7 billion), China (USD 61 billion), the
Philippines (USD 30 billion) and Pakistan (USD 20 billion).
Q Source: IFAD study as mentioned above
TEST - 1 (MOCK TEST - 1)
www.insightsias.com 36
© Insights Active Learning| All rights reserved. You may not reproduce, distribute or exploit the contents in any form without written
permission by copyright owner. Copyright infringers may face civil and criminal liability
31 Bhimbetka, in present-day Madhya Pradesh, is famous for
A. Hosting prehistoric cave paintings
B. Largest Buddha Bronze statues
C. Monolithic temples
D. None of the above
Your Answer : A
Correct Answer : A
Answer Justification :
Learning: Bhimbetka rock shelters are an archaeological site of the Paleolithic, exhibiting the
earliest traces of human life on the Indian Subcontinent, and thus the beginning of the Indian Stone
Age.
Some of the Stone Age rock paintings found among the Bhimbetka rock shelters are some 30,000
years old. The caves also deliver early evidence of dance. They were declared a World Heritage Site
in 2003.
Q Source: Page 15: NCERT Class VI Our Pasts - I
TEST - 1 (MOCK TEST - 1)
www.insightsias.com 37
© Insights Active Learning| All rights reserved. You may not reproduce, distribute or exploit the contents in any form without written
permission by copyright owner. Copyright infringers may face civil and criminal liability
32 Which of these types of trading/agreements in securities markets is NOT permitted in India?
A. Futures contract
B. Options contract
C. Forward contract
D. All are permitted
Your Answer : B
Correct Answer : D
Answer Justification :
Justification: Option A: Futures Contract has been permitted long back. It is a contract between
two parties where both parties agree to purchase or sell a particular commodity or any other
financial instrument at a predetermined price at a specified time in the future.
Option B: Commodity Exchanges have been requesting SEBI for a long time to permit options
trading in commodities be allowed. It was permitted recently.
Options Contract is a derivative product that offers an investor the right to purchase without
any obligation to buy at the specified price/date.
Suppose you want your dream car which as of now costs RM 100,000 (RM is a currency unit).
You don’t have the money to buy the car now, but you will have after 3 months.
A smart move would be to buy an “option” to purchase this car in the future (after 3 months).
You pay the car owner a contract premium (say RM 1,000) to exercise this option.
Remember, you have the option to buy or not buy the car.
See the diagram below now. Suppose after 3 months, car prices increase to RM 150,000. This
is a win case for you, since the options contract was to buy it at RM 100,000. You gain.
Suppose car price reduces. Now, you won’t buy the car, because the contract was not an
obligation (only a right) to buy the car. You would now pay the options premium (RM 1000 to
the owner), and may be buy the car from the market.
TEST - 1 (MOCK TEST - 1)
www.insightsias.com 38
© Insights Active Learning| All rights reserved. You may not reproduce, distribute or exploit the contents in any form without written
permission by copyright owner. Copyright infringers may face civil and criminal liability
Option C: You would heard of the Forward Markets Commission that earlier regulated
forward markets. Now SEBI has subsumed the organization.
Q Source:
http://www.livemint.com/Money/nfnDOuk11PlqWI43mgA1BL/Sebi-permits-options-trading-incommodity-
futures.html
TEST - 1 (MOCK TEST - 1)
www.insightsias.com 39
© Insights Active Learning| All rights reserved. You may not reproduce, distribute or exploit the contents in any form without written
permission by copyright owner. Copyright infringers may face civil and criminal liability
33 Which of these earliest cities was located to the east of Indus river?
1. Dholavira
2. Rakhi Garhi
3. Kalibangan
4. Sotkakoh
Select the correct answer using the codes below.
A. 1, 2 and 3 only
B. 2 and 4 only
C. 1 and 3 only
D. 1, 2, 3 and 4
Your Answer : B
Correct Answer : A
Answer Justification :
Justification:
Learning: These cities were found in the Punjab and Sind in Pakistan, and in Gujarat, Rajasthan,
Haryana and the Punjab in India.
Archaeologists have found a set of unique objects in almost all these cities: red pottery
painted with designs in black, stone weights, seals, special beads, copper tools, and long
stone blades.
TEST - 1 (MOCK TEST - 1)
www.insightsias.com 40
© Insights Active Learning| All rights reserved. You may not reproduce, distribute or exploit the contents in any form without written
permission by copyright owner. Copyright infringers may face civil and criminal liability
Cities such as Kalibangan and Lothal had fire altars, where sacrifices may have been
performed.
In some cities, like Mohenjodaro, a very special tank, which archaeologists call the Great
Bath, was found.
Q Source: Page 33: NCERT Class VI Our Pasts - I
TEST - 1 (MOCK TEST - 1)
www.insightsias.com 41
© Insights Active Learning| All rights reserved. You may not reproduce, distribute or exploit the contents in any form without written
permission by copyright owner. Copyright infringers may face civil and criminal liability
34 The Ministry of Law and Justice has launched the ‘Tele-Law’ initiative in collaboration with the
Ministry of Electronics and Information Technology. Consider the following with reference to it.
1. It would subsume National Legal Service Authority (NALSA) and its State Level authorities to
create a Virtual Legal Service Authority (VLSA).
2. It will provide legal aid services to the marginalized communities and citizens living in rural areas.
3. It would help people to seek legal advice from lawyers with the help of video conferencing facility.
Select the correct answer using the codes below.
A. 1 and 2 only
B. 2 and 3 only
C. 3 only
D. 1, 2 and 3
Your Answer :
Correct Answer : B
Answer Justification :
Justification: Statement 2: Under the initiative, a Para Legal Volunteer (PLV), would be employed
in each Common Service Centres (CSC – which are IT service points in blocks established with
government support and regulation).
The volunteer would serve as the first point of contact for the marginalized communities in
the rural areas who will help them understand the legal issues, explain the advice given by
lawyers and assist in further action as per the advice of the lawyer.
Around 1000 women PLVs will also be trained under this initiative to provide legal aid
services through the CSCs.
Statement 3: The National Legal Service Authority (NALSA) will also provide a panel of lawyers who
will provide advice from the state capitals to the applicants in the CSCs through video conferencing.
Learning: Apart from the lawyers, law school clinics, District Legal Service Authorities, voluntary
service providers and Non-Government Organisations working on legal aid and empowerment can
also be connected through the CSCs.
In the initial phase, the initiative will be tested as a pilot project across 500 Common Service
Centres (CSC) in Uttar Pradesh and Bihar.
The government will also put in place a robust monitoring and evaluation system to assess the
quality of the legal advice delivered to the people.
Q Source:
http://www.thehindu.com/news/national/india-launches-free-legal-aid-via-video-conferencing-to-villa
gers/article18970545.ece
TEST - 1 (MOCK TEST - 1)
www.insightsias.com 42
© Insights Active Learning| All rights reserved. You may not reproduce, distribute or exploit the contents in any form without written
permission by copyright owner. Copyright infringers may face civil and criminal liability
35 The Indian constitution prohibits discrimination between individuals on the basis of
A. Gender and caste only
B. Gender, caste, religion, race and place of birth
C. Gender, education, caste and place of birth
D. Caste, education, religion and race only
Your Answer : B
Correct Answer : B
Answer Justification :
Learning: These provisions come under the Right to Equality under fundamental rights.
The State shall not discriminate against any citizen on grounds only of religion, race, caste, sex,
place of birth or any of them.
No citizen shall, on grounds only of religion, race, caste, sex, place of birth or any of them, be
subject to any disability, liability, restriction or condition with regard to
access to shops, public restaurants, hotels and places of public entertainment; or
the use of wells, tanks, bathing ghats, roads and places of public resort maintained wholly or
partly out of State funds or dedicated to the use of the general public.
Nothing in this article shall prevent the State from making any special provision for women,
children, SCs or STs.
Q Source: Improvisation: Page 20: NCERT Class VI Social and Political Life - I
TEST - 1 (MOCK TEST - 1)
www.insightsias.com 43
© Insights Active Learning| All rights reserved. You may not reproduce, distribute or exploit the contents in any form without written
permission by copyright owner. Copyright infringers may face civil and criminal liability
36 With reference to Ancient Southern India, Tolkappiyam refers to the five-fold division of lands.
Consider the following matches with their description.
1. Kurinji: Desert
2. Mullai: Pastoral
3. Neydal: Agricultural
4. Marudam: Hilly tracks
5. Palai: Coastal
Select the correct matches using the codes below.
A. 1, 3 and 4 only
B. 2 only
C. 1, 4 and 5 only
D. 1, 2, 3, 4 and 5
Your Answer :
Correct Answer : B
Answer Justification :
Justification: This was the description: Kurinji (hilly tracks), Mullai (pastoral), Marudam
(agricultural), Neydal (coastal) and Palai (desert).
Learning: The people living in these five divisions had their respective chief occupations as well as
gods for worship.
Kurinji – chief deity was Murugan – chief occupation, hunting and honey collection.
Mullai – chief deity Mayon (Vishnu) – chief occupation, cattle-rearing and dealing with dairy
products.
Marudam – chief deity Indira – chief occupation, agriculture.
Neydal – chief deity Varunan – chief occupation fishing and salt manufacturing.
Palai – chief deity Korravai – chief occupation robbery.
Q Source: Page 90: TN 11th Standard History Textbook
TEST - 1 (MOCK TEST - 1)
www.insightsias.com 44
© Insights Active Learning| All rights reserved. You may not reproduce, distribute or exploit the contents in any form without written
permission by copyright owner. Copyright infringers may face civil and criminal liability
37 “Kanishka must have conquered the greater part of the Gangetic plain”, the evidence of this
statement comes from
1. Coins of Kanishka are found in places like Mathura, Sravasti and Benares.
2. His coins exhibit only Buddhist images, and not Hindu gods, a cult that was popular then in the
Gangetic plain.
Which of the above is/are correct?
A. 1 only
B. 2 only
C. Both 1 and 2
D. None
Your Answer : A
Correct Answer : A
Answer Justification :
Justification: Statement 1: At the time of his accession his empire included Afghanistan, Gandhara,
Sind and Punjab. Subsequently he conquered Magadha and extended his power as far as Pataliputra
and Bodh Gaya. This evidence comes from his coins that are found in the gangetic plains.
Statement 2: Kanishka embraced Buddhism in the early part of his reign. However, his coins exhibit
the images of not only Buddha but also Greek and Hindu gods. So, 2 is wrong.
Q Source: Page 79: TN 11th Standard History Textbook
TEST - 1 (MOCK TEST - 1)
www.insightsias.com 45
© Insights Active Learning| All rights reserved. You may not reproduce, distribute or exploit the contents in any form without written
permission by copyright owner. Copyright infringers may face civil and criminal liability
38 The Government has issued Public Procurement (Preference to Make in India), Order 2017 recently.
Consider the following with reference to its provisions:
1. The order applies to manufacturing sector only.
2. Ordinarily, all government procurement shall be based on 100% local content sourcing.
3. Import of non-essential commodities shall be severely regulated to promote domestic
manufacturing.
Select the correct answer using the codes below.
A. 1 only
B. 2 and 3 only
C. 3 only
D. None of the above
Your Answer : C
Correct Answer : D
Answer Justification :
Justification: This policy promotes manufacturing and production of goods and services in India
with a view to enhancing income and employment. Since services are also covered, S1 is wrong.
In pursuance of this Order, purchase preference shall be given to local suppliers in all
procurements undertaken by procuring entities in the manner specified in the order.
As per the order the minimum local content shall ordinarily be 50%. The Nodal Ministry may
prescribe a higher or lower percentage in respect of any particular item and may also
prescribe the manner of calculation of local content.
Q Source: http://dipp.nic.in/sites/default/files/publicProcurement_MakeinIndia_15June2017.pdf
TEST - 1 (MOCK TEST - 1)
www.insightsias.com 46
© Insights Active Learning| All rights reserved. You may not reproduce, distribute or exploit the contents in any form without written
permission by copyright owner. Copyright infringers may face civil and criminal liability
39 An important item of trade from Ancient India, it was highly valuable in Roman Empire so much that
it was known as ‘black gold’. What does the item refer to?
A. Crude oil
B. Coal
C. Dark Garnet stones
D. Pepper
Your Answer : A
Correct Answer : D
Answer Justification :
Learning: Frequently referred to as "black gold" in ancient India, on account of its demand and
trade value mainly due to its high trade value, the Zamorin ruler of Malabar coast had a flourishing
trade centre for export of pepper and other spices in Kerala.
The main mercantile traders were Arabs and people from Middle East.
The early Roman Empire got direct access to the Malabar Coast in India and its range of
exotic spices after their conquest of Egypt in 30 BC.
The prices of pepper were extremely high in the Middle Ages and the trade was completely
dominated by the Romans.
Q Source: Page 99: NCERT Class VI Our Pasts - I
TEST - 1 (MOCK TEST - 1)
www.insightsias.com 47
© Insights Active Learning| All rights reserved. You may not reproduce, distribute or exploit the contents in any form without written
permission by copyright owner. Copyright infringers may face civil and criminal liability
40 Consider the following:
1. Temple at Deogarh near Jhansi
2. Giant copper statue of Buddha originally found at Sultanganj
3. Delhi Iron pillar
4. Bagh caves paintings
The above belong to periods of which of these dynasties of India?
A. Peshwas
B. Guptas
C. Paramara
D. Western Ganga
Your Answer :
Correct Answer : B
Answer Justification :
Learning: The Delhi Iron pillar of the Gupta period is still free from rust though completely
exposed to sun and rain for so many centuries.
The paintings of the Gupta period are seen at Bagh caves near Gwalior.
Moreover, the mural paintings of Ajantha mostly illustrate the life of the Buddha as depicted in the
Jataka stories.
There was little influence of Gandhara style on Gupta art. But the beautiful statue of standing
Buddha at Mathura reveals a little Greek style. The Buddha statue unearthed at Saranath was
unique piece of Gupta art.
Q Source: Page 106-107: TN 11th Standard History Textbook
TEST - 1 (MOCK TEST - 1)
www.insightsias.com 48
© Insights Active Learning| All rights reserved. You may not reproduce, distribute or exploit the contents in any form without written
permission by copyright owner. Copyright infringers may face civil and criminal liability
41 Under the SMARTGRAM initiative of Rashtrapati (President) Bhavan, what is considered as a ‘smart
gram (village)’?
A. A village that is on the transition to become a municipal area
B. A village where a majority of the population is digitally literate
C. A village having required physical and social infrastructure governed with a layer of smart
information and communication systems
D. A village that is close to sub-urban areas of smart cities
Your Answer : C
Correct Answer : C
Answer Justification :
Learning: Under the project the selected villages will see new ventures in agriculture, skill
development, energy, education, heath, employment generation, and entrepreneurship.
The model is based on the convergence of resources and effort by the central government, state
government, district administration, PRIs, private sector etc. to enhance the environment,
connectivity and wellbeing of each village.
Q Source: http://pib.nic.in/newsite/PrintRelease.aspx?relid=161641
TEST - 1 (MOCK TEST - 1)
www.insightsias.com 49
© Insights Active Learning| All rights reserved. You may not reproduce, distribute or exploit the contents in any form without written
permission by copyright owner. Copyright infringers may face civil and criminal liability
42 Consider the following statistics.
1. India ranks first among the world’s milk producing Nations.
2. Most of the milk produced in India comes from cooperatives.
Which of the above is/are correct?
A. 1 only
B. 2 only
C. Both 1 and 2
D. None
Your Answer : A
Correct Answer : A
Answer Justification :
Justification: Statement 1: India continued to be the largest milk producing nation (since 1998)
with milk production of nearly 155 million tonnes during 2015-16. India also has the largest bovine
population in the World.
About 54% of the milk produced in the country is surplus for marketing in the domestic market, of
which only about 20% is procured/processed by the organised sector being equally, shared by
Cooperatives and Private dairy organizations.
Statement 2: The dairy cooperatives collectively procure around 16 million tonnes of milk.
Learning: Government is mandated to double milk producers’ income at farm level by providing
rural milk producers with greater access to the organised milk processing sector.
Department of Animal Husbandry, Dairying & Fisheries has formulated a Draft National Action Plan
for Dairy Development recently.
Q Source: PIB SS (Release ID :167289)
TEST - 1 (MOCK TEST - 1)
www.insightsias.com 50
© Insights Active Learning| All rights reserved. You may not reproduce, distribute or exploit the contents in any form without written
permission by copyright owner. Copyright infringers may face civil and criminal liability
43 With reference to the Indus Valley civilization, consider the following statements:
1. Indus valley civilization was the biggest exporter of semi-precious stones and a major importer of
agricultural products.
2. Fishing was a regular occupation while hunting and bull fighting were banned.
3. Linga worship was prevalent and even natural figures like trees were worshipped.
4. Lothal was an important point of trade between the Harappan civilization and the remaining part of
India as well as Mesopotamia.
Select the correct answer using the codes below.
A. 1 and 2 only
B. 3 and 4 only
C. 1, 2 and 3 only
D. 2, 3 and 4 only
Your Answer : D
Correct Answer : B
Answer Justification :
Justification: Statement 1: Gold, copper, tin and several semi-precious stones were imported. Main
exports were several agricultural products such as wheat, barely, peas, oil seeds etc.
Statement 2: Hunting and bull fighting were common pastimes, not banned. Marbles, balls and dice
were also used for games.
Statement 3: The chief female deity was the Mother Goddess represented in terracotta figurines. In
latter times, Linga worship was prevalent. Trees and animals were also worshipped by the
Harappans. They also believed in ghosts and evil forces and used amulets as protection against
them.
Statement 4: Lothal is an important Harappan centre, you should read more about it.
Q Source: Page 21-23: TN 11th Standard History Textbook
TEST - 1 (MOCK TEST - 1)
www.insightsias.com 51
© Insights Active Learning| All rights reserved. You may not reproduce, distribute or exploit the contents in any form without written
permission by copyright owner. Copyright infringers may face civil and criminal liability
44 Many Chinese Buddhist pilgrims visited Indian subcontinent to visit places associated with the life of
the Buddha. Who among the following is NOT one of them?
A. Fa Xian
B. Xuan Zang
C. I-Qing
D. Alan Chen
Your Answer :
Correct Answer : D
Answer Justification :
Learning: Three such Chinese Buddhist pilgrims are best-known.
Fa Xian, who came to the subcontinent about 1600 years ago, Xuan Zang (who came around 1400
years ago) and I-Qing, who came about 50 years after Xuan Zang are the famous one.
They have documented previous socio-economic, political and religious details of Indian life then.
We will cover all travellers in detail in coming tests.
Q Source: Page 105: NCERT Class VI Our Pasts - I
TEST - 1 (MOCK TEST - 1)
www.insightsias.com 52
© Insights Active Learning| All rights reserved. You may not reproduce, distribute or exploit the contents in any form without written
permission by copyright owner. Copyright infringers may face civil and criminal liability
45 Consider the following statements.
1. Anyone who is a resident of the concerned village is a member of the Gram Sabha.
2. All plans for the work of the Gram Panchayat are placed before the people in Gram Sabha.
Which of the above is/are correct?
A. 1 only
B. 2 only
C. Both 1 and 2
D. None
Your Answer : B
Correct Answer : B
Answer Justification :
Justification: Statement 1: Only one who is 18 years old or more and who has the right to vote is a
member of the Gram Sabha.
The Gram Sabha is a meeting of all adults who live in the area covered by a Panchayat. This could
be only one village or a few villages.
Statement 2: The Gram Sabha prevents the Panchayat from doing wrong things like misusing
money or favouring certain people.
It plays an important role in keeping an eye on the elected representatives and in making them
responsible to the persons who elected them.
Q Source: Improvisation: Page 43-45: NCERT Class VI Social and Political Life - I
TEST - 1 (MOCK TEST - 1)
www.insightsias.com 53
© Insights Active Learning| All rights reserved. You may not reproduce, distribute or exploit the contents in any form without written
permission by copyright owner. Copyright infringers may face civil and criminal liability
46 With reference to the Vedic civilization, Rajasuya, Asvamedha and Vajpeya were
A. Rituals and sacrifices performed by the King to strengthen his position
B. Mercenary army contingents that were owned by the local landlords
C. Land tax imposed on areas inside the jurisdiction of the Empire
D. Body of officials who maintained a record of the extent of empire, its defeats and successions
Your Answer : A
Correct Answer : A
Answer Justification :
Context: Larger kingdoms were formed during the later Vedic period as many jana or tribes were
amalgamated to form janapadas or rashtras in the later Vedic period.
Hence the royal power had increased along with the increase in the size of kingdom.
Learning: So, the king performed various rituals and sacrifices to strengthen his position. They
include Rajasuya (consecration ceremony), Asvamedha (horse sacrifice) and Vajpeya (chariot race).
The kings also assumed titles like Rajavisvajanan, Ahilabhuvanapathi, (lord of all earth), Ekrat and
Samrat (sole ruler) to showcase his power.
Q Source: Page 32: TN 11th Standard History Textbook
TEST - 1 (MOCK TEST - 1)
www.insightsias.com 54
© Insights Active Learning| All rights reserved. You may not reproduce, distribute or exploit the contents in any form without written
permission by copyright owner. Copyright infringers may face civil and criminal liability
47 Jainism advocates three principles known as Triratnas (three gems) which are right faith, right
knowledge and right conduct. On the other hand, Buddhism preaches four noble truths and the
eightfold path. What is/are common between the teachings of both religions?
1. Acceptance of the theory that God exists and it has multiple aspects
2. Understanding that no object possesses a soul and is inanimate in the larger flow of life
3. To practice ahimsa and abstain from acquiring precious metals like Gold and Silver
4. Penance and extreme austerity to punish the body
Select the correct answer using the codes below.
A. 1 and 2 only
B. 3 only
C. 2 and 3 only
D. 1 and 4 only
Your Answer : B
Correct Answer : B
Answer Justification :
Justification: We will explain it in the light of teachings of Mahavira.
Justification: Statement 1 and 2: Right Knowledge (as per Mahavira) is the acceptance of the
theory that there is no God and that the world has been existing without a creator and that all
objects possess a soul.
Buddhism sheds any discussion on God and soul as futile. Buddha also taught that the soul does not
exist, so there is no meeting ground in this case. So, 1 and 2 are wrong.
Statement 3: Right conduct (Jainism) refers to the observance of the five great vows such as not to
injure life, not to lie, not to steal, not to acquire property and not to lead an immoral life.
Buddhism also forbids its monks from acquiring property (that may bring a sense of ego and pride
to its monks and nuns) and injuring life. So, 3 is a meeting ground.
Statement 4: Only Jainism advocates extreme penance, Buddhism preaches moderation. So, 4 is
wrong.
Q Source: Page 39: TN 11th Standard History Textbook
TEST - 1 (MOCK TEST - 1)
www.insightsias.com 55
© Insights Active Learning| All rights reserved. You may not reproduce, distribute or exploit the contents in any form without written
permission by copyright owner. Copyright infringers may face civil and criminal liability
48 Which of these is NOT one of the major styles or schools of ancient Indian art?
A. Amravati
B. Mathura
C. Gandhara
D. Sarnath
Your Answer : D
Correct Answer : D
Answer Justification :
Learning: The Amravati school of art had great influence on art in Sri Lanka and South-East Asia
as products from here were carried to those countries. It also had influence over South Indian
sculpture.
Mathura: During the first century AD, Gandhara and Mathura School of Art flourished mainly
during reign of Kushana emperor Kanishka.
Mathura School had developed indigenously.
Apart from Mathura, other important centres of this art were Sarnath and Kosambi.
The material used in this school was the spotted red sandstone. This art reached its peak
during the Gupta period in 6th or 7th century.
Gandhara will be covered later.
Q Source: Improvisation: Page 66: NCERT Class VI Our Pasts - I
TEST - 1 (MOCK TEST - 1)
www.insightsias.com 56
© Insights Active Learning| All rights reserved. You may not reproduce, distribute or exploit the contents in any form without written
permission by copyright owner. Copyright infringers may face civil and criminal liability
49 With reference to ANUGA 2017, consider the following:
1. It is an inter-governmental platform organized by UNSC for promoting defence technology
cooperation.
2. India will be a co-Partner Country in 2017 edition of the event.
3. Agreements signed under the event are mandated to take the place of national laws and impose
regulatory burdens on party nations.
Select the correct answer using the codes below.
A. 1 only
B. 2 and 3 only
C. 2 only
D. 3 only
Your Answer :
Correct Answer : C
Answer Justification :
Justification: Statement 1: ANUGA is the world’s biggest and most important trade fair for Food
and beverage trade held in Germany.
It takes place biennially (every 2 years). ANUGA 2017 is the 34th edition.
ANUGA offers an extensive supporting programme with Lectures, Special exhibition and
attractive industry events
More than fifty per cent of the industry visitors are foreigners.
Statement 2: India is to become a co-Partner Country in ANUGA 2017. It gives India an opportunity
to showcase its strengths and opportunities to an international audience in the food sector and
invite investments.
Statement 3: There is no such provision.
Q Source: http://pib.nic.in/newsite/PrintRelease.aspx?relid=165597
TEST - 1 (MOCK TEST - 1)
www.insightsias.com 57
© Insights Active Learning| All rights reserved. You may not reproduce, distribute or exploit the contents in any form without written
permission by copyright owner. Copyright infringers may face civil and criminal liability
50 Indian Railways Organization for Alternate Fuel (IROAF) has been recently awarded the coveted
National level “Golden Peacock Award for the Year 2017 for Eco-Innovation”. The award was given
for
1. Substitution of Diesel in passenger trains by environment friendly Compressed Natural Gas (CNG).
2. Installation of open source licensing system for electricity generation and procurement
Which of the above is/are correct?
A. 1 only
B. 2 only
C. Both 1 and 2
D. None
Your Answer :
Correct Answer : A
Answer Justification :
Justification: Indian Railways has achieved a major landmark in the field of Eco friendly fuel
technologies by this innovation.
Use of CNG in Train Sets for passenger transportation has been done for the first time in the world.
This innovation (20% substitution of diesel by CNG) will reduce emissions considerably by
bringing down NOx, CO2 and Particulate Matter besides achieving economy in fuel cost by
eight per cent.
It could save up to over thirteen hundred crores annually if implemented over entire fleet of
Diesel Locomotives of Indian Railways.
An improved technology of 40% substitution is at present being developed by IROAF. This will
enhance the potential of saving of fuel cost of IR to about Rs. thirty four hundred crore per annum.
Q Source: PIB Releases: 11th July
TEST - 1 (MOCK TEST - 1)
www.insightsias.com 58
© Insights Active Learning| All rights reserved. You may not reproduce, distribute or exploit the contents in any form without written
permission by copyright owner. Copyright infringers may face civil and criminal liability
51 The Mauryan state had a well-organized civil service, this is evident from
1. Civil servants called Amatyas who looked after the day-to-day administration were selected based
on merit.
2. Adyakshas officers helped control the retail and wholesale prices of goods and ensured their steady
supply in the state.
3. All officials were interviewed and appointed directly by the emperor.
Select the correct answer using the codes below.
A. 1 and 2 only
B. 2 and 3 only
C. 3 only
D. 2 only
Your Answer : A
Correct Answer : A
Answer Justification :
Justification: A council of ministers called Mantriparishad assisted the king in administrative
matters.
It consisted of Purohita, Mahamantri, Senapati and Yuvaraja.
Statement 1: Amatyas were similar to the IAS officers of independent India. The method of selection
of Amatyas was elaborately given by Kautilya.
Statement 2: Department of Commerce and Industry had controlled the retail and wholesale prices
of goods and tried to ensure their steady supply through its officers called Adyakshas. It also
controlled weights and measures, levied custom duties and regulated foreign trade.
Statement 3: Selection process was different for different officials.
Learning: Asoka also appointed Dhamma Mahamatras to supervise the spread of Dhamma.
Even the military service was well organized. The Mauryan army was well organized and it was
under the control of Senapati. The salaries were paid in cash. Kautilya refers to the salaries of
different ranks of military officers.
Q Source: Page 66-67: TN 11th Standard History Textbook
TEST - 1 (MOCK TEST - 1)
www.insightsias.com 59
© Insights Active Learning| All rights reserved. You may not reproduce, distribute or exploit the contents in any form without written
permission by copyright owner. Copyright infringers may face civil and criminal liability
52 Ptolemy, a Greek writer, mentions many important ports in the Deccan. Among them, the greatest
port of the Satavahanas in Western Deccan was
A. Kalyani
B. Ganjam
C. Satakarni
D. Pulamayi
Your Answer : C
Correct Answer : A
Answer Justification :
Learning: Vashishtaputra Pulamayi extended the Satavahana power up to the mouth of the Krishna
river.
He issued coins on which the image of ships was inscribed. They reveal the naval power and
maritime trade of the Satavahanas.
Their greatest port was Kalyani on the west Deccan. Gandakasela and Ganjam on the east coast
were the other important seaports.
The last great ruler of Satavahanas was Yajna Sri Satakarni.
Q Source: Page 76: TN 11th Standard History Textbook
TEST - 1 (MOCK TEST - 1)
www.insightsias.com 60
© Insights Active Learning| All rights reserved. You may not reproduce, distribute or exploit the contents in any form without written
permission by copyright owner. Copyright infringers may face civil and criminal liability
53 The National Mission on Cultural Mapping of India intends to
1. Create repository of information about cultural assets
2. Launch massive cultural awareness campaigns
3. Open a direct channel of communication of artists with the Government
4. Hold talent hunt competitions from Block level to National level
Select the correct answer using the codes below.
A. 1 and 4 only
B. 2 and 3 only
C. 1, 2 and 3 only
D. 1, 2, 3 and 4
Your Answer : C
Correct Answer : D
Answer Justification :
Learning: The mission aims at converting the vast and widespread cultural canvas of India into an
objective cultural map to preserve our cultural heritage.
The Mission also seeks to open a direct channel of communication of artists with the
Government and peer to peer communication among artists for talent honing and handholding
of each other.
With a vision that no cultural talent or heritage should remain unattended (so that it could be
nurtured), the Ministry of Culture has planned to hold talent hunt competitions with various
rounds from Block level to National level.
The Artist/Organisations also may register online at www.culturalmapping.nic.in.
The Cultural Mapping of India portal will serve as a repository/consolidated database of
information about cultural assets and resources i.e. a cultural inventory at one place for
essential planning, sustainable economic development and for preserving the scattered and
near extinct art forms.
Q Source: http://pib.nic.in/newsite/PrintRelease.aspx?relid=165655
TEST - 1 (MOCK TEST - 1)
www.insightsias.com 61
© Insights Active Learning| All rights reserved. You may not reproduce, distribute or exploit the contents in any form without written
permission by copyright owner. Copyright infringers may face civil and criminal liability
54 This council held near Srinagar prepared an authoritative commentary on the Buddhist Tripitakas
and gave the Mahayana doctrine a final shape. The council was convened by
A. Kanishka
B. Samprati
C. Bindusara Amitraghata
D. Devavarman
Your Answer :
Correct Answer : A
Answer Justification :
Learning: He convened this Fourth Buddhist Council that discussed matters relating to Buddhist
theology and doctrine.
It was held at the Kundalavana monastery near Srinagar in Kashmir under the presidentship of
Vasumitra. About 500 monks attended the Council.
Learning: Asvagosha was a great philosopher, poet and dramatist and was particularly close to
Kanishka. He became his religious advisor in his later years and composed Buddhacharita.
Nagarjuna from south India also adorned the court of Kanishka. The famous physician of ancient
India Charaka was also patronized by him.
Q Source: Page: TN 11th Standard History Textbook
TEST - 1 (MOCK TEST - 1)
www.insightsias.com 62
© Insights Active Learning| All rights reserved. You may not reproduce, distribute or exploit the contents in any form without written
permission by copyright owner. Copyright infringers may face civil and criminal liability
55 Consider the following about The Energy Resources Institute (TERI).
1. It is a non-profit policy research organisation.
2. It is an attached agency of the International Centre for Climate Governance (ICCG).
Which of the above is/are correct?
A. 1 only
B. 2 only
C. Both 1 and 2
D. None
Your Answer : C
Correct Answer : A
Answer Justification :
Justification: Statement 1: TERI conducts research work in the fields of energy, environment and
sustainable development.
Statement 2: The Energy Resources Institute (TERI) was ranked second among world’s best climate
think tank by the International Centre for Climate Governance (ICCG).
ICCG is an internationally renowned centre founded in 2009.
The main focus of ICCG activities is the design of climate policy and related governance issues.
Q Source: www.insightsonindia.com/2017/07/06/insights-daily-current-affairs-06-july-2017/
TEST - 1 (MOCK TEST - 1)
www.insightsias.com 63
© Insights Active Learning| All rights reserved. You may not reproduce, distribute or exploit the contents in any form without written
permission by copyright owner. Copyright infringers may face civil and criminal liability
56 Which of these ancient scripts generally writes in left to right direction?
A. Brahmi
B. Kharosthi
C. Harappan
D. All of the above
Your Answer : B
Correct Answer : A
Answer Justification :
Learning: Brahmi: is one of the oldest writing systems used in South and Central Asia from the 1st
millennium BCE.
Most modern Indian scripts have developed from the Brahmi script over hundreds of years.
Harappan: Iravatham Mahadevan established that the Harappan script is from right to left.
Kharosthi: is an ancient script used in ancient Gandhara (primarily modern-day Afghanistan and
Pakistan) to write the Gandhari Prakrit and Sanskrit.
Q Source: Improvisation: Page 81: NCERT Class VI Our Pasts - I
TEST - 1 (MOCK TEST - 1)
www.insightsias.com 64
© Insights Active Learning| All rights reserved. You may not reproduce, distribute or exploit the contents in any form without written
permission by copyright owner. Copyright infringers may face civil and criminal liability
57 Consider the following statements.
1. Innovate in India (i3) program offers young entrepreneurs an avenue to engage with the best
practices in the domestic financial and banking industry.
2. National Biopharma Mission will concentrate, inter alia, on development of medical devices and
biotherapeutics to upgrade India’s product development capabilities to globally competitive levels.
Which of the above is/are correct?
A. 1 only
B. 2 only
C. Both 1 and 2
D. None
Your Answer : B
Correct Answer : B
Answer Justification :
Justification: Statement 1: i3 is a flagship program of the Government of India in collaboration
with World Bank.
It is committed to make India a hub for design and development of novel, affordable and
effective biopharmaceutical products and solutions by engaging with entrepreneurs.
It will strive to increase India’s global biopharmaceutical market share from current 2.8% to
5% and generate additional business opportunity of $16 billion.
Statement 2 (S2): It will bring together expertise from national and international corridors to
provide strategic direction and guidance to move promising solutions through the product
development value chain.
The mission will be implemented by Biotechnology Industry Research Assistance Council (BIRAC), a
PSU of Department of Biotechnology.
Besides, it will also work on establishment of shared infrastructure and facilities and at the same
time creating and enhancing technology transfer capabilities in public and private sector.
Q Source: http://pib.nic.in/newsite/PrintRelease.aspx?relid=166951
TEST - 1 (MOCK TEST - 1)
www.insightsias.com 65
© Insights Active Learning| All rights reserved. You may not reproduce, distribute or exploit the contents in any form without written
permission by copyright owner. Copyright infringers may face civil and criminal liability
58 In a democratic state, government derives its ultimate sovereignty from
A. Citizens of the state
B. Territory of the State
C. Laws governing the state
D. Parliament
Your Answer : D
Correct Answer : A
Answer Justification :
Justification: The chain is straightforward.
People elect representatives, representatives form Parliament, Parliament chooses the government,
government makes laws, and those laws regulate the territory of the state.
So, the root of sovereignty can be easily traced to people.
Q Source: Improvisation: Page 31: NCERT Class VI Social and Political Life - I
TEST - 1 (MOCK TEST - 1)
www.insightsias.com 66
© Insights Active Learning| All rights reserved. You may not reproduce, distribute or exploit the contents in any form without written
permission by copyright owner. Copyright infringers may face civil and criminal liability
59 Chandragupta II, a ruler of the Gupta Empire, assumed the famous title of ‘Sakari’. What did the title
imply?
A. Destroyer of Sakas
B. Lord of the world
C. Well Wisher of masses
D. Devotee of Shankara
Your Answer :
Correct Answer : A
Answer Justification :
Learning: The greatest of the military achievements of Chandragupta II was his war against the
Saka satraps of western India.
Rudrasimha III, the last ruler of the Saka satrap was defeated, dethroned and killed.
His territories in western Malwa and the Kathiawar Peninsula were annexed into the Gupta
Empire.
After this victory he performed the horse sacrifice and assumed the title Sakari, meaning,
‘destroyer of Sakas’. He also called himself Vikramaditya.
Q Source: Page 101: TN 11th Standard History Textbook
TEST - 1 (MOCK TEST - 1)
www.insightsias.com 67
© Insights Active Learning| All rights reserved. You may not reproduce, distribute or exploit the contents in any form without written
permission by copyright owner. Copyright infringers may face civil and criminal liability
60 Consider the following statements about the recently established Global Foreign Exchange
Committee (GFXC).
1. It was established as a research arm of International Monetary Fund (IMF).
2. GFXC will enforce a FX Global Code that will legally regulate the manipulation of floating foreign
exchange by Central Banks.
3. It will promote collaboration and communication among the local foreign exchange committees
(FXCs) of member nations.
4. It will replace the institution of Bank of International Settlements (BIS).
5. Indian government has boycotted GFXC due to its non-democratic governance structure.
Select the correct answer using the codes below.
A. 2, 3 and 4 only
B. 1, 2 and 5 only
C. 3 only
D. 1, 2, 3 and 4 only
Your Answer : C
Correct Answer : C
Answer Justification :
Justification: This is a very important development, hence the long explanation.
Statement 1: GFXC was established in 2017 as a forum bringing together central banks and private
sector participants with the aim to promote a robust, liquid, open, and appropriately transparent FX
market.
Statement 2: FX Global Code is a set of global principles of good practice in the foreign exchange
market, developed to provide a common set of guidelines to promote the integrity and effective
functioning of the wholesale foreign exchange market.
The Global Code does not impose legal or regulatory obligations on Market Participants, nor
does it substitute for regulation.
It is rather intended to serve as a supplement to any and all local laws, rules and regulations
by identifying global good practices and processes.
Statement 3: The objectives of the GFXC are:
to promote collaboration and communication among the local foreign exchange committees
(FXCs) and non-GFXC jurisdictions with significant FX markets;
to exchange views on trends and developments in global FX markets, including on the
structure and functioning of those markets, drawing on information gathered at the various
FXCs; and
TEST - 1 (MOCK TEST - 1)
www.insightsias.com 68
© Insights Active Learning| All rights reserved. You may not reproduce, distribute or exploit the contents in any form without written
permission by copyright owner. Copyright infringers may face civil and criminal liability
to promote, maintain and update on a regular basis the FX Global Code (the Code) and to
consider good practices regarding effective mechanisms to support adherence.
Statement 4: The GFXC committee has been set up under guidance of the Bank for International
Settlements (BIS), an international financial organisation.
Statement 5: India will soon join Global Foreign Exchange Committee (GFXC).
Q Source:
http://www.thehindubusinessline.com/markets/forex/india-to-join-new-global-foreign-exchange-com
mittee/article9756342.ece
TEST - 1 (MOCK TEST - 1)
www.insightsias.com 69
© Insights Active Learning| All rights reserved. You may not reproduce, distribute or exploit the contents in any form without written
permission by copyright owner. Copyright infringers may face civil and criminal liability
61 Why the sale of Oxytocin, despite being a natural hormone, is heavily regulated?
1. It was used as a depressant by a large segment of population.
2. Its indiscriminate use by dairy farmers was causing irreversible hormone damage in milch animals.
3. Consumption of food laced with Oxytocin can cause cardiac problems in humans.
4. It is abused to speed up child birth in women.
Select the correct answer using the codes below.
A. 1 and 3 only
B. 2, 3 and 4 only
C. 2 and 4 only
D. 1, 2, 3 and 4
Your Answer : B
Correct Answer : B
Answer Justification :
Justification: Statement 1: Oxytocin has also been dubbed the love or the bliss hormone due to its
effects on behaviour. It isn’t a depressant. So, 1 is wrong.
Statement 2: The drug Oxytocin is banned in India under Prevention of Cruelty to Animals Act and
Food and Drug Adulteration Prevention Act, 1960.
In mammals, Oxytocin stimulates ejection of milk from the mammary glands.
But, its use can cause reproductive damage in animals and even their death.
Statement 3: Overdose of the drug in humans can lead to haemorrhage, palpitation, low blood
pressure and in some cases even death.
Statement 4: The release of oxytocin by the pituitary gland acts to regulate two female reproductive
functions: Childbirth and Breast-feeding. So, it is used to speed up child birth which is wrong and
should be shunned.
So, after being banned in retail markets, the Indian Drug Controller will soon restrict
manufacturing of controversial hormone drug Oxytocin to public sector undertakings (PSUs).
Q Source:
http://www.thehindu.com/news/national/only-psus-may-be-let-to-make-oxytocin/article18868286.ece
TEST - 1 (MOCK TEST - 1)
www.insightsias.com 70
© Insights Active Learning| All rights reserved. You may not reproduce, distribute or exploit the contents in any form without written
permission by copyright owner. Copyright infringers may face civil and criminal liability
62 Consider the following statements.
1. Manimegalai is a Tamil epic that criticizes Buddhism while investigating the strengths of
contemporary Hindu traditions.
2. Silappatikaram, produced at Sangam, is a poetic rendition with details of Tamil culture.
Which of the above is/are correct?
A. 1 only
B. 2 only
C. Both 1 and 2
D. None
Your Answer :
Correct Answer : B
Answer Justification :
Justification: The corpus of Sangam literature includes Tolkappiyam, Ettutogai, Pattuppattu,
Pathinenkilkanakku, and the two epic – Silappathigaram and Manimegalai.
Statement 1: The aim of the author of Manimegalai, Seethalai Saathanar, was to compare Buddhism
favourably with the other prevailing religions in South India in order to propagate Buddhism.
He criticizes Hinduism, and exposes the weaknesses of the other contemporary Indian
religions, while praising the Buddha's Teaching, the Dharma, as the most perfect religion. So,
1 is wrong.
Statement 2: Regarded as one of the great works of Tamil literature, the Silappatikaram is a poetic
rendition with details of Tamil culture; its varied religions; its town plans and city types; the
mingling of different people; and the arts of dance and music.
Q Source: Improvisation: Page 85-86: TN 11th Standard History Textbook
TEST - 1 (MOCK TEST - 1)
www.insightsias.com 71
© Insights Active Learning| All rights reserved. You may not reproduce, distribute or exploit the contents in any form without written
permission by copyright owner. Copyright infringers may face civil and criminal liability
63 Consider the following statements.
1. Harsha prohibited the use of animal food in his kingdom and punished those who killed any living
being.
2. Harsha gave away his enormous wealth as gifts to the members of all religious sects during the
Allahabad Conference.
Which of the above is/are correct?
A. 1 only
B. 2 only
C. Both 1 and 2
D. None
Your Answer : C
Correct Answer : C
Answer Justification :
Justification: Statement 1: In his early life, Harsha was a devout Saiva but later he became an
ardent Hinayana Buddhist. Hiuen Tsang converted him to Mahayana Buddhism. This led him to
shed eating meat and enforce the same as well.
Statement 2: Hiuen Tsang mentions in his account about the conference held at Allahabad, known
as Prayag.
It was the one among the conferences routinely convened by Harsha once in five years. Harsha gave
away his enormous wealth as gifts to the members of all religious sects.
According to Hiuen Tsang, Harsha was so lavish that he emptied the treasury and even gave away
the clothes and jewels he was wearing. His statement might be one of admiring exaggeration.
Q Source: Page 116: TN 11th Standard History Textbook
TEST - 1 (MOCK TEST - 1)
www.insightsias.com 72
© Insights Active Learning| All rights reserved. You may not reproduce, distribute or exploit the contents in any form without written
permission by copyright owner. Copyright infringers may face civil and criminal liability
64 With reference to the rights and responsibilities of the Attorney General of India, consider the
following:
1. He has right of audience in all courts within the territory of India.
2. He has the right to speak and take part in the proceeding of a joint sitting of the Parliament.
3. He is entitled to vote on crucial national resolutions and motions in Rajya Sabha, excluding
legislations.
4. He can be made a member of any parliamentary committee with a right to discuss and vote.
5. He is not allowed to take up private practice as long as he is an incumbent.
Select the correct answer using the codes below.
A. 1 and 2 only
B. 1, 3 and 4 only
C. 1, 4 and 5 only
D. 2, 3 and 5 only
Your Answer : C
Correct Answer : A
Answer Justification :
Justification: He is the first law officer of the government of India and acts as top advocate for
Union Government. He is responsible for giving advice to President/ Union Government upon legal
matters.
Statement 2 and 3: He has the right to speak and take part in proceedings of both the houses of
parliament including joint sittings. But cannot vote in parliament.
Statement 4: He can also be made a member of any parliamentary committee but in the committee
also, he has no power to vote. He has all the powers and privileges that of a member of parliament.
Statement 5: Attorney General of India is not a full time Government servant. He is an advocate of
the government and is allowed to take up private practice, provided the other party is not the state.
Further, without permission of the government he cannot defend the accused persons in criminal
matters.
Q Source:
http://www.thehindu.com/news/national/attorney-general-rohatgi-will-defend-indias-record-on-rights
-abuses-at-unhrcs-review/article18312468.ece
TEST - 1 (MOCK TEST - 1)
www.insightsias.com 73
© Insights Active Learning| All rights reserved. You may not reproduce, distribute or exploit the contents in any form without written
permission by copyright owner. Copyright infringers may face civil and criminal liability
65 In the context of the rule of South Indian dynasties, Devadhana and Brahmadeya referred to?
A. Pristine land
B. Divine king rule
C. Treasure appropriated to the Gods
D. Land grants to the temples
Your Answer : C
Correct Answer : D
Answer Justification :
Justification: Land tax was the primary source of the government revenue. The Brahmadeya and
Devadhana lands were exempted from tax.
Learning: This is a lateral explanation and deals with aspects other than those mentioned in the
question, because detailing the topic at hand is not of much use.
The Pallavas had a well organized administrative system. The Pallava state was divided into
Kottams. The Kottam was administered by officers appointed by the king.
It was also the responsibility of the central government to provide irrigation facilities to the
lands.
Traders and artisans such as carpenters, goldsmiths, washer-men, oil-pressers and weavers
paid taxes to the government.
The Pallava inscriptions throw much light on the village assemblies called sabhas and their
committees. They maintained records of all village lands, looked after local affairs and
managed temples.
Q Source: Page 126: TN 11th Standard History Textbook
TEST - 1 (MOCK TEST - 1)
www.insightsias.com 74
© Insights Active Learning| All rights reserved. You may not reproduce, distribute or exploit the contents in any form without written
permission by copyright owner. Copyright infringers may face civil and criminal liability
66 They introduced the art of excavating temples from the rock, and the Dravidian style of temple
architecture began with their rule. They were?
A. Cheras
B. Pallavas
C. Eastern Chalukyas
D. Jaffna
Your Answer : B
Correct Answer : B
Answer Justification :
Learning: Pallava age was a great age of temple building.
It was a gradual evolution starting from the cave temples to monolithic rathas and culminated in
structural temples.
The development of temple architecture under the Pallavas can be seen in several stages, which
started with Mahendravarman I introducing rock-cut temples.
This style of Pallava temples are seen at places like Mandagappattu, Mahendravadi, Mamandur,
Dalavanur, Tiruchirappalli, Vallam, Siyamangalam and Tirukalukkunram.
Q Source: Page 127: TN 11th Standard History Textbook
TEST - 1 (MOCK TEST - 1)
www.insightsias.com 75
© Insights Active Learning| All rights reserved. You may not reproduce, distribute or exploit the contents in any form without written
permission by copyright owner. Copyright infringers may face civil and criminal liability
67 In Delhi Sultanate, the Sultan was assisted by a number of departments and officials in his
administration. Consider the following with reference to these departments:
1. Diwani Insha dealt with the correspondence between the ruler and the officials.
2. Diwani Ariz gave grants for the construction and maintenance of mosques, tombs and madrasas.
Which of the above is/are correct?
A. 1 only
B. 2 only
C. Both 1 and 2
D. None
Your Answer :
Correct Answer : A
Answer Justification :
Justification: Statement 1: It is correct.
Statement 2: The military department was called Diwani Ariz. It was headed by Ariz-i-mumalik. He
was responsible for recruiting the soldiers and administering the military department.
Learning: Under the Delhi sultanate, the post of Naib was the most powerful one. The Naib
practically enjoyed all the powers of the Sultan and exercised general control over all the
departments.
Next to him was the Wazir who was heading the finance department called Diwani Wizarat.
Diwani Rasalat was the department of religious affairs. It was headed by chief Sadr. Grants
were made by this department for religious construction.
Q Source: Page 192: TN 11th Standard History Textbook
TEST - 1 (MOCK TEST - 1)
www.insightsias.com 76
© Insights Active Learning| All rights reserved. You may not reproduce, distribute or exploit the contents in any form without written
permission by copyright owner. Copyright infringers may face civil and criminal liability
68 In the context of Ancient India, Amogavarsha’s work Kavirajamarga is significant because?
A. It was the first poetic work in Kannada language.
B. The largest repository of royal assets was commissioned in this work.
C. All artists of the empire contributed to this work.
D. It was a prose written in the middle of a battle ground.
Your Answer :
Correct Answer : A
Answer Justification :
Learning: The Kannada literature saw its beginning during the period of the Rashtrakutas.
Amogavarsha’s Kavirajamarga was the first poetic work in Kannada language.
The name literally means "Royal Path for Poets" and was written as a guide book for poets and
scholars (Kavishiksha).
Q Source: Page 137: TN 11th Standard History Textbook
TEST - 1 (MOCK TEST - 1)
www.insightsias.com 77
© Insights Active Learning| All rights reserved. You may not reproduce, distribute or exploit the contents in any form without written
permission by copyright owner. Copyright infringers may face civil and criminal liability
69 Several accords have been signed between the Government of India (GoI) and different groups to
bring peace and prosperity to the North-eastern states. Consider the following with reference to
them.
1. Mizo accord promised statehood to Mizoram.
2. Assam Accord was reached between GoI and ULFA calming the Assam agitation.
3. Nagaland peace accord between GoI and the National Socialist Council of Nagaland (NSCN) was
signed to end the Naga insurgency.
Select the correct answer using the codes below.
A. 1 and 3 only
B. 3 only
C. 2 and 3 only
D. 1 and 2 only
Your Answer : A
Correct Answer : A
Answer Justification :
Justification: Statement 1: Under the Mizo accord, the Mizo National Front (MNF) fighters
surrendered en masse and government fulfilled its promise by giving full-fledged state status to
Mizoram, along with its own High Court.
Mizo was notified as an official Indian language. Further, Mizo religious and social customs
were promised Constitutional protection.
As per the accord, the MNF agreed to forgo all contacts with other insurgent groups in the
Northeast. Mizo Accord is considered to be the most successful agreement with insurgents.
Statement 2: ULFA is a banned organization. So, 2 is wrong.
The accord brought an end to the Assam Agitation and paved the way for the leaders of the
agitation to form a political party and form a government in the state of Assam soon after. However,
not all promises have been fulfilled that remains a bone of contention.
Statement 3: This article explains the history and issues with Naga insurgency very clearly and
chronologically: A must read – This is an important issue.
http://indianexpress.com/article/explained/simply-put-towards-accord-step-by-step/
Q Source:
http://www.thehindu.com/news/national/mizoram-celebrates-31st-anniversary-of-mizo-peace-accord/
article19187194.ece
TEST - 1 (MOCK TEST - 1)
www.insightsias.com 78
© Insights Active Learning| All rights reserved. You may not reproduce, distribute or exploit the contents in any form without written
permission by copyright owner. Copyright infringers may face civil and criminal liability
70 The system of village autonomy with sabhas and their committees developed through the ages and
reached its culmination during the Chola rule. A village was divided in several wards, what was/were
the qualifications to become a ward member?
1. A woman could not become a ward member.
2. Only those who have completed seventy years of age
3. Have knowledge of Vedas
4. Owns a residence
Select the correct answer using the codes below.
A. 3 and 4 only
B. 1, 2 and 3 only
C. 1 and 4 only
D. 1, 2, 3 and 4
Your Answer : C
Correct Answer : A
Answer Justification :
Learning: That village was divided into thirty wards and each was to nominate its members to the
village council.
The qualifications to become a ward member were:
Ownership of at least one fourth veli of land.
Own residence.
Above thirty years and below seventy years of age.
Knowledge of Vedas.
However, certain norms of disqualification were also mentioned in the inscriptions. They were:
Those who had been members of the committees for the past three years.
Those who had failed to submit accounts as committee members.
Those who had committed sins.
Those who had stolen the property of others
Two inscriptions belonging to the period of Parantaka I found at Uttiramerur provide details of the
TEST - 1 (MOCK TEST - 1)
www.insightsias.com 79
© Insights Active Learning| All rights reserved. You may not reproduce, distribute or exploit the contents in any form without written
permission by copyright owner. Copyright infringers may face civil and criminal liability
formation and functions of village councils.
Q Source: Page 148: TN 11th Standard History Textbook
TEST - 1 (MOCK TEST - 1)
www.insightsias.com 80
© Insights Active Learning| All rights reserved. You may not reproduce, distribute or exploit the contents in any form without written
permission by copyright owner. Copyright infringers may face civil and criminal liability
71 In the ancient traditions, traders’ voyages often refer which of these lands to Suvarnabhumi (the land
of gold)?
A. Countries of the East Asia
B. African nations
C. Sri Lankan islands
D. European countries
Your Answer :
Correct Answer : A
Answer Justification :
Learning: The east coast of India is studded with numerous ports and Indians undertook frequent
voyages to these lands.
Also, Indians began to colonize the East Asia in the Gupta period. It was further encouraged
by the Pallavas.
The Indian colonists established great kingdoms (with Indian names) and some of them lasted
for more than a thousand years.
Till the arrival of Islam in the fifteenth century, Indian culture dominated this region.
Q Source: Page 156: TN 11th Standard History Textbook
TEST - 1 (MOCK TEST - 1)
www.insightsias.com 81
© Insights Active Learning| All rights reserved. You may not reproduce, distribute or exploit the contents in any form without written
permission by copyright owner. Copyright infringers may face civil and criminal liability
72 The system of four ashrams is advocated in
A. Buddhism
B. Jainism
C. Lokayatas
D. Brahmanism
Your Answer : C
Correct Answer : D
Answer Justification :
Learning: Around the time when Jainism and Buddhism were becoming popular, brahmins
developed the system of ashramas.
Ashramas refer to stage of life: brahmacharya, grihastha, vanaprastha and samnyasa.
Brahmin, kshatriya and vaishya men were expected to lead simple lives and study the Vedas
during the early years of their life (brahmacharya).
Then they had to marry and live as householders (grihastha).
Then they had to live in the forest and meditate (vanaprastha). Finally, they had to give up
everything and become samnyasins.
The system of ashramas allowed men to spend some part of their lives in meditation.
Generally, women were not allowed to study the Vedas, and they had to follow the ashramas
chosen by their husbands.
Q Source: Page 173: TN 11th Standard History Textbook
TEST - 1 (MOCK TEST - 1)
www.insightsias.com 82
© Insights Active Learning| All rights reserved. You may not reproduce, distribute or exploit the contents in any form without written
permission by copyright owner. Copyright infringers may face civil and criminal liability
73 ‘Petya’ recently seen in news was a/an
A. Laser beaming innovation
B. Form of cyber-attack
C. Alternate to fossil fuels
D. Emergency helpline launched by PETA
Your Answer : B
Correct Answer : B
Answer Justification :
Learning: It is a ransomware, similar to the Wannacry ransomware.
In India, this ransomware crippled the operations at one of the terminals of the Jawaharlal Nehru
Port Trust and impacted the external trade by affecting the systems dealing with the cargo and
ships.
Ransomware is a type of malicious software that threatens to publish the victim's data or
perpetually block access to it unless a ransom is paid.
Q Source:
https://www.theguardian.com/technology/2017/jun/27/petya-ransomware-cyber-attack-who-what-wh
y-how
TEST - 1 (MOCK TEST - 1)
www.insightsias.com 83
© Insights Active Learning| All rights reserved. You may not reproduce, distribute or exploit the contents in any form without written
permission by copyright owner. Copyright infringers may face civil and criminal liability
74 In 1329-30 Muhammad bin Tughlaq introduced a token copper currency in place of silver, but banned
it later and promised to exchange silver coins for these copper coins. This was done because
A. Copper was scarce in the empire, but had huge demand in the world.
B. Token currency was trampled and disrespected by people.
C. Goldsmiths began to forge the token coins on a large scale
D. Treasury became empty with issue of token coins.
Your Answer : A
Correct Answer : C
Answer Justification :
Learning: There was a shortage of silver through out the world in the fourteenth century.
Kublai Khan issued paper money in China. In the same manner, Muhammad bin Tughlaq
issued copper coins at par with the value of the silver tanka coins.
But he was not able to prevent forging the new coins. The goldsmiths began to forge the
token coins on a large scale. Soon the new coins were not accepted in the markets due to its
widescale duplication and thus credibility.
Finally, Muhammad bin Tughlaq stopped the circulation of token currency and promised to
exchange silver coins for the copper coins.
Many people exchanged the new coins but the treasury became empty.
According the Barani, the heap of copper coins remained lying on roadside in Tughlaqabad.
Q Source: Page 184: TN 11th Standard History Textbook
TEST - 1 (MOCK TEST - 1)
www.insightsias.com 84
© Insights Active Learning| All rights reserved. You may not reproduce, distribute or exploit the contents in any form without written
permission by copyright owner. Copyright infringers may face civil and criminal liability
75 Writing in the journal Young India in 1931, who said, "I cannot possibly bear the idea that a man who
has got wealth should get the vote but a man who has got character but no wealth or literacy should
have no vote…. for the crime of being a poor man…"?
A. Jawahar Lal Nehru
B. M.K. Gandhi
C. Rajendra Prasad
D. Maulana Abul Kalam Azad
Your Answer :
Correct Answer : B
Answer Justification :
Learning: Young India, Harijan and Hind Swaraj were some of the most famous works of Gandhi.
Young India was a weekshed - a weekly paper or journal.
He used Young India to spread his unique ideology and thoughts regarding the use of nonviolence
in organising movements and to urge readers to consider, organise, and plan for India's eventual
independence from Britain.
The journal was reprinted in USA by the India Home Rule League of America.
Q Source: Improvisation: Page 33: NCERT Class VI Social and Political Life - I
TEST - 1 (MOCK TEST - 1)
www.insightsias.com 85
© Insights Active Learning| All rights reserved. You may not reproduce, distribute or exploit the contents in any form without written
permission by copyright owner. Copyright infringers may face civil and criminal liability
76 In the context of Pallava’s history, what was Dakshinachitra?
A. A compendium of temples
B. A spiritual treatise on Pallava practices
C. An autobiography of a Pallava ruler
D. An instruction manual for painters
Your Answer :
Correct Answer : D
Answer Justification :
Learning: Mahendravarman, one of the greatest of the rulers of the Pallava dynasty, was found of
painting and is said to have commissioned a scholar at his court to pen down an instruction manual
for painters, which is a treatise on south Indian paintings called Dakshinachitra.
Unfortunately, it has been lost in the course of time.
Mahendravarman I gave himself the title of “Vchitrachitta” (the man with new fangled ideas).
Q Source: Page 129: TN 11th Standard History Textbook
TEST - 1 (MOCK TEST - 1)
www.insightsias.com 86
© Insights Active Learning| All rights reserved. You may not reproduce, distribute or exploit the contents in any form without written
permission by copyright owner. Copyright infringers may face civil and criminal liability
77 In medieval India, takkavi loans were given in order to
A. Improve agriculture
B. Build religious shrines
C. Construct flood saving embankments in cities
D. Serve sovereign debt
Your Answer :
Correct Answer : A
Answer Justification :
Learning: Takkavi loans were a measure of reprieve taken after few suppressing moves of Tughlaq.
In order to overcome financial difficulties in his empire, Muhammad bin Tughlaq increased
the land revenue on the farmers of Doab (land between Ganges and Yamuna rivers).
It was an excessive and arbitrary step on the farmers. A severe famine was also ravaging that
region at that time. It had resulted in a serious peasant revolts. The revolts were crushed.
However, the Sultan realized later that adequate relief measures and the promotion of
agriculture were the real solution to the problem.
He launched a scheme by which takkavi loans (loans for cultivation) were given to the farmers
to buy seed and to extend agriculture.
Q Source: Page 193: TN 11th Standard History Textbook
TEST - 1 (MOCK TEST - 1)
www.insightsias.com 87
© Insights Active Learning| All rights reserved. You may not reproduce, distribute or exploit the contents in any form without written
permission by copyright owner. Copyright infringers may face civil and criminal liability
78 Consider the following statements.
1. Sankara denounced Hindu revivalist movement citing them as going against rationality and
modernity.
2. Ramanuja advocated prabattimarga or the path of self-surrender to God for salvation.
Which of the above is/are correct?
A. 1 only
B. 2 only
C. Both 1 and 2
D. None
Your Answer :
Correct Answer : B
Answer Justification :
Learning: Statement 1: In the ninth century Sankara started a Hindu revivalist movement giving a
new orientation to Hinduism. So, 1 is wrong.
However, his doctrine of Advaita or Monism was too abstract to appeal to the common man.
Moreover, there was a reaction against the Advaita concept of Nirgunabrahman (God without
attributes) with the emergence of the idea of Sagunabrahman (God with attributes).
Statement 2: In the twelfth century, Ramanuja, preached Visishtadvaita.
According to him God is Sagunabrahman. The creative process and all the objects in creation
are real but not illusory as was held by Sankaracharya.
Therefore, God, soul, matter are real. But God is inner substance and the rest are his
attributes.
He invited the downtrodden to Vaishnavism
Q Source: Page 202: TN 11th Standard History Textbook
TEST - 1 (MOCK TEST - 1)
www.insightsias.com 88
© Insights Active Learning| All rights reserved. You may not reproduce, distribute or exploit the contents in any form without written
permission by copyright owner. Copyright infringers may face civil and criminal liability
79 Consider the following statements.
1. A Supernova helps us know about the Universe since they are the longest existing phenomenon in
space.
2. Pulsars are de-magnetized neutron stars that neither rotate nor emit electromagnetic radiation.
3. Black hole refers to the remains of a supergiant star that has collapsed into itself.
4. The effects of microgravity can be seen when astronauts and objects float in space.
Select the correct answer using the codes below.
A. 1 and 3 only
B. 3 and 4 only
C. 2, 3 and 4 only
D. 1, 2 and 4 only
Your Answer : D
Correct Answer : B
Answer Justification :
Background: China has launched its first X-ray space telescope to study black holes, pulsars and
gamma-ray bursts.
Justification: Statement 1: A supernova happens where there is a change in the core, or centre, of
a star.
A supernova burns for only a short period of time, but it can tell scientists a lot about the universe.
One kind of supernova has shown scientists that we live in an expanding universe, one that is
growing at an ever increasing rate.
Statement 2: Pulsars are highly magnetized, rotating neutron stars that emit a beam of
electromagnetic radiation.
They are spherical, compact objects that are about the size of a large city but contain more
mass than that of the sun.
Scientists are using pulsars to study extreme states of matter, search for planets beyond
Earth’s solar system and measure cosmic distance.
Statement 3: A black hole cannot be seen because of the strong gravity that is pulling all of the light
into the black hole's centre.
However, scientists can see the effects of its strong gravity on the stars and gases around it.
If a star is orbiting a certain point in space, scientists can study the star's motion to find out if
it is orbiting a black hole.
TEST - 1 (MOCK TEST - 1)
www.insightsias.com 89
© Insights Active Learning| All rights reserved. You may not reproduce, distribute or exploit the contents in any form without written
permission by copyright owner. Copyright infringers may face civil and criminal liability
Statement 4: Microgravity is the condition in which people or objects appear to be weightless. This
occurs when gravitation force around is weak (but not zero) or when objects are in a state of free
fall (as in a satellite).
For example, astronauts can move equipment weighing hundreds of pounds with their fingertips
due to microgravity.
Q Source: NASA Website and China’s telescope launch
TEST - 1 (MOCK TEST - 1)
www.insightsias.com 90
© Insights Active Learning| All rights reserved. You may not reproduce, distribute or exploit the contents in any form without written
permission by copyright owner. Copyright infringers may face civil and criminal liability
80 The highest court of appeal in the Vijayanagar Empire was
A. Gram Sabha
B. Mantri Parishad
C. Qazi’s Court
D. The King
Your Answer :
Correct Answer : D
Answer Justification :
Learning: The administration under the Vijayanagar Empire was organized in the way that the king
enjoyed absolute authority in executive, judicial and legislative matters. He was the highest court of
appeal.
The king was assisted by a council of ministers in his day to day administration. But, word of the
king was final.
Moreover, the succession to the throne was on the principle of hereditary.
Q Source: Page 212: TN 11th Standard History Textbook
TEST - 1 (MOCK TEST - 1)
www.insightsias.com 91
© Insights Active Learning| All rights reserved. You may not reproduce, distribute or exploit the contents in any form without written
permission by copyright owner. Copyright infringers may face civil and criminal liability
81 The beginning of agriculture can be ascribed most suitably to which of these times?
A. 2,500 years ago with the onset of Magadha empire
B. 4,700 years ago with the appearance of the first cities on the Indus
C. Nearly 8,000-10,000 years ago
D. About 25,000 years ago in the first Penistone glacial period
Your Answer : B
Correct Answer : C
Answer Justification :
Justification & Learning: Option D: People evolved into their current form some 200,000 years
ago (see human evolution), yet they did not begin to engage in agriculture until about
15,000–10,000 years before the present.
Wild grains were collected and eaten from at least 20,000 BC. So, D is wrong.
Option C: The first agriculture appears to have developed at the closing of the last Pleistocene
glacial period, or Ice Age (about 11,700 years ago).
From around 9,500 BC, the Neolithic founder crops such as emmer wheat, einkorn wheat, hulled
barley, peas etc were cultivated in the Levant. Rice was domesticated in China between 11,500 and
6,200 BC.
Q Source: Page 10: NCERT Class VI Our Pasts - I
TEST - 1 (MOCK TEST - 1)
www.insightsias.com 92
© Insights Active Learning| All rights reserved. You may not reproduce, distribute or exploit the contents in any form without written
permission by copyright owner. Copyright infringers may face civil and criminal liability
82 With reference to the International Tribunal for the Law of the Sea (ITLOS), consider the following:
1. It is an organ of the United Nations adjudicating disputes arising out of the interpretation and
application of the UN Convention on the Law of the Seas (UNCLOS).
2. Each state party of the ITLOS nominates a representative to become a member of ITLOS.
Which of the above is/are correct?
A. 1 only
B. 2 only
C. Both 1 and 2
D. None
Your Answer :
Correct Answer : D
Answer Justification :
Justification: Statement 1: The Tribunal is an independent judicial body but maintains close links
with the United Nations.
The Tribunal and the UN have entered into agreement concerning cooperation and relationship.
UN has granted the Tribunal observer status in the General Assembly.
Statement 2: The Tribunal is composed of 21 independent members, elected from among persons
enjoying the highest reputation for fairness and integrity and of recognized competence in the field
of the law of the sea.
Recently, law expert Neeru Chadha has become the first Indian women to elected to the
International Tribunal for the Law of the Seas (ITLOS).
Neeru has also earned a distinction of becoming the second Indian woman to occupy a top
position at the United Nations after Lakshmi Pandit who served as the President of the United
Nations General Assembly.
Q Source:
http://www.thehindu.com/news/international/neeru-chadha-becomes-1st-indian-woman-as-member-o
f-itlos/article19053500.ece
TEST - 1 (MOCK TEST - 1)
www.insightsias.com 93
© Insights Active Learning| All rights reserved. You may not reproduce, distribute or exploit the contents in any form without written
permission by copyright owner. Copyright infringers may face civil and criminal liability
83 This archaeological site consist of a number of low archaeological mounds created by generations of
superimposed mudbrick structures. Located close to the mouth of the Bolan Pass, it was abandoned
by the time of the emergence of the literate urbanised phase of the Indus Civilisation. An entry in the
UNESCO tentative list it is?
A. Lothal
B. Mehrgarh
C. Inamgaon
D. Surkotada
Your Answer :
Correct Answer : B
Answer Justification :
Learning: It was a small farming village. It is one of the earliest sites with evidence of farming and
herding in South Asia.
Mehrgarh is now seen as a precursor to the Indus Valley Civilization, displaying the whole sequence
from earliest settlement and the start of agriculture, to the mature Harappan Civilisation (based on
the above Q description of site abandonment).
Most of its archaeological deposits are buried deep beneath accumulations of alluvium.
Q Source: Improvisation: Page 29 (image): NCERT Class VI Our Pasts - I
TEST - 1 (MOCK TEST - 1)
www.insightsias.com 94
© Insights Active Learning| All rights reserved. You may not reproduce, distribute or exploit the contents in any form without written
permission by copyright owner. Copyright infringers may face civil and criminal liability
84 Integrated Child Development Services (ICDS) Scheme has a bearing on
1. Infant Mortality rate (IMR)
2. Maternal Mortality rate (MMR)
3. Reduction of malnutrition in children
4. Pre-school non-formal education
Select the correct answer using the codes below.
A. 1 and 3 only
B. 2 and 4 only
C. 1, 2 and 3 only
D. 1, 2, 3 and 4
Your Answer : C
Correct Answer : D
Answer Justification :
Justification: It is a centrally sponsored scheme implemented by the states/UTs. The Scheme was
launched in 1975.
Statement 1 and 3: The scheme has the following objectives:
to improve the nutritional and health status of children in the age-group 0-6 years; this leads
to an improvement in IMR.
to reduce mortality, morbidity, malnutrition and school dropout;
to promote policy coordination and implementation for promoting child development;
to lay an adequate foundation for the psychological, physical and social well-being of the
children etc.
Statement 2 and 4: Under the scheme, the services such as Supplementary nutrition; Pre-school
non-formal education; nutrition and health education, immunization, health check-ups; and Referral
services through Anganwadi Centres for children below 6 years of age as well as to pregnant
women& lactating mothers are offered. These improve MMR.
Learning: Infant mortality rate (IMR) is the number of deaths per 1,000 live births of children
under one year of age. It is presently around 40 in India.
The Maternal mortality rate (MMR) is the annual number of female deaths per 100,000 live births
from any cause related to or aggravated by pregnancy or its management (excluding accidental or
incidental causes). The recent World Bank data puts the MMR for India reported in 2015 at 174.
Q Source: CRY report on children in India
TEST - 1 (MOCK TEST - 1)
www.insightsias.com 95
© Insights Active Learning| All rights reserved. You may not reproduce, distribute or exploit the contents in any form without written
permission by copyright owner. Copyright infringers may face civil and criminal liability
85 The river(s) that do NOT find mention in Rigveda is?
A. Ganga and Yamuna
B. Indus and its other tributaries
C. Saraswati
D. All of the above find mention.
Your Answer : D
Correct Answer : D
Answer Justification :
Learning: Option B: There is a hymn in the form of a dialogue between sage Vishvamitra, and two
rivers, (Beas and Sutlej) that were worshipped as goddesses.
Option C: The most prominent river of the Rigveda is the Sarasvati, next to the Indus.
Option A: The Ganga and Yamuna are named only once.
Q Source: Page 45: NCERT Class VI Our Pasts - I
TEST - 1 (MOCK TEST - 1)
www.insightsias.com 96
© Insights Active Learning| All rights reserved. You may not reproduce, distribute or exploit the contents in any form without written
permission by copyright owner. Copyright infringers may face civil and criminal liability
86 There are plenty of source materials to reconstruct the history of the Gupta period. Some of the
renowned material, in this context, includes
1. Puranas throw light on the royal genealogy of the Gupta kings.
2. Devichandraguptam and Mudhrakshasam provide information regarding the rise of the Guptas
3. Meherauli Iron Pillar Inscription mention their conquests.
4. Fahien’s writings gives an account of the social, economic and religious condition of the empire.
Select the correct answer using the codes below.
A. 1 and 4 only
B. 2, 3 and 4 only
C. 1 and 3 only
D. 1, 2, 3 and 4
Your Answer : D
Correct Answer : D
Answer Justification :
Learning: S1 and S2 mention commonly known sources.
Statement 3: The Meherauli Iron Pillar inscription mentions the extensive conquests of
Chandragupta I. He is considered to be the founder of the Gupta era which starts with his accession
in A.D. 320.
The Allahabad Pillar inscription mentions that Samudragupta defeated twelve rulers in his South
Indian Expedition.
Statement 4: We will cover a separate question on Fahien’s accounts. It is important to note that his
main interest of visiting India was religious.
Q Source: Page 96-97: TN 11th Standard History Textbook
TEST - 1 (MOCK TEST - 1)
www.insightsias.com 97
© Insights Active Learning| All rights reserved. You may not reproduce, distribute or exploit the contents in any form without written
permission by copyright owner. Copyright infringers may face civil and criminal liability
87 Buddhist Viharas are
A. A monument to meditate and chant for the entire Sangha
B. Permanent shelters or monasteries for dwelling and rest
C. Divine extensions of stupas
D. Sites containing relics of Buddha
Your Answer : B
Correct Answer : B
Answer Justification :
Learning: To begin with, both Jaina and Buddhist monks went from place to place throughout the
year, teaching people.
The only time they stayed in one place was during the rainy season, when it was very difficult
to travel.
Then, their supporters built temporary shelters for them in gardens, or they lived in natural
caves in hilly areas.
As time went on, many supporters of the monks and nuns, and they themselves, felt the need
for more permanent shelters and so monasteries were built.
These were known as viharas.
Q Source: Page 71: NCERT Class VI Our Pasts - I
TEST - 1 (MOCK TEST - 1)
www.insightsias.com 98
© Insights Active Learning| All rights reserved. You may not reproduce, distribute or exploit the contents in any form without written
permission by copyright owner. Copyright infringers may face civil and criminal liability
88 PRAGATI is a unique integrating and interactive platform to
1. Address the grievances of citizens
2. Monitoring and reviewing important government projects
Which of the above is/are correct?
A. 1 only
B. 2 only
C. Both 1 and 2
D. None
Your Answer : B
Correct Answer : C
Answer Justification :
Justification: PRAGATI stands for Pro-Active Governance And Timely Implementation.
Key features of the PRAGATI application are as follows:
It is a three-tier system (PMO, Union Government Secretaries, and Chief Secretaries of the
States);
Prime Minister will hold a monthly programme where he will interact with the Government of
India Secretaries, and Chief Secretaries through Video-conferencing enabled by data and geoinformatics
visuals;
Issues to be flagged before the PM are picked up from the available database regarding
Public Grievances, on-going Programmes and pending Projects;
It will also take into consideration various correspondences to PM’s office by the common
people or from high dignitaries of States and/or developers of public projects;
Q Source: PIB AKT/NT: Release ID :167366
TEST - 1 (MOCK TEST - 1)
www.insightsias.com 99
© Insights Active Learning| All rights reserved. You may not reproduce, distribute or exploit the contents in any form without written
permission by copyright owner. Copyright infringers may face civil and criminal liability
89 In ancient Southern India, kadaisiyar and adimai were used to refer to
A. Ordinary ploughmen
B. Landless labourers, including slaves
C. Tax officials
D. Large landowners
Your Answer :
Correct Answer : B
Answer Justification :
Learning: Society was segregated even in ancient India.
There were at least three different kinds of people living in most villages in the southern and
northern parts of the ancient Indian subcontinent.
In the Tamil region, large landowners were known as vellalar, ordinary ploughmen were known as
uzhavar, and landless labourers, including slaves, were known as kadaisiyar and adimai.
In the northern region, this distinction was made in terms of land, political position and assets held.
Q Source: Page 88: NCERT Class VI Our Pasts - I
TEST - 1 (MOCK TEST - 1)
www.insightsias.com 100
© Insights Active Learning| All rights reserved. You may not reproduce, distribute or exploit the contents in any form without written
permission by copyright owner. Copyright infringers may face civil and criminal liability
90 Popularly known Jatakas were
A. Stories of previous lives of the Buddha
B. Objects of concentration
C. Donation passed from student to teacher
D. Arahants who sacrificed their lives for the service of Sangha
Your Answer : A
Correct Answer : A
Answer Justification :
Learning: They are a voluminous body of literature native to India concerning the previous births
of Gautama Buddha in both human and animal form.
The future Buddha may appear as a king, an outcast, a god, an elephant—but, in whatever
form, he exhibits some virtue that the tale thereby inculcates.
In Theravada Buddhism, the Jatakas are a textual division of the Pali Canon, included in the
Khuddaka Nikaya of the Sutta Pitaka.
The term Jataka may also refer to a traditional commentary on this book. Q Source: Page 89:
NCERT Class VI Our Pasts – I
TEST - 1 (MOCK TEST - 1)
www.insightsias.com 101
© Insights Active Learning| All rights reserved. You may not reproduce, distribute or exploit the contents in any form without written
permission by copyright owner. Copyright infringers may face civil and criminal liability
91 Consider the following statements.
1. The Anguttara Nikaya, a part of Abhidhamma Pitaka, contains thematically linked discourses of
Buddha.
2. The Majjhima Nikaya, a segment of Vinaya Pitaka, contains the smallest discourses given by the
Buddha to his close disciples.
Which of the above is/are correct?
A. 1 only
B. 2 only
C. Both 1 and 2
D. None
Your Answer :
Correct Answer : D
Answer Justification :
Justification: Both are a part of Sutta Pitaka. The word Nikaya is most commonly used in reference
to the Buddhist texts of the Sutta Piáš­aka.
Statement 1: This is incorrect because Abhidhamma Pitaka is a detailed scholastic reworking of
material appearing in the Suttas, not a collection of original discourses by the Buddha himself.
Statement 2: This is incorrect because Vinaya pitaka contains rules for the Sangha, it isn’t known
for hosting discourses.
Q Source: Improvisation: Page 47: TN 11th Standard History Textbook
TEST - 1 (MOCK TEST - 1)
www.insightsias.com 102
© Insights Active Learning| All rights reserved. You may not reproduce, distribute or exploit the contents in any form without written
permission by copyright owner. Copyright infringers may face civil and criminal liability
92 Changthangi goat of Northern Himalayan region is known for yielding
A. Pashmina wool
B. Shahtoosh
C. Himayalan Yew
D. Pitas and Humas
Your Answer : A
Correct Answer : A
Answer Justification :
Learning: They are a breed of goat inhabiting the plateaus in Tibet and neighbouring areas of
Ladakh in Jammu & Kashmir.
They are raised for ultra-fine cashmere wool Pashmina, which literally translates to "Soft Gold" in
Kashmiri.
The Changthangi goats have revitalized the poor economy of Changthang, Leh and Ladakh region.
Q Source: Page 9: NCERT Class VI Social and Political Life - I
TEST - 1 (MOCK TEST - 1)
www.insightsias.com 103
© Insights Active Learning| All rights reserved. You may not reproduce, distribute or exploit the contents in any form without written
permission by copyright owner. Copyright infringers may face civil and criminal liability
93 Consider the following about the role and contributions of B.R. Ambedkar in modern India:
1. He inspired the Dalit Buddhist Movement.
2. He was Independent India's first law minister.
3. He served as a Governor of Reserve Bank of India.
Select the correct answer using the codes below.
A. 1 and 2 only
B. 2 and 3 only
C. 3 only
D. 1 and 3 only
Your Answer :
Correct Answer : A
Answer Justification :
Justification: Statement 1: He spent his life advocating political rights and social freedom for
Dalits. In 1956 he converted to Buddhism, initiating mass conversions of Dalits.
Statement 3: The Reserve Bank of India (RBI), was based on the ideas that Ambedkar presented to
the Hilton Young Commission. He, however, did not serve as its Governor.
Ambedkar was the first Indian to pursue a doctorate in economics abroad. He argued that
industrialisation and agricultural growth could enhance the Indian economy.
Q Source: Improvisation: Page 20: NCERT Class VI Social and Political Life - I
TEST - 1 (MOCK TEST - 1)
www.insightsias.com 104
© Insights Active Learning| All rights reserved. You may not reproduce, distribute or exploit the contents in any form without written
permission by copyright owner. Copyright infringers may face civil and criminal liability
94 SATH - ‘Sustainable Action for Transforming Human capital’ initiative of NITI Aayog focuses on
A. Creating ‘role model’ states for health systems
B. Revamping higher education
C. Building workforce for the future in areas of cutting edge technology
D. Eradicating manual scavenging
Your Answer : C
Correct Answer : A
Answer Justification :
Learning: This was launched with the State Governments for furthering the agenda of cooperative
federalism.
NITI Aayog under the SATH initiative plans to identify and build three future ‘role model’
states for health systems.
After identification of three states, NITI Aayog will work in close collaboration with the state
machinery of these three states.
It will help in designing a robust roadmap, develop a program governance structure, establish
monitoring and tracking mechanisms, and provide support to the state institutions to achieve
the end objectives.
SATH program will be implemented by NITI Aayog along with McKinsey & Company and IPE
Global consortium.
Parameters such as MMR, IMR, incidence of malaria will be considered for determining
potential impact.
Q Source:
http://www.thehindu.com/business/niti-aayog-unveils-sath-programme/article18956584.ece
TEST - 1 (MOCK TEST - 1)
www.insightsias.com 105
© Insights Active Learning| All rights reserved. You may not reproduce, distribute or exploit the contents in any form without written
permission by copyright owner. Copyright infringers may face civil and criminal liability
95 The decision to locate a common well in a particular area of the village can be taken by
A. Central Government only
B. State Government only
C. District bureaucracy only
D. Panchayati Raj Institutions (PRIs)
Your Answer : D
Correct Answer : D
Answer Justification :
Learning: PRIs were established with an objective of democratic decentralization.
State legislatures can devolve powers to PRIs under 73rd constitutional amendment. Such powers
can involve management of local markets, public works, sanitation, public health etc.
Under these provisions, a public well can be established by PRIs.
Q Source: Improvisation: Page 30: NCERT Class VI Social and Political Life - I
TEST - 1 (MOCK TEST - 1)
www.insightsias.com 106
© Insights Active Learning| All rights reserved. You may not reproduce, distribute or exploit the contents in any form without written
permission by copyright owner. Copyright infringers may face civil and criminal liability
96 With reference to Buddhist disciples, consider the following.
1. Sariputta was a chief female disciple of Gautama Buddha.
2. Khema, who joined the Buddhist Sangha, was one of the queens of King Bimbisara.
3. Moggallana, a disciple of Buddha was known for his psychic powers.
4. King Ajatasatru of Magadha and King Prasenajit of Kosala became Buddha’s disciples.
Select the correct answer using the codes below.
A. 1 and 2 only
B. 3 and 4 only
C. 2, 3 and 4 only
D. 1 and 3 only
Your Answer :
Correct Answer : C
Answer Justification :
Justification: Statement 1: Sariputta was one of two chief male disciples of Gautama Buddha along
with Moggallana, counterparts to the bhikkhunis Khema and Uppalavanna, his two chief female
disciples. They were to maintain the order of monks and nuns.
Statement 2: The conversion of Khema was one of the rare cases where the Buddha used his
psychic powers to make a change in the heart of another.
Statement 3: Moggallana attained enlightenment shortly after joining the Sangha. As a teacher, he
became known for his psychic powers, which he used extensively in his teaching methods.
Statement 4: Kings like Prasenajit of Kosala and Bimbisara and Ajatasatru of Magadha too accepted
the doctrines of the Buddha and became his disciples.
Q Source: Improvisation: Page 41: TN 11th Standard History Textbook
TEST - 1 (MOCK TEST - 1)
www.insightsias.com 107
© Insights Active Learning| All rights reserved. You may not reproduce, distribute or exploit the contents in any form without written
permission by copyright owner. Copyright infringers may face civil and criminal liability
97 With reference to the International Telecommunication Union (ITU), consider the following
statements.
1. It is a non-profit non-governmental organization comprising private sector ISP representatives.
2. It releases the Global Cybersecurity Index (GCI).
3. It is responsible for allocating global radio spectrum and satellite orbits.
4. It develops the technical standards that ensure networks and technologies to seamlessly
interconnect.
5. NASSCOM is the nodal agency for coordinating with ITU from India, as a member country.
6. It strives to improve access to Information and Communication Technology among the underserved
communities worldwide.
Select the correct answer using the codes below.
A. 1, 4 and 5 only
B. 2, 3, 4 and 6 only
C. 3, 5 and 6 only
D. 1, 2, 3, 4, 5 and 6
Your Answer : D
Correct Answer : B
Answer Justification :
Justification: Statement 1: ITU is the United Nations specialised agency for information and
communication technologies (ICTs). ITU has 193 countries as its members and also 800 privatesector
entities and academic institutions.
Statement 2: India has been ranked 23rd (“maturing category” of the index ) out of 165 nations in
the second Global Cybersecurity Index (GCI) that measures the commitment of nations across the
world to cyber security.
ITU has ranked the countries based on the countries’ legal, technical and organisational
institutions, educational and research capabilities, as well as their cooperation in informationsharing
networks.
Statement 3 and 4: There must be a global agency that coordinates these standards (lie ICANN),
else telecommunication between different nations would have been difficult.
Statement 6: India is a member country since 1869 (british times), but it is the Department of
Telecommunications that is the nodal agency for such coordination.
Q Source: Release of Global Cybersecurity Index (GCI) 2017
TEST - 1 (MOCK TEST - 1)
www.insightsias.com 108
© Insights Active Learning| All rights reserved. You may not reproduce, distribute or exploit the contents in any form without written
permission by copyright owner. Copyright infringers may face civil and criminal liability
98 Firoz Tughlaq’s welfare state included which of these administrative measures?
1. Treating Hindus and Muslims alike in terms of political and economic rights
2. Abolition of the practice of slavery
3. Establishment of government department to take care of orphans and widows
Select the correct answer using the codes below.
A. 1 and 3 only
B. 2 and 3 only
C. 3 only
D. 1 and 2 only
Your Answer :
Correct Answer : C
Answer Justification :
Justification: Statement 1: As Firoz was guided by the ulemas, he was intolerant towards Shia
Muslims and Sufis. He treated Hindus as second grade citizens and imposed Jiziya. In this respect
he was the precursor of Sikandar Lodi and Aurangazeb.
Statement 2: Also he increased the number of slaves by capturing the defeated soldiers and young
persons. So, 2 is wrong.
Statement 3: A new department called Diwan-i-Khairat was created to take care of orphans and
widows. Free hospitals and marriage bureaus for poor Muslims were also established.
Q Source: Page 185-186: TN 11th Standard History Textbook
TEST - 1 (MOCK TEST - 1)
www.insightsias.com 109
© Insights Active Learning| All rights reserved. You may not reproduce, distribute or exploit the contents in any form without written
permission by copyright owner. Copyright infringers may face civil and criminal liability
99 With reference to the National Mission on Education through ICT (NMEICT), consider the following:
1. Certification of attainments of any kind at any level acquired through formal or non-formal means in
conventional or non-conventional fields
2. Platform for sharing of ideas and techniques and pooling of knowledge resources
3. Scholarship and Talent management including identification, nurturing and disbursement
electronically
Which of the above is/are the objectives of NMEICT?
A. 1 and 2 only
B. 2 only
C. 1 and 3 only
D. 1, 2 and 3
Your Answer :
Correct Answer : D
Answer Justification :
Learning: These are the guiding philosophies for NMECIT –
(a) no talent of the country should be allowed to go waste,
(b) all the services available through the content delivery portal Sakshat (NMEICT) should be free
and freely available material on the web should be used so as to avoid reinventing the wheel.
Scheme: It has been envisaged as a Centrally Sponsored Scheme to leverage the potential of ICT,
in teaching and learning process for the benefit of all the learners in Higher Education Institutions
in any time anywhere mode.
The Mission has two major components:
providing connectivity, along with provision for access devices, to institutions and learners;
Content generation.
The content portion of this Mission would have an ambitious vision of catering to the learning needs
of more than 50 crore Indians (working population) and of providing a one stop solution to all the
requirements of the learning community.
The objectives and details can be read at
http://mhrd.gov.in/sites/upload_files/mhrd/files/upload_document/MissionDocument.pdf
TEST - 1 (MOCK TEST - 1)
www.insightsias.com 110
© Insights Active Learning| All rights reserved. You may not reproduce, distribute or exploit the contents in any form without written
permission by copyright owner. Copyright infringers may face civil and criminal liability
100 India Infrastructure Fund (IIF)
1. is a SEBI-registered domestic venture capital fund
2. sponsored entirely by foreign governments
3. invests only in greenfield projects
Select the correct answer using the codes below.
A. 1 only
B. 2 and 3 only
C. 1 and 3 only
D. None of the above
Your Answer :
Correct Answer : A
Answer Justification :
Justification: Statement 1: It is focused on long-term equity investments in a diversified portfolio of
infrastructure projects.
Statement 2: IIF has been sponsored by IDFC Limited (IDFC), along with Citigroup Inc.(Citi) and
India Infrastructure Finance Company Limited (IIFCL) as founder investors.
The establishment of IIF has received the strong support of the Government of India.
Statement 3: IIF’s portfolio is expected to comprise greenfield, brownfield and operational
assets/projects in core infrastructure sub-sectors including transport etc.
The current size of IIF is INR 38 billion with investor commitments from institutional investors in
India, USA, Canada, Europe, Japan and the Middle-East.
China-led Asian Infrastructure Investment Bank (AIIB) has approved $150 million equity investment
loan to the India Infrastructure Fund. This loan will be the bank’s first equity investment to fund
private projects.
Q Source: As mentioned above (China AIIB investment)
TEST - 1 (MOCK TEST - 1)
www.insightsias.com 111
© Insights Active Learning| All rights reserved. You may not reproduce, distribute or exploit the contents in any form without written
permission by copyright owner. Copyright infringers may face civil and criminal liabilitycture Investment Bank (AIIB) has approved $150 million equity investment
loan to the India Infrastructure Fund. This loan will be the bank’s first equity investment to fund
private projects.
Q Source: As mentioned above (China AIIB investment)
TEST - 1 (MOCK TEST - 1)
www.insightsias.com 111
© Insights Active Learning| All rights reserved. You may not reproduce, distribute or exploit the contents in any form without written
permission by copyright owner. Copyright infringers may face civil and criminal liability
Correct Answer : A
Answer Justification :
Justification: Statement 1: It is focused on long-term equity investments in a diversified portfolio of
infrastructure projects.
Statement 2: IIF has been sponsored by IDFC Limited (IDFC), along with Citigroup Inc.(Citi) and
India Infrastructure Finance Company Limited (IIFCL) as founder investors.
The establishment of IIF has received the strong support of the Government of India.
Statement 3: IIF’s portfolio is expected to comprise greenfield, brownfield and operational
assets/projects in core infrastructure sub-sectors including transport etc.
The current size of IIF is INR 38 billion with investor commitments from institutional investors in
India, USA, Canada, Europe, Japan and the Middle-East.
China-led Asian Infrastructure Investment Bank (AIIB) has approved $150 million equity investment
loan to the India Infrastructure Fund. This loan will be the bank’s first equity investment to fund
private projects.
Q Source: As mentioned above (China AIIB investment)
TEST - 1 (MOCK TEST - 1)
www.insightsias.com 111
© Insights Active Learning| All rights reserved. You may not reproduce, distribute or exploit the contents in any form without written
permission by copyright owner. Copyright infringers may face civil and criminal liability

TEST - 1 (MOCK TEST - 1) www.insightsias.com 1 © Insights Active Learning| All rights reserved. You may not reproduce, distribute or expl...

Followers

Related Posts Plugin for WordPress, Blogger...